Location via proxy:   [ UP ]  
[Report a bug]   [Manage cookies]                

An analogue of Green’s functions for quasiregular maps

Mark Broderius Department of Mathematics, Northern Illinois University, Dekalb, IL 60115, USA mbroderius1@niu.edu  and  Alastair N. Fletcher Department of Mathematics, Northern Illinois University, Dekalb, IL 60115, USA afletcher@niu.edu
(Date: August 21, 2024)
Abstract.

Green’s functions are highly useful in analyzing the dynamical behavior of polynomials in their escaping set. The aim of this paper is to construct an analogue of Green’s functions for planar quasiregular mappings of degree two and constant complex dilatation. These Green’s functions are dynamically natural, in that they semi-conjugate our quasiregular mappings to the real squaring map. However, they do not share the same regularity properties as Green’s functions of polynomials. We use these Green’s functions to investigate properties of the boundary of the escaping set and give several examples to illustrate behavior that does not occur for the dynamics of quadratic polynomials.

1. Introduction

1.1. Background

The iteration theory of quadratic polynomials is a very well understood area of complex dynamics. Since the first fractal images of the Mandelbrot set appeared in the 1980s, and the work of Douady and Hubbard [6, 7] initiated a great deal of interest in this subject, there has been a consistent surge of interest. These fascinating images that arise from simply defined functions yields a surprisingly intricate theory that is still not completely settled. We refer to the texts of Beardon [1], Carleson and Gamelin [5], and Milnor [14] for introductions to complex dynamics.

The introduction of quasiconformal and quasiregular mappings into the theory of complex dynamics was another major reason for the impetus of the 1980s. Douady and Hubbard’s work on polynomial-like mappings [8] and Sullivan’s proof of the No Wandering Domains Theorem for rational maps [16] illustrated the utility of this approach. For more applications of quasiregular mappings in complex dynamics, we refer to the text of Branner and Fagella [4].

More recently, the study of the iteration theory of quasiregular mappings themselves has become an object of interest. This approach naturally extends complex dynamics into higher real dimensions. However, due to tools available only in two dimensions such as the Measurable Riemann Mapping Theorem and the Stoilow Decomposition Theorem, more can be said in the two dimensional case. For an introduction to quasiregular dynamics, we refer to Bergweiler’s survey [2] as a starting point.

The subject of our study is the class of quasiregular maps of the form

(1.1) HK,θ,c(z)=[(K+12)z+e2iθ(K12)z¯]2+c,subscript𝐻𝐾𝜃𝑐𝑧superscriptdelimited-[]𝐾12𝑧superscript𝑒2𝑖𝜃𝐾12¯𝑧2𝑐H_{K,\theta,c}(z)=\left[\left(\frac{K+1}{2}\right)z+e^{2i\theta}\left(\frac{K-% 1}{2}\right)\overline{z}\right]^{2}+c,italic_H start_POSTSUBSCRIPT italic_K , italic_θ , italic_c end_POSTSUBSCRIPT ( italic_z ) = [ ( divide start_ARG italic_K + 1 end_ARG start_ARG 2 end_ARG ) italic_z + italic_e start_POSTSUPERSCRIPT 2 italic_i italic_θ end_POSTSUPERSCRIPT ( divide start_ARG italic_K - 1 end_ARG start_ARG 2 end_ARG ) over¯ start_ARG italic_z end_ARG ] start_POSTSUPERSCRIPT 2 end_POSTSUPERSCRIPT + italic_c ,

where K>1𝐾1K>1italic_K > 1, θ(π/2,π/2]𝜃𝜋2𝜋2\theta\in(-\pi/2,\pi/2]italic_θ ∈ ( - italic_π / 2 , italic_π / 2 ] and c𝑐c\in\mathbb{C}italic_c ∈ blackboard_C. The dynamics of these mappings were first studied by the second named author and Goodman in [11], and subsequently by the second named author and Fryer in [9, 10]. We will review the important points from these papers in the preliminary section, but here let us point out that these are the simplest non-injective quasiregular mappings as they have constant complex dilatation

μHK,θ,ce2iθ(K1K+1)𝔻.subscript𝜇subscript𝐻𝐾𝜃𝑐superscript𝑒2𝑖𝜃𝐾1𝐾1𝔻\mu_{H_{K,\theta,c}}\equiv e^{2i\theta}\left(\frac{K-1}{K+1}\right)\in\mathbb{% D}.italic_μ start_POSTSUBSCRIPT italic_H start_POSTSUBSCRIPT italic_K , italic_θ , italic_c end_POSTSUBSCRIPT end_POSTSUBSCRIPT ≡ italic_e start_POSTSUPERSCRIPT 2 italic_i italic_θ end_POSTSUPERSCRIPT ( divide start_ARG italic_K - 1 end_ARG start_ARG italic_K + 1 end_ARG ) ∈ blackboard_D .

It is evident that we can write

HK,θ,c=PchK,θ,subscript𝐻𝐾𝜃𝑐subscript𝑃𝑐subscript𝐾𝜃H_{K,\theta,c}=P_{c}\circ h_{K,\theta},italic_H start_POSTSUBSCRIPT italic_K , italic_θ , italic_c end_POSTSUBSCRIPT = italic_P start_POSTSUBSCRIPT italic_c end_POSTSUBSCRIPT ∘ italic_h start_POSTSUBSCRIPT italic_K , italic_θ end_POSTSUBSCRIPT ,

where Pc(z)=z2+csubscript𝑃𝑐𝑧superscript𝑧2𝑐P_{c}(z)=z^{2}+citalic_P start_POSTSUBSCRIPT italic_c end_POSTSUBSCRIPT ( italic_z ) = italic_z start_POSTSUPERSCRIPT 2 end_POSTSUPERSCRIPT + italic_c and hK,θsubscript𝐾𝜃h_{K,\theta}italic_h start_POSTSUBSCRIPT italic_K , italic_θ end_POSTSUBSCRIPT is the affine map that stretches by a factor K>1𝐾1K>1italic_K > 1 in the direction eiθsuperscript𝑒𝑖𝜃e^{i\theta}italic_e start_POSTSUPERSCRIPT italic_i italic_θ end_POSTSUPERSCRIPT. This Stoilow decomposition of HK,θ,csubscript𝐻𝐾𝜃𝑐H_{K,\theta,c}italic_H start_POSTSUBSCRIPT italic_K , italic_θ , italic_c end_POSTSUBSCRIPT is also a characterization of degree two quasiregular maps in \mathbb{C}blackboard_C with constant complex dilatation: every such map is conformally conjugate to one of the form (1.1). Observe that if K=1𝐾1K=1italic_K = 1, then we return to the case of quadratic polynomials. One of the goals of this paper is to illustrate new phenomena that can occur in this setting, when compared to complex dynamics. Figure 1 gives an example where the function has a saddle fixed point, a situation that cannot occur for holomorphic functions.

Refer to caption
Figure 1. The dynamical plane for H5,0,0.1subscript𝐻500.1H_{5,0,-0.1}italic_H start_POSTSUBSCRIPT 5 , 0 , - 0.1 end_POSTSUBSCRIPT with the saddle fixed point marked.

1.2. Quadratic polynomials

We briefly review some of the well-known material from the dynamics of quadratic polynomials to set the stage for what is to come. We recall that every quadratic polynomial is linearly conjugate to one of the form Pc(z)=z2+csubscript𝑃𝑐𝑧superscript𝑧2𝑐P_{c}(z)=z^{2}+citalic_P start_POSTSUBSCRIPT italic_c end_POSTSUBSCRIPT ( italic_z ) = italic_z start_POSTSUPERSCRIPT 2 end_POSTSUPERSCRIPT + italic_c.

Given a non-injective polynomial P𝑃Pitalic_P, \mathbb{C}blackboard_C can be decomposed into the escaping set

I(P)={z:Pn(z)}𝐼𝑃conditional-set𝑧superscript𝑃𝑛𝑧I(P)=\{z\in\mathbb{C}:P^{n}(z)\to\infty\}italic_I ( italic_P ) = { italic_z ∈ blackboard_C : italic_P start_POSTSUPERSCRIPT italic_n end_POSTSUPERSCRIPT ( italic_z ) → ∞ }

and the bounded orbit set

BO(P)={z: there exists M>0 such that |Pn(z)|M for all n}.𝐵𝑂𝑃conditional-set𝑧 there exists 𝑀0 such that superscript𝑃𝑛𝑧𝑀 for all 𝑛BO(P)=\{z\in\mathbb{C}:\text{ there exists }M>0\text{ such that }|P^{n}(z)|% \leq M\text{ for all }n\in\mathbb{N}\}.italic_B italic_O ( italic_P ) = { italic_z ∈ blackboard_C : there exists italic_M > 0 such that | italic_P start_POSTSUPERSCRIPT italic_n end_POSTSUPERSCRIPT ( italic_z ) | ≤ italic_M for all italic_n ∈ blackboard_N } .

The bounded orbit set is often called the filled Julia set in the literature, and denoted by K(P)𝐾𝑃K(P)italic_K ( italic_P ), but we will reserve the use of K𝐾Kitalic_K for the maximal dilatation of quasiregular mappings.

The chaotic set is called the Julia set, and denoted by J(P)𝐽𝑃J(P)italic_J ( italic_P ), whereas the stable set is called the Fatou set, and denoted by F(P)𝐹𝑃F(P)italic_F ( italic_P ). Slightly more formally, the Fatou set is where the family of iterates locally forms a normal family, and the Julia set is the complement of the Fatou set. The importance of the escaping set is that the definition of the Julia set is a difficult one to check, but the fact that

J(P)=I(P)𝐽𝑃𝐼𝑃J(P)=\partial I(P)italic_J ( italic_P ) = ∂ italic_I ( italic_P )

gives a highly useful way to visualize the Julia set. Critical fixed points of polynomials are called superattracting fixed points and are necessarily in the Fatou set. In particular, if we extend P𝑃Pitalic_P to the Riemann sphere subscript\mathbb{C}_{\infty}blackboard_C start_POSTSUBSCRIPT ∞ end_POSTSUBSCRIPT, then we see that \infty is a superattracting fixed point of P𝑃Pitalic_P.

For quadratic polynomials, we have a dichotomy in the dynamical behavior. If 0BO(P)0𝐵𝑂𝑃0\in BO(P)0 ∈ italic_B italic_O ( italic_P ), then both BO(P)𝐵𝑂𝑃BO(P)italic_B italic_O ( italic_P ) and J(P)𝐽𝑃J(P)italic_J ( italic_P ) are connected. On the other hand, if 0I(P)0𝐼𝑃0\in I(P)0 ∈ italic_I ( italic_P ) then both BO(P)𝐵𝑂𝑃BO(P)italic_B italic_O ( italic_P ) and J(P)𝐽𝑃J(P)italic_J ( italic_P ) are a Cantor set, that is, a totally disconnected, compact, perfect set. These distinctive behaviors give rise to the definition of the Mandelbrot set in parameter space:

={c:0BO(Pc)}.conditional-set𝑐0𝐵𝑂subscript𝑃𝑐\mathcal{M}=\{c\in\mathbb{C}:0\in BO(P_{c})\}.caligraphic_M = { italic_c ∈ blackboard_C : 0 ∈ italic_B italic_O ( italic_P start_POSTSUBSCRIPT italic_c end_POSTSUBSCRIPT ) } .

Böttcher’s Theorem allows us to conformally conjugate near a superattracting fixed point of local index d𝑑ditalic_d to the power map zzdmaps-to𝑧superscript𝑧𝑑z\mapsto z^{d}italic_z ↦ italic_z start_POSTSUPERSCRIPT italic_d end_POSTSUPERSCRIPT. For quadratic polynomials, as \infty is a superattracting fixed point, we may conformally conjugate to zz2maps-to𝑧superscript𝑧2z\mapsto z^{2}italic_z ↦ italic_z start_POSTSUPERSCRIPT 2 end_POSTSUPERSCRIPT in a neighborhood of \infty. More precisely, there exists a conformal map φcsubscript𝜑𝑐\varphi_{c}italic_φ start_POSTSUBSCRIPT italic_c end_POSTSUBSCRIPT and a neighborhood Wcsubscript𝑊𝑐W_{c}italic_W start_POSTSUBSCRIPT italic_c end_POSTSUBSCRIPT of \infty such that

(1.2) φc(Pc(z))=[φc(z)]2subscript𝜑𝑐subscript𝑃𝑐𝑧superscriptdelimited-[]subscript𝜑𝑐𝑧2\varphi_{c}(P_{c}(z))=\left[\varphi_{c}(z)\right]^{2}italic_φ start_POSTSUBSCRIPT italic_c end_POSTSUBSCRIPT ( italic_P start_POSTSUBSCRIPT italic_c end_POSTSUBSCRIPT ( italic_z ) ) = [ italic_φ start_POSTSUBSCRIPT italic_c end_POSTSUBSCRIPT ( italic_z ) ] start_POSTSUPERSCRIPT 2 end_POSTSUPERSCRIPT

for zWc𝑧subscript𝑊𝑐z\in W_{c}italic_z ∈ italic_W start_POSTSUBSCRIPT italic_c end_POSTSUBSCRIPT. It is desirable to extend (1.2) to the largest possible domain. The only obstruction to extending Wcsubscript𝑊𝑐W_{c}italic_W start_POSTSUBSCRIPT italic_c end_POSTSUBSCRIPT to larger domains is if the orbit of the critical point at 00 escapes. Therefore, if 000\in\mathcal{M}0 ∈ caligraphic_M, then (1.2) holds on all of I(Pc)𝐼subscript𝑃𝑐I(P_{c})italic_I ( italic_P start_POSTSUBSCRIPT italic_c end_POSTSUBSCRIPT ), whereas if 000\notin\mathcal{M}0 ∉ caligraphic_M, then we cannot do so.

A resolution to this problem is to use harmonic Green’s functions. The real valued function

Gc(z)=log|φc(z)|subscript𝐺𝑐𝑧subscript𝜑𝑐𝑧G_{c}(z)=\log|\varphi_{c}(z)|italic_G start_POSTSUBSCRIPT italic_c end_POSTSUBSCRIPT ( italic_z ) = roman_log | italic_φ start_POSTSUBSCRIPT italic_c end_POSTSUBSCRIPT ( italic_z ) |

satisfies the functional equation

(1.3) Gc(Pc(z))=2Gc(z)subscript𝐺𝑐subscript𝑃𝑐𝑧2subscript𝐺𝑐𝑧G_{c}(P_{c}(z))=2G_{c}(z)italic_G start_POSTSUBSCRIPT italic_c end_POSTSUBSCRIPT ( italic_P start_POSTSUBSCRIPT italic_c end_POSTSUBSCRIPT ( italic_z ) ) = 2 italic_G start_POSTSUBSCRIPT italic_c end_POSTSUBSCRIPT ( italic_z )

initially in Wcsubscript𝑊𝑐W_{c}italic_W start_POSTSUBSCRIPT italic_c end_POSTSUBSCRIPT. This time the functional equation allows us to extend the domain of definition of Gcsubscript𝐺𝑐G_{c}italic_G start_POSTSUBSCRIPT italic_c end_POSTSUBSCRIPT to all of I(Pc)𝐼subscript𝑃𝑐I(P_{c})italic_I ( italic_P start_POSTSUBSCRIPT italic_c end_POSTSUBSCRIPT ), and it turns out that Gcsubscript𝐺𝑐G_{c}italic_G start_POSTSUBSCRIPT italic_c end_POSTSUBSCRIPT is precisely the Green’s function for the exterior domain of BO(Pc)𝐵𝑂subscript𝑃𝑐BO(P_{c})italic_B italic_O ( italic_P start_POSTSUBSCRIPT italic_c end_POSTSUBSCRIPT ) with a pole at \infty. In particular, Gcsubscript𝐺𝑐G_{c}italic_G start_POSTSUBSCRIPT italic_c end_POSTSUBSCRIPT is harmonic and Gc(z)0subscript𝐺𝑐𝑧0G_{c}(z)\to 0italic_G start_POSTSUBSCRIPT italic_c end_POSTSUBSCRIPT ( italic_z ) → 0 as dist(z,J(Pc))0dist𝑧𝐽subscript𝑃𝑐0\operatorname{dist}(z,J(P_{c}))\to 0roman_dist ( italic_z , italic_J ( italic_P start_POSTSUBSCRIPT italic_c end_POSTSUBSCRIPT ) ) → 0.

The equipotentials Gc(z)=tsubscript𝐺𝑐𝑧𝑡G_{c}(z)=titalic_G start_POSTSUBSCRIPT italic_c end_POSTSUBSCRIPT ( italic_z ) = italic_t give a particularly striking picture of the dynamics of Pcsubscript𝑃𝑐P_{c}italic_P start_POSTSUBSCRIPT italic_c end_POSTSUBSCRIPT. If c𝑐c\in\mathcal{M}italic_c ∈ caligraphic_M, then these equipotentials give a foliation of I(Pc)𝐼subscript𝑃𝑐I(P_{c})italic_I ( italic_P start_POSTSUBSCRIPT italic_c end_POSTSUBSCRIPT ) through simple closed curves. On the other hand, if c𝑐c\notin\mathcal{M}italic_c ∉ caligraphic_M, then while the equipotentials for t𝑡titalic_t large enough are simple closed curves, there is a critical value given by Gc(0)=t0subscript𝐺𝑐0subscript𝑡0G_{c}(0)=t_{0}italic_G start_POSTSUBSCRIPT italic_c end_POSTSUBSCRIPT ( 0 ) = italic_t start_POSTSUBSCRIPT 0 end_POSTSUBSCRIPT for which the level curve forms a figure-eight shape. For 0<t<t00𝑡subscript𝑡00<t<t_{0}0 < italic_t < italic_t start_POSTSUBSCRIPT 0 end_POSTSUBSCRIPT, the level curves are then disconnected.

1.3. Statement of results

For mappings of the form (1.1), the plane can be again decomposed into the escaping set and the bounded orbit set.

In [9], an analogous result to Böttcher’s Theorem is established with gives the existence of a quasiconformal map φK,θ,csubscript𝜑𝐾𝜃𝑐\varphi_{K,\theta,c}italic_φ start_POSTSUBSCRIPT italic_K , italic_θ , italic_c end_POSTSUBSCRIPT which conjugates HK,θ,csubscript𝐻𝐾𝜃𝑐H_{K,\theta,c}italic_H start_POSTSUBSCRIPT italic_K , italic_θ , italic_c end_POSTSUBSCRIPT to HK,θ,0subscript𝐻𝐾𝜃0H_{K,\theta,0}italic_H start_POSTSUBSCRIPT italic_K , italic_θ , 0 end_POSTSUBSCRIPT on a neighborhood WK,θ,csubscript𝑊𝐾𝜃𝑐W_{K,\theta,c}italic_W start_POSTSUBSCRIPT italic_K , italic_θ , italic_c end_POSTSUBSCRIPT of \infty contained in the escaping set, that is,

φK,θ,c(HK,θ,c(z))=HK,θ,0(φK,θ,c(z))subscript𝜑𝐾𝜃𝑐subscript𝐻𝐾𝜃𝑐𝑧subscript𝐻𝐾𝜃0subscript𝜑𝐾𝜃𝑐𝑧\varphi_{K,\theta,c}(H_{K,\theta,c}(z))=H_{K,\theta,0}(\varphi_{K,\theta,c}(z))italic_φ start_POSTSUBSCRIPT italic_K , italic_θ , italic_c end_POSTSUBSCRIPT ( italic_H start_POSTSUBSCRIPT italic_K , italic_θ , italic_c end_POSTSUBSCRIPT ( italic_z ) ) = italic_H start_POSTSUBSCRIPT italic_K , italic_θ , 0 end_POSTSUBSCRIPT ( italic_φ start_POSTSUBSCRIPT italic_K , italic_θ , italic_c end_POSTSUBSCRIPT ( italic_z ) )

for zWK,θ,c𝑧subscript𝑊𝐾𝜃𝑐z\in W_{K,\theta,c}italic_z ∈ italic_W start_POSTSUBSCRIPT italic_K , italic_θ , italic_c end_POSTSUBSCRIPT. The question then arises of whether this conjugation can be extended. Once again, the Mandelbrot set in parameter space plays an important role. For K>1𝐾1K>1italic_K > 1 and θ(π/2,π/2]𝜃𝜋2𝜋2\theta\in(-\pi/2,\pi/2]italic_θ ∈ ( - italic_π / 2 , italic_π / 2 ], we define the Mandelbrot set

K,θ={c:0BO(HK,θ,c)}.subscript𝐾𝜃conditional-set𝑐0𝐵𝑂subscript𝐻𝐾𝜃𝑐\mathcal{M}_{K,\theta}=\{c\in\mathbb{C}:0\in BO(H_{K,\theta,c})\}.caligraphic_M start_POSTSUBSCRIPT italic_K , italic_θ end_POSTSUBSCRIPT = { italic_c ∈ blackboard_C : 0 ∈ italic_B italic_O ( italic_H start_POSTSUBSCRIPT italic_K , italic_θ , italic_c end_POSTSUBSCRIPT ) } .

We note that this time there is not an equivalent formulation in terms of the connectedness of the Julia set, as J(H)𝐽𝐻J(H)italic_J ( italic_H ) and I(H)𝐼𝐻\partial I(H)∂ italic_I ( italic_H ) may differ for quasiregular maps, see for example [3, Example 7.3].

Returning to the Böttcher coordinate, it was shown in [9, Theorem 2.4] that if cK,θ𝑐subscript𝐾𝜃c\in\mathcal{M}_{K,\theta}italic_c ∈ caligraphic_M start_POSTSUBSCRIPT italic_K , italic_θ end_POSTSUBSCRIPT, then φK,θ,csubscript𝜑𝐾𝜃𝑐\varphi_{K,\theta,c}italic_φ start_POSTSUBSCRIPT italic_K , italic_θ , italic_c end_POSTSUBSCRIPT may be extended to a locally quasiconformal map on all of I(HK,θ,c)𝐼subscript𝐻𝐾𝜃𝑐I(H_{K,\theta,c})italic_I ( italic_H start_POSTSUBSCRIPT italic_K , italic_θ , italic_c end_POSTSUBSCRIPT ), whereas if cK,θ𝑐subscript𝐾𝜃c\notin\mathcal{M}_{K,\theta}italic_c ∉ caligraphic_M start_POSTSUBSCRIPT italic_K , italic_θ end_POSTSUBSCRIPT, then it cannot. Our first main result shows that the Green’s function idea also works in this setting.

Theorem 1.1.

Let K>1𝐾1K>1italic_K > 1, θ(π/2,π/2]𝜃𝜋2𝜋2\theta\in(-\pi/2,\pi/2]italic_θ ∈ ( - italic_π / 2 , italic_π / 2 ] and c𝑐c\in\mathbb{C}italic_c ∈ blackboard_C. There exists a non-negative, continuous function GK,θ,c::subscript𝐺𝐾𝜃𝑐G_{K,\theta,c}:\mathbb{C}\to\mathbb{R}italic_G start_POSTSUBSCRIPT italic_K , italic_θ , italic_c end_POSTSUBSCRIPT : blackboard_C → blackboard_R that is identically zero on BO(HK,θ,c)𝐵𝑂subscript𝐻𝐾𝜃𝑐BO(H_{K,\theta,c})italic_B italic_O ( italic_H start_POSTSUBSCRIPT italic_K , italic_θ , italic_c end_POSTSUBSCRIPT ), non-zero on I(HK,θ,c)𝐼subscript𝐻𝐾𝜃𝑐I(H_{K,\theta,c})italic_I ( italic_H start_POSTSUBSCRIPT italic_K , italic_θ , italic_c end_POSTSUBSCRIPT ) and such that

GK,θ,c(HK,θ,c(z))=2GK,θ,c(z)subscript𝐺𝐾𝜃𝑐subscript𝐻𝐾𝜃𝑐𝑧2subscript𝐺𝐾𝜃𝑐𝑧G_{K,\theta,c}(H_{K,\theta,c}(z))=2G_{K,\theta,c}(z)italic_G start_POSTSUBSCRIPT italic_K , italic_θ , italic_c end_POSTSUBSCRIPT ( italic_H start_POSTSUBSCRIPT italic_K , italic_θ , italic_c end_POSTSUBSCRIPT ( italic_z ) ) = 2 italic_G start_POSTSUBSCRIPT italic_K , italic_θ , italic_c end_POSTSUBSCRIPT ( italic_z )

for all z𝑧z\in\mathbb{C}italic_z ∈ blackboard_C.

We do not expect GK,θ,csubscript𝐺𝐾𝜃𝑐G_{K,\theta,c}italic_G start_POSTSUBSCRIPT italic_K , italic_θ , italic_c end_POSTSUBSCRIPT to be harmonic, although we leave the question of the regularity of this function to future work. However, GK,θ,csubscript𝐺𝐾𝜃𝑐G_{K,\theta,c}italic_G start_POSTSUBSCRIPT italic_K , italic_θ , italic_c end_POSTSUBSCRIPT still has equipotentials that are useful dynamically. For t>0𝑡0t>0italic_t > 0, let us define

E(t)=EK,θ,c(t)={z:GK,θ,c(z)=t},𝐸𝑡subscript𝐸𝐾𝜃𝑐𝑡conditional-set𝑧subscript𝐺𝐾𝜃𝑐𝑧𝑡E(t)=E_{K,\theta,c}(t)=\{z\in\mathbb{C}:G_{K,\theta,c}(z)=t\},italic_E ( italic_t ) = italic_E start_POSTSUBSCRIPT italic_K , italic_θ , italic_c end_POSTSUBSCRIPT ( italic_t ) = { italic_z ∈ blackboard_C : italic_G start_POSTSUBSCRIPT italic_K , italic_θ , italic_c end_POSTSUBSCRIPT ( italic_z ) = italic_t } ,

and

U(t)=UK,θ,c(t)={z:GK,θ,c(z)>t}.𝑈𝑡subscript𝑈𝐾𝜃𝑐𝑡conditional-set𝑧subscript𝐺𝐾𝜃𝑐𝑧𝑡U(t)=U_{K,\theta,c}(t)=\{z\in\mathbb{C}:G_{K,\theta,c}(z)>t\}.italic_U ( italic_t ) = italic_U start_POSTSUBSCRIPT italic_K , italic_θ , italic_c end_POSTSUBSCRIPT ( italic_t ) = { italic_z ∈ blackboard_C : italic_G start_POSTSUBSCRIPT italic_K , italic_θ , italic_c end_POSTSUBSCRIPT ( italic_z ) > italic_t } .

Using these notions, we will give an alternative proof and mild refinement of [11, Theorem 5.3 and 5.4].

Theorem 1.2.

Let K>1𝐾1K>1italic_K > 1, θ(π/2,π/2]𝜃𝜋2𝜋2\theta\in(-\pi/2,\pi/2]italic_θ ∈ ( - italic_π / 2 , italic_π / 2 ] and c𝑐c\in\mathbb{C}italic_c ∈ blackboard_C. If cK,θ𝑐subscript𝐾𝜃c\in\mathcal{M}_{K,\theta}italic_c ∈ caligraphic_M start_POSTSUBSCRIPT italic_K , italic_θ end_POSTSUBSCRIPT, then I(HK,θ,c)𝐼subscript𝐻𝐾𝜃𝑐\partial I(H_{K,\theta,c})∂ italic_I ( italic_H start_POSTSUBSCRIPT italic_K , italic_θ , italic_c end_POSTSUBSCRIPT ) is connected. If cK,θ𝑐subscript𝐾𝜃c\notin\mathcal{M}_{K,\theta}italic_c ∉ caligraphic_M start_POSTSUBSCRIPT italic_K , italic_θ end_POSTSUBSCRIPT, then I(HK,θ,c)𝐼subscript𝐻𝐾𝜃𝑐\partial I(H_{K,\theta,c})∂ italic_I ( italic_H start_POSTSUBSCRIPT italic_K , italic_θ , italic_c end_POSTSUBSCRIPT ) has uncountably many components.

Recalling the dichotomy that J(Pc)𝐽subscript𝑃𝑐J(P_{c})italic_J ( italic_P start_POSTSUBSCRIPT italic_c end_POSTSUBSCRIPT ) is either connected or a Cantor set, we now turn to the question of what happens when I(HK,θ,c)𝐼subscript𝐻𝐾𝜃𝑐\partial I(H_{K,\theta,c})∂ italic_I ( italic_H start_POSTSUBSCRIPT italic_K , italic_θ , italic_c end_POSTSUBSCRIPT ) has uncountably many components. First, we give a condition that ensures that I(HK,θ,c)𝐼subscript𝐻𝐾𝜃𝑐\partial I(H_{K,\theta,c})∂ italic_I ( italic_H start_POSTSUBSCRIPT italic_K , italic_θ , italic_c end_POSTSUBSCRIPT ) is not a Cantor set. Recall that a periodic point of period n𝑛nitalic_n for f𝑓fitalic_f is a solution of fn(z)=zsuperscript𝑓𝑛𝑧𝑧f^{n}(z)=zitalic_f start_POSTSUPERSCRIPT italic_n end_POSTSUPERSCRIPT ( italic_z ) = italic_z. Note that a fixed point is considered a periodic point of all periods.

Theorem 1.3.

Let K>1𝐾1K>1italic_K > 1, θ(π/2,π/2]𝜃𝜋2𝜋2\theta\in(-\pi/2,\pi/2]italic_θ ∈ ( - italic_π / 2 , italic_π / 2 ] and cK,θ𝑐subscript𝐾𝜃c\notin\mathcal{M}_{K,\theta}italic_c ∉ caligraphic_M start_POSTSUBSCRIPT italic_K , italic_θ end_POSTSUBSCRIPT. Suppose there exists n𝑛n\in\mathbb{N}italic_n ∈ blackboard_N such that HK,θ,csubscript𝐻𝐾𝜃𝑐H_{K,\theta,c}italic_H start_POSTSUBSCRIPT italic_K , italic_θ , italic_c end_POSTSUBSCRIPT has at least 2n+1superscript2𝑛12^{n}+12 start_POSTSUPERSCRIPT italic_n end_POSTSUPERSCRIPT + 1 periodic points of period n𝑛nitalic_n. Then I(HK,θ,c)𝐼subscript𝐻𝐾𝜃𝑐\partial I(H_{K,\theta,c})∂ italic_I ( italic_H start_POSTSUBSCRIPT italic_K , italic_θ , italic_c end_POSTSUBSCRIPT ) is not a Cantor set.

Finally, we aim to illustrate that the situation in Theorem 1.3 can indeed occur and exhibit some of the other features that can occur for the dynamics of these mappings that contrast with the dynamics of quadratic polynomials.

Theorem 1.4.

We may choose parameters K>1𝐾1K>1italic_K > 1, θ(π/2,π/2]𝜃𝜋2𝜋2\theta\in(-\pi/2,\pi/2]italic_θ ∈ ( - italic_π / 2 , italic_π / 2 ] and c𝑐c\in\mathbb{C}italic_c ∈ blackboard_C such that each of the following cases may occur:

  1. (a)

    HK,θ,csubscript𝐻𝐾𝜃𝑐H_{K,\theta,c}italic_H start_POSTSUBSCRIPT italic_K , italic_θ , italic_c end_POSTSUBSCRIPT has either two, three or four fixed points in \mathbb{C}blackboard_C, and four is the maximum possible.

  2. (b)

    HK,θ,csubscript𝐻𝐾𝜃𝑐H_{K,\theta,c}italic_H start_POSTSUBSCRIPT italic_K , italic_θ , italic_c end_POSTSUBSCRIPT has an attracting fixed, yet cK,θ𝑐subscript𝐾𝜃c\notin\mathcal{M}_{K,\theta}italic_c ∉ caligraphic_M start_POSTSUBSCRIPT italic_K , italic_θ end_POSTSUBSCRIPT, that is, there is an attracting fixed point in one of uncountably many components of the bounded orbit set.

  3. (c)

    Given K,θ𝐾𝜃K,\thetaitalic_K , italic_θ, there exist parameters c𝑐c\in\mathbb{C}italic_c ∈ blackboard_C such that HK,θ,csubscript𝐻𝐾𝜃𝑐H_{K,\theta,c}italic_H start_POSTSUBSCRIPT italic_K , italic_θ , italic_c end_POSTSUBSCRIPT has a saddle fixed point. Moreover, there exist parameters K,θ,c𝐾𝜃𝑐K,\theta,citalic_K , italic_θ , italic_c such that HK,θ,csubscript𝐻𝐾𝜃𝑐H_{K,\theta,c}italic_H start_POSTSUBSCRIPT italic_K , italic_θ , italic_c end_POSTSUBSCRIPT has a saddle fixed point z0subscript𝑧0z_{0}italic_z start_POSTSUBSCRIPT 0 end_POSTSUBSCRIPT, and the intersection of the component of BO(HK,θ,c)𝐵𝑂subscript𝐻𝐾𝜃𝑐BO(H_{K,\theta,c})italic_B italic_O ( italic_H start_POSTSUBSCRIPT italic_K , italic_θ , italic_c end_POSTSUBSCRIPT ) containing z0subscript𝑧0z_{0}italic_z start_POSTSUBSCRIPT 0 end_POSTSUBSCRIPT with an open neighbourhood of z0subscript𝑧0z_{0}italic_z start_POSTSUBSCRIPT 0 end_POSTSUBSCRIPT is a smooth curve.

Refer to caption
Figure 2. The dynamical plane of H1/2,0,3/2i/2subscript𝐻12032𝑖2H_{1/2,0,-3/2-i/2}italic_H start_POSTSUBSCRIPT 1 / 2 , 0 , - 3 / 2 - italic_i / 2 end_POSTSUBSCRIPT with the attracting fixed point z0subscript𝑧0z_{0}italic_z start_POSTSUBSCRIPT 0 end_POSTSUBSCRIPT marked.

Figure 2 illustrates an example of the second case in Theorem 1.4, and Figure 1 illustrates an example of the third case.

The paper is organized as follows. In Section 2, we recall preliminary material on quasiregular mappings in the plane, and known material on the dynamics of the mappings HK,θ,csubscript𝐻𝐾𝜃𝑐H_{K,\theta,c}italic_H start_POSTSUBSCRIPT italic_K , italic_θ , italic_c end_POSTSUBSCRIPT. In Section 3 we prove Theorem 1.1 by constructing our analogue of a Green’s function. In Section 4 we use equipotentials for our Green’s functions to establish Theorem 1.2. In Section 5 we prove Theorem 1.3. In Section 6, we give a classification of the type of fixed point of HK,0,csubscript𝐻𝐾0𝑐H_{K,0,c}italic_H start_POSTSUBSCRIPT italic_K , 0 , italic_c end_POSTSUBSCRIPT based on its location and finally in Section 7 we use the classification to help establish the examples in Theorem 1.4.

2. Preliminaries

2.1. Quasiregular mappings

A quasiconformal mapping f::𝑓f:\mathbb{C}\to\mathbb{C}italic_f : blackboard_C → blackboard_C is a homeomorphism such that f𝑓fitalic_f is in the Sobolev space W2,loc1()subscriptsuperscript𝑊12𝑙𝑜𝑐W^{1}_{2,loc}(\mathbb{C})italic_W start_POSTSUPERSCRIPT 1 end_POSTSUPERSCRIPT start_POSTSUBSCRIPT 2 , italic_l italic_o italic_c end_POSTSUBSCRIPT ( blackboard_C ) and there exists k[0,1)𝑘01k\in[0,1)italic_k ∈ [ 0 , 1 ) such that the complex dilatation μf=fz¯/fzsubscript𝜇𝑓subscript𝑓¯𝑧subscript𝑓𝑧\mu_{f}=f_{\overline{z}}/f_{z}italic_μ start_POSTSUBSCRIPT italic_f end_POSTSUBSCRIPT = italic_f start_POSTSUBSCRIPT over¯ start_ARG italic_z end_ARG end_POSTSUBSCRIPT / italic_f start_POSTSUBSCRIPT italic_z end_POSTSUBSCRIPT satisfies

|μf(z)|ksubscript𝜇𝑓𝑧𝑘|\mu_{f}(z)|\leq k| italic_μ start_POSTSUBSCRIPT italic_f end_POSTSUBSCRIPT ( italic_z ) | ≤ italic_k

almost everywhere in \mathbb{C}blackboard_C. The dilatation at z𝑧z\in\mathbb{C}italic_z ∈ blackboard_C is

Kf(z):=1+|μf(z)|1|μf(z)|.assignsubscript𝐾𝑓𝑧1subscript𝜇𝑓𝑧1subscript𝜇𝑓𝑧K_{f}(z):=\frac{1+|\mu_{f}(z)|}{1-|\mu_{f}(z)|}.italic_K start_POSTSUBSCRIPT italic_f end_POSTSUBSCRIPT ( italic_z ) := divide start_ARG 1 + | italic_μ start_POSTSUBSCRIPT italic_f end_POSTSUBSCRIPT ( italic_z ) | end_ARG start_ARG 1 - | italic_μ start_POSTSUBSCRIPT italic_f end_POSTSUBSCRIPT ( italic_z ) | end_ARG .

A mapping is called K𝐾Kitalic_K-quasiconformal if Kf(z)Ksubscript𝐾𝑓𝑧𝐾K_{f}(z)\leq Kitalic_K start_POSTSUBSCRIPT italic_f end_POSTSUBSCRIPT ( italic_z ) ≤ italic_K almost everywhere. The smallest such constant is called the maximal dilatation and denoted by K(f)𝐾𝑓K(f)italic_K ( italic_f ). If we drop the assumption on injectivity, then f𝑓fitalic_f is called a quasiregular mapping. We refer to the books of Rickman [15] and Iwaniec and Martin [13] for much more on the development of the theory of quasiregular mappings.

In the plane, we have the following two crucial results. First, there is a surprising correspondence between quasiconformal mappings and measurable functions, see for example [13, p.8].

Theorem 2.1 (Measurable Riemann Mapping Theorem).

Suppose that μL()𝜇superscript𝐿\mu\in L^{\infty}(\mathbb{C})italic_μ ∈ italic_L start_POSTSUPERSCRIPT ∞ end_POSTSUPERSCRIPT ( blackboard_C ) with μk<1subscriptnorm𝜇𝑘1||\mu||_{\infty}\leq k<1| | italic_μ | | start_POSTSUBSCRIPT ∞ end_POSTSUBSCRIPT ≤ italic_k < 1. Then there exists a quasiconformal map f::𝑓f:\mathbb{C}\to\mathbb{C}italic_f : blackboard_C → blackboard_C with complex dilatation equal to μ𝜇\muitalic_μ almost everywhere. Moreover, f𝑓fitalic_f is unique if it fixes 0,1010,10 , 1 and \infty.

Moreover, every quasiregular mapping has an important decomposition, see for example [13, p.254].

Theorem 2.2 (Stoilow Decomposition Theorem).

Let f::𝑓f:\mathbb{C}\to\mathbb{C}italic_f : blackboard_C → blackboard_C be a quasiregular mapping. Then there exists a holomorphic function g𝑔gitalic_g and a quasiconformal mapping hhitalic_h such that f=gh𝑓𝑔f=g\circ hitalic_f = italic_g ∘ italic_h.

In fact, in the plane, some sources use this decomposition as the definition of a quasiregular mapping. However, this approach does not generalize to higher dimensions, so even though this paper is two dimensional, we will keep Stoilow Decomposition as a theorem.

2.2. The mappings HK,θ,csubscript𝐻𝐾𝜃𝑐H_{K,\theta,c}italic_H start_POSTSUBSCRIPT italic_K , italic_θ , italic_c end_POSTSUBSCRIPT

Here we review the properties of the mappings we will focus on in this paper. Let K>1𝐾1K>1italic_K > 1 and θ(π/2,π/2]𝜃𝜋2𝜋2\theta\in(-\pi/2,\pi/2]italic_θ ∈ ( - italic_π / 2 , italic_π / 2 ]. Then we define hK,θsubscript𝐾𝜃h_{K,\theta}italic_h start_POSTSUBSCRIPT italic_K , italic_θ end_POSTSUBSCRIPT to be the stretch by factor K𝐾Kitalic_K in the direction of eiθsuperscript𝑒𝑖𝜃e^{i\theta}italic_e start_POSTSUPERSCRIPT italic_i italic_θ end_POSTSUPERSCRIPT. Evidently, if θ=0𝜃0\theta=0italic_θ = 0, then

hK,0(x+iy)=Kx+iy.subscript𝐾0𝑥𝑖𝑦𝐾𝑥𝑖𝑦h_{K,0}(x+iy)=Kx+iy.italic_h start_POSTSUBSCRIPT italic_K , 0 end_POSTSUBSCRIPT ( italic_x + italic_i italic_y ) = italic_K italic_x + italic_i italic_y .

Interpreting hK,θsubscript𝐾𝜃h_{K,\theta}italic_h start_POSTSUBSCRIPT italic_K , italic_θ end_POSTSUBSCRIPT in terms of hK,0subscript𝐾0h_{K,0}italic_h start_POSTSUBSCRIPT italic_K , 0 end_POSTSUBSCRIPT, if ρθsubscript𝜌𝜃\rho_{\theta}italic_ρ start_POSTSUBSCRIPT italic_θ end_POSTSUBSCRIPT denotes the rotation counter-clockwise by angle θ𝜃\thetaitalic_θ, then we see that

hK,θ=ρθhK,0ρθ.subscript𝐾𝜃subscript𝜌𝜃subscript𝐾0subscript𝜌𝜃h_{K,\theta}=\rho_{\theta}\circ h_{K,0}\circ\rho_{-\theta}.italic_h start_POSTSUBSCRIPT italic_K , italic_θ end_POSTSUBSCRIPT = italic_ρ start_POSTSUBSCRIPT italic_θ end_POSTSUBSCRIPT ∘ italic_h start_POSTSUBSCRIPT italic_K , 0 end_POSTSUBSCRIPT ∘ italic_ρ start_POSTSUBSCRIPT - italic_θ end_POSTSUBSCRIPT .

From this we can obtain the following explicit formulas for hK,θsubscript𝐾𝜃h_{K,\theta}italic_h start_POSTSUBSCRIPT italic_K , italic_θ end_POSTSUBSCRIPT:

hK,θ(z)subscript𝐾𝜃𝑧\displaystyle h_{K,\theta}(z)italic_h start_POSTSUBSCRIPT italic_K , italic_θ end_POSTSUBSCRIPT ( italic_z ) =(K+12)z+e2iθ(K12)z¯absent𝐾12𝑧superscript𝑒2𝑖𝜃𝐾12¯𝑧\displaystyle=\left(\frac{K+1}{2}\right)z+e^{2i\theta}\left(\frac{K-1}{2}% \right)\overline{z}= ( divide start_ARG italic_K + 1 end_ARG start_ARG 2 end_ARG ) italic_z + italic_e start_POSTSUPERSCRIPT 2 italic_i italic_θ end_POSTSUPERSCRIPT ( divide start_ARG italic_K - 1 end_ARG start_ARG 2 end_ARG ) over¯ start_ARG italic_z end_ARG
=[x(Kcos2(θ)+sin2(θ))+y(K1)sin(θ)cos(θ)]absentdelimited-[]𝑥𝐾superscript2𝜃superscript2𝜃𝑦𝐾1𝜃𝜃\displaystyle=\left[x(K\cos^{2}(\theta)+\sin^{2}(\theta))+y(K-1)\sin(\theta)% \cos(\theta)\right]= [ italic_x ( italic_K roman_cos start_POSTSUPERSCRIPT 2 end_POSTSUPERSCRIPT ( italic_θ ) + roman_sin start_POSTSUPERSCRIPT 2 end_POSTSUPERSCRIPT ( italic_θ ) ) + italic_y ( italic_K - 1 ) roman_sin ( italic_θ ) roman_cos ( italic_θ ) ]
+i[x(K1)cos(θ)sin(θ)+y(Ksin2(θ)+cos2(θ))].𝑖delimited-[]𝑥𝐾1𝜃𝜃𝑦𝐾superscript2𝜃superscript2𝜃\displaystyle\hskip 36.135pt+i\left[x(K-1)\cos(\theta)\sin(\theta)+y(K\sin^{2}% (\theta)+\cos^{2}(\theta))\right].+ italic_i [ italic_x ( italic_K - 1 ) roman_cos ( italic_θ ) roman_sin ( italic_θ ) + italic_y ( italic_K roman_sin start_POSTSUPERSCRIPT 2 end_POSTSUPERSCRIPT ( italic_θ ) + roman_cos start_POSTSUPERSCRIPT 2 end_POSTSUPERSCRIPT ( italic_θ ) ) ] .

It is clear that

μhK,θe2iθ(K1K+1),subscript𝜇subscript𝐾𝜃superscript𝑒2𝑖𝜃𝐾1𝐾1\mu_{h_{K,\theta}}\equiv e^{2i\theta}\left(\frac{K-1}{K+1}\right),italic_μ start_POSTSUBSCRIPT italic_h start_POSTSUBSCRIPT italic_K , italic_θ end_POSTSUBSCRIPT end_POSTSUBSCRIPT ≡ italic_e start_POSTSUPERSCRIPT 2 italic_i italic_θ end_POSTSUPERSCRIPT ( divide start_ARG italic_K - 1 end_ARG start_ARG italic_K + 1 end_ARG ) ,

and it follows from the Measurable Riemann Mapping Theorem and [11, Proposition 3.1] that any quasiconformal mapping of \mathbb{C}blackboard_C with constant complex dilatation given as above arises by conjugating hK,θsubscript𝐾𝜃h_{K,\theta}italic_h start_POSTSUBSCRIPT italic_K , italic_θ end_POSTSUBSCRIPT by a complex linear map.

If we view quasiconformal mappings with constant complex dilatation as the simplest in their class, then we can view quadratic polynomials as the simplest non-injective holomorphic functions. In light of the Stoilow Decomposition Theorem, the simplest quasiregular mappings arise as compositions of quadratic polynomials and mappings of the form hK,θsubscript𝐾𝜃h_{K,\theta}italic_h start_POSTSUBSCRIPT italic_K , italic_θ end_POSTSUBSCRIPT. As defined in the introduction, for K>1𝐾1K>1italic_K > 1, θ(π/2,π/2]𝜃𝜋2𝜋2\theta\in(-\pi/2,\pi/2]italic_θ ∈ ( - italic_π / 2 , italic_π / 2 ] and c𝑐c\in\mathbb{C}italic_c ∈ blackboard_C, we have HK,θ,c=PchK,θsubscript𝐻𝐾𝜃𝑐subscript𝑃𝑐subscript𝐾𝜃H_{K,\theta,c}=P_{c}\circ h_{K,\theta}italic_H start_POSTSUBSCRIPT italic_K , italic_θ , italic_c end_POSTSUBSCRIPT = italic_P start_POSTSUBSCRIPT italic_c end_POSTSUBSCRIPT ∘ italic_h start_POSTSUBSCRIPT italic_K , italic_θ end_POSTSUBSCRIPT. For later use, we note that if r>0𝑟0r>0italic_r > 0, then

(2.1) hK,θ(rz)=rhK,θ(z) and HK,θ,0(rz)=r2HK,θ,0(z).subscript𝐾𝜃𝑟𝑧𝑟subscript𝐾𝜃𝑧 and subscript𝐻𝐾𝜃0𝑟𝑧superscript𝑟2subscript𝐻𝐾𝜃0𝑧h_{K,\theta}(rz)=rh_{K,\theta}(z)\text{ and }H_{K,\theta,0}(rz)=r^{2}H_{K,% \theta,0}(z).italic_h start_POSTSUBSCRIPT italic_K , italic_θ end_POSTSUBSCRIPT ( italic_r italic_z ) = italic_r italic_h start_POSTSUBSCRIPT italic_K , italic_θ end_POSTSUBSCRIPT ( italic_z ) and italic_H start_POSTSUBSCRIPT italic_K , italic_θ , 0 end_POSTSUBSCRIPT ( italic_r italic_z ) = italic_r start_POSTSUPERSCRIPT 2 end_POSTSUPERSCRIPT italic_H start_POSTSUBSCRIPT italic_K , italic_θ , 0 end_POSTSUBSCRIPT ( italic_z ) .

It is worth remarking on the domains of K𝐾Kitalic_K and θ𝜃\thetaitalic_θ. Evidently a stretch in the direction eiθsuperscript𝑒𝑖𝜃e^{i\theta}italic_e start_POSTSUPERSCRIPT italic_i italic_θ end_POSTSUPERSCRIPT is the same as a stretch in the direction ei(θ+π)superscript𝑒𝑖𝜃𝜋e^{i(\theta+\pi)}italic_e start_POSTSUPERSCRIPT italic_i ( italic_θ + italic_π ) end_POSTSUPERSCRIPT, and so we only need consider θ(π/2,π/2]𝜃𝜋2𝜋2\theta\in(-\pi/2,\pi/2]italic_θ ∈ ( - italic_π / 2 , italic_π / 2 ]. A stretch by factor K>1𝐾1K>1italic_K > 1 in the direction eiθsuperscript𝑒𝑖𝜃e^{i\theta}italic_e start_POSTSUPERSCRIPT italic_i italic_θ end_POSTSUPERSCRIPT is conjugate to a contraction by factor 1/K1𝐾1/K1 / italic_K in the direction ei(θ+π/2)superscript𝑒𝑖𝜃𝜋2e^{i(\theta+\pi/2)}italic_e start_POSTSUPERSCRIPT italic_i ( italic_θ + italic_π / 2 ) end_POSTSUPERSCRIPT. We could thus restrict the domain of θ𝜃\thetaitalic_θ further and allow any positive value for K𝐾Kitalic_K, but we will usually use the convention that K>1𝐾1K>1italic_K > 1 instead. There may be occasion to conveniently allow K>0𝐾0K>0italic_K > 0, but we will alert the reader when this is the case. We recall from [11, Proposition 3.1] that every degree two quasiregular mapping of the plane with constant complex dilatation is linearly conjugate to HK,θ,csubscript𝐻𝐾𝜃𝑐H_{K,\theta,c}italic_H start_POSTSUBSCRIPT italic_K , italic_θ , italic_c end_POSTSUBSCRIPT for some choice of parameters.

2.3. Dynamics of HK,θ,csubscript𝐻𝐾𝜃𝑐H_{K,\theta,c}italic_H start_POSTSUBSCRIPT italic_K , italic_θ , italic_c end_POSTSUBSCRIPT

As HK,θ,csubscript𝐻𝐾𝜃𝑐H_{K,\theta,c}italic_H start_POSTSUBSCRIPT italic_K , italic_θ , italic_c end_POSTSUBSCRIPT is quasiregular, we may consider the behavior of its iterates. In the plane, every uniformly quasiregular mapping, that is, one for which there is a uniform bound on the maximal dilatation of the iterates, is known to be a quasiconformal conjugate of a holomorphic map. This means that the dynamics of such mappings yields essentially nothing new when compared to the features of complex dynamics. Importantly, from the point of view of independent interest, the mappings HK,θ,csubscript𝐻𝐾𝜃𝑐H_{K,\theta,c}italic_H start_POSTSUBSCRIPT italic_K , italic_θ , italic_c end_POSTSUBSCRIPT are not uniformly quasiregular [10, Theorem 1.12].

The escaping set for HK,θ,csubscript𝐻𝐾𝜃𝑐H_{K,\theta,c}italic_H start_POSTSUBSCRIPT italic_K , italic_θ , italic_c end_POSTSUBSCRIPT is a non-empty, open neighborhood of \infty by [11, Theorem 4.3]. It follows that the bounded orbit set is the complement of the escaping set. All of I(HK,θ,c),BO(HK,θ,c)𝐼subscript𝐻𝐾𝜃𝑐𝐵𝑂subscript𝐻𝐾𝜃𝑐I(H_{K,\theta,c}),BO(H_{K,\theta,c})italic_I ( italic_H start_POSTSUBSCRIPT italic_K , italic_θ , italic_c end_POSTSUBSCRIPT ) , italic_B italic_O ( italic_H start_POSTSUBSCRIPT italic_K , italic_θ , italic_c end_POSTSUBSCRIPT ) and I(HK,θ,c)𝐼subscript𝐻𝐾𝜃𝑐\partial I(H_{K,\theta,c})∂ italic_I ( italic_H start_POSTSUBSCRIPT italic_K , italic_θ , italic_c end_POSTSUBSCRIPT ) are completely invariant under HK,θ,csubscript𝐻𝐾𝜃𝑐H_{K,\theta,c}italic_H start_POSTSUBSCRIPT italic_K , italic_θ , italic_c end_POSTSUBSCRIPT.

When c=0𝑐0c=0italic_c = 0, we can guarantee a neighborhood of 00 is in BO(HK,θ,0)𝐵𝑂subscript𝐻𝐾𝜃0BO(H_{K,\theta,0})italic_B italic_O ( italic_H start_POSTSUBSCRIPT italic_K , italic_θ , 0 end_POSTSUBSCRIPT ).

Lemma 2.3.

Let K>1𝐾1K>1italic_K > 1 and θ(π/2,π/2]𝜃𝜋2𝜋2\theta\in(-\pi/2,\pi/2]italic_θ ∈ ( - italic_π / 2 , italic_π / 2 ]. Then the ball {z:|z|<(2K2)1}conditional-set𝑧𝑧superscript2superscript𝐾21\{z:|z|<(2K^{2})^{-1}\}{ italic_z : | italic_z | < ( 2 italic_K start_POSTSUPERSCRIPT 2 end_POSTSUPERSCRIPT ) start_POSTSUPERSCRIPT - 1 end_POSTSUPERSCRIPT } is contained in BO(HK,θ,0)𝐵𝑂subscript𝐻𝐾𝜃0BO(H_{K,\theta,0})italic_B italic_O ( italic_H start_POSTSUBSCRIPT italic_K , italic_θ , 0 end_POSTSUBSCRIPT ).

Proof.

If |z|<(2K2)1𝑧superscript2superscript𝐾21|z|<(2K^{2})^{-1}| italic_z | < ( 2 italic_K start_POSTSUPERSCRIPT 2 end_POSTSUPERSCRIPT ) start_POSTSUPERSCRIPT - 1 end_POSTSUPERSCRIPT, then we have

|HK,θ,0(z)|subscript𝐻𝐾𝜃0𝑧\displaystyle|H_{K,\theta,0}(z)|| italic_H start_POSTSUBSCRIPT italic_K , italic_θ , 0 end_POSTSUBSCRIPT ( italic_z ) | =|(K+12)z+e2iθ(K12)z¯|2absentsuperscript𝐾12𝑧superscript𝑒2𝑖𝜃𝐾12¯𝑧2\displaystyle=\left|\left(\frac{K+1}{2}\right)z+e^{2i\theta}\left(\frac{K-1}{2% }\right)\overline{z}\right|^{2}= | ( divide start_ARG italic_K + 1 end_ARG start_ARG 2 end_ARG ) italic_z + italic_e start_POSTSUPERSCRIPT 2 italic_i italic_θ end_POSTSUPERSCRIPT ( divide start_ARG italic_K - 1 end_ARG start_ARG 2 end_ARG ) over¯ start_ARG italic_z end_ARG | start_POSTSUPERSCRIPT 2 end_POSTSUPERSCRIPT
K2|z|2absentsuperscript𝐾2superscript𝑧2\displaystyle\leq K^{2}|z|^{2}≤ italic_K start_POSTSUPERSCRIPT 2 end_POSTSUPERSCRIPT | italic_z | start_POSTSUPERSCRIPT 2 end_POSTSUPERSCRIPT
<K2|z|2K2absentsuperscript𝐾2𝑧2superscript𝐾2\displaystyle<\frac{K^{2}|z|}{2K^{2}}< divide start_ARG italic_K start_POSTSUPERSCRIPT 2 end_POSTSUPERSCRIPT | italic_z | end_ARG start_ARG 2 italic_K start_POSTSUPERSCRIPT 2 end_POSTSUPERSCRIPT end_ARG
=|z|2.absent𝑧2\displaystyle=\frac{|z|}{2}.= divide start_ARG | italic_z | end_ARG start_ARG 2 end_ARG .

The conclusion follows. ∎

We also have control of the growth of HK,θ,csubscript𝐻𝐾𝜃𝑐H_{K,\theta,c}italic_H start_POSTSUBSCRIPT italic_K , italic_θ , italic_c end_POSTSUBSCRIPT near infinity.

Lemma 2.4.

Let K>1𝐾1K>1italic_K > 1, θ(π/2,π/2]𝜃𝜋2𝜋2\theta\in(-\pi/2,\pi/2]italic_θ ∈ ( - italic_π / 2 , italic_π / 2 ] and c𝑐c\in\mathbb{C}italic_c ∈ blackboard_C. If |z|max{|c|,2}𝑧𝑐2|z|\geq\max\{|c|,2\}| italic_z | ≥ roman_max { | italic_c | , 2 }, then

12|HK,θ,c(z)||z|2(K2+1).12subscript𝐻𝐾𝜃𝑐𝑧superscript𝑧2superscript𝐾21\frac{1}{2}\leq\frac{|H_{K,\theta,c}(z)|}{|z|^{2}}\leq(K^{2}+1).divide start_ARG 1 end_ARG start_ARG 2 end_ARG ≤ divide start_ARG | italic_H start_POSTSUBSCRIPT italic_K , italic_θ , italic_c end_POSTSUBSCRIPT ( italic_z ) | end_ARG start_ARG | italic_z | start_POSTSUPERSCRIPT 2 end_POSTSUPERSCRIPT end_ARG ≤ ( italic_K start_POSTSUPERSCRIPT 2 end_POSTSUPERSCRIPT + 1 ) .
Proof.

If |z|max{|c|,1}𝑧𝑐1|z|\geq\max\{|c|,1\}| italic_z | ≥ roman_max { | italic_c | , 1 }, then

|HK,θ,c(z)|subscript𝐻𝐾𝜃𝑐𝑧\displaystyle|H_{K,\theta,c}(z)|| italic_H start_POSTSUBSCRIPT italic_K , italic_θ , italic_c end_POSTSUBSCRIPT ( italic_z ) | =|hK,θ(z)2+c|absentsubscript𝐾𝜃superscript𝑧2𝑐\displaystyle=\left|h_{K,\theta}(z)^{2}+c\right|= | italic_h start_POSTSUBSCRIPT italic_K , italic_θ end_POSTSUBSCRIPT ( italic_z ) start_POSTSUPERSCRIPT 2 end_POSTSUPERSCRIPT + italic_c |
|hK,θ(z)|2+|c|absentsuperscriptsubscript𝐾𝜃𝑧2𝑐\displaystyle\leq|h_{K,\theta}(z)|^{2}+|c|≤ | italic_h start_POSTSUBSCRIPT italic_K , italic_θ end_POSTSUBSCRIPT ( italic_z ) | start_POSTSUPERSCRIPT 2 end_POSTSUPERSCRIPT + | italic_c |
K2|z|2+|z|absentsuperscript𝐾2superscript𝑧2𝑧\displaystyle\leq K^{2}|z|^{2}+|z|≤ italic_K start_POSTSUPERSCRIPT 2 end_POSTSUPERSCRIPT | italic_z | start_POSTSUPERSCRIPT 2 end_POSTSUPERSCRIPT + | italic_z |
(K2+1)|z|2,absentsuperscript𝐾21superscript𝑧2\displaystyle\leq(K^{2}+1)|z|^{2},≤ ( italic_K start_POSTSUPERSCRIPT 2 end_POSTSUPERSCRIPT + 1 ) | italic_z | start_POSTSUPERSCRIPT 2 end_POSTSUPERSCRIPT ,

which gives the upper bound. For the lower bound, if |z|max{|c|,2}𝑧𝑐2|z|\geq\max\{|c|,2\}| italic_z | ≥ roman_max { | italic_c | , 2 }, then

|HK,θ,c(z)|subscript𝐻𝐾𝜃𝑐𝑧\displaystyle|H_{K,\theta,c}(z)|| italic_H start_POSTSUBSCRIPT italic_K , italic_θ , italic_c end_POSTSUBSCRIPT ( italic_z ) | |hK,θ(z)|2|c|absentsuperscriptsubscript𝐾𝜃𝑧2𝑐\displaystyle\geq|h_{K,\theta}(z)|^{2}-|c|≥ | italic_h start_POSTSUBSCRIPT italic_K , italic_θ end_POSTSUBSCRIPT ( italic_z ) | start_POSTSUPERSCRIPT 2 end_POSTSUPERSCRIPT - | italic_c |
|z|2|z|absentsuperscript𝑧2𝑧\displaystyle\geq|z|^{2}-|z|≥ | italic_z | start_POSTSUPERSCRIPT 2 end_POSTSUPERSCRIPT - | italic_z |
=|z|2(1|z|1)absentsuperscript𝑧21superscript𝑧1\displaystyle=|z|^{2}(1-|z|^{-1})= | italic_z | start_POSTSUPERSCRIPT 2 end_POSTSUPERSCRIPT ( 1 - | italic_z | start_POSTSUPERSCRIPT - 1 end_POSTSUPERSCRIPT )
|z|22,absentsuperscript𝑧22\displaystyle\geq\frac{|z|^{2}}{2},≥ divide start_ARG | italic_z | start_POSTSUPERSCRIPT 2 end_POSTSUPERSCRIPT end_ARG start_ARG 2 end_ARG ,

as required. ∎

As this lemma also illustrates, the escaping set of HK,θ,csubscript𝐻𝐾𝜃𝑐H_{K,\theta,c}italic_H start_POSTSUBSCRIPT italic_K , italic_θ , italic_c end_POSTSUBSCRIPT is non-empty, and we can therefore ask for a conjugation to a simpler mapping in a neighborhood of infinity, analogous to Böttcher’s Theorem. The following result yields this.

Theorem 2.5 (Theorem 2.1, [9]).

Let K>1𝐾1K>1italic_K > 1, θ(π/2,π/2]𝜃𝜋2𝜋2\theta\in(-\pi/2,\pi/2]italic_θ ∈ ( - italic_π / 2 , italic_π / 2 ] and c𝑐c\in\mathbb{C}italic_c ∈ blackboard_C. Then there exist a neighborhood of infinity W=WK,θ,c𝑊subscript𝑊𝐾𝜃𝑐W=W_{K,\theta,c}italic_W = italic_W start_POSTSUBSCRIPT italic_K , italic_θ , italic_c end_POSTSUBSCRIPT and a quasiconformal map φ=φK,θ,c𝜑subscript𝜑𝐾𝜃𝑐\varphi=\varphi_{K,\theta,c}italic_φ = italic_φ start_POSTSUBSCRIPT italic_K , italic_θ , italic_c end_POSTSUBSCRIPT defined in W𝑊Witalic_W such that

φHK,θ,c=HK,θ,0φ𝜑subscript𝐻𝐾𝜃𝑐subscript𝐻𝐾𝜃0𝜑\varphi\circ H_{K,\theta,c}=H_{K,\theta,0}\circ\varphiitalic_φ ∘ italic_H start_POSTSUBSCRIPT italic_K , italic_θ , italic_c end_POSTSUBSCRIPT = italic_H start_POSTSUBSCRIPT italic_K , italic_θ , 0 end_POSTSUBSCRIPT ∘ italic_φ

holds in W𝑊Witalic_W.

2.4. Riemann-Hurwitz formula

Finally, we will need the following version of the Riemann-Hurwitz formula. Due to the Stoilow Decomposition Theorem, there are no technical issues when applying the Riemann-Hurwitz formula with quasiregular mappings instead of holomorphic functions, but we refer to [11, Corollary 5.2] for more details.

Theorem 2.6.

Let D1subscript𝐷1D_{1}italic_D start_POSTSUBSCRIPT 1 end_POSTSUBSCRIPT and D2subscript𝐷2D_{2}italic_D start_POSTSUBSCRIPT 2 end_POSTSUBSCRIPT be domains in subscript\mathbb{C}_{\infty}blackboard_C start_POSTSUBSCRIPT ∞ end_POSTSUBSCRIPT whose boundaries consist of a finite number of simple closed curves. Let f𝑓fitalic_f be a proper quasiregular map of degree d𝑑ditalic_d from D1subscript𝐷1D_{1}italic_D start_POSTSUBSCRIPT 1 end_POSTSUBSCRIPT onto D2subscript𝐷2D_{2}italic_D start_POSTSUBSCRIPT 2 end_POSTSUBSCRIPT with L𝐿Litalic_L branch points including multiplicity. Then every zD2𝑧subscript𝐷2z\in D_{2}italic_z ∈ italic_D start_POSTSUBSCRIPT 2 end_POSTSUBSCRIPT has the same number d𝑑ditalic_d of pre-images including multiplicity and

2d1=d(2d2)L,2subscript𝑑1𝑑2subscript𝑑2𝐿2-d_{1}=d(2-d_{2})-L,2 - italic_d start_POSTSUBSCRIPT 1 end_POSTSUBSCRIPT = italic_d ( 2 - italic_d start_POSTSUBSCRIPT 2 end_POSTSUBSCRIPT ) - italic_L ,

where djsubscript𝑑𝑗d_{j}italic_d start_POSTSUBSCRIPT italic_j end_POSTSUBSCRIPT is the number of boundary components of Djsubscript𝐷𝑗D_{j}italic_D start_POSTSUBSCRIPT italic_j end_POSTSUBSCRIPT.

3. Constructing the Green’s function

In this section, we will construct our analogue of Green’s function and prove Theorem 1.1. First, let us denote by Rϕsubscript𝑅italic-ϕR_{\phi}italic_R start_POSTSUBSCRIPT italic_ϕ end_POSTSUBSCRIPT the ray {teiϕ:t0}conditional-set𝑡superscript𝑒𝑖italic-ϕ𝑡0\{te^{i\phi}:t\geq 0\}{ italic_t italic_e start_POSTSUPERSCRIPT italic_i italic_ϕ end_POSTSUPERSCRIPT : italic_t ≥ 0 }. The bounded orbit set BO(HK,θ,0)𝐵𝑂subscript𝐻𝐾𝜃0BO(H_{K,\theta,0})italic_B italic_O ( italic_H start_POSTSUBSCRIPT italic_K , italic_θ , 0 end_POSTSUBSCRIPT ) is starlike about z=0𝑧0z=0italic_z = 0, as the next lemma shows. We remark that this result has appeared as [10, Corollary 1.10], although there is some opacity to that proof which the following proof makes transparent.

Lemma 3.1.

Let K>1𝐾1K>1italic_K > 1 and θ(π/2,π/2]𝜃𝜋2𝜋2\theta\in(-\pi/2,\pi/2]italic_θ ∈ ( - italic_π / 2 , italic_π / 2 ]. For any ϕ[0,2π)italic-ϕ02𝜋\phi\in[0,2\pi)italic_ϕ ∈ [ 0 , 2 italic_π ), the set I(HK,θ,0)Rϕ𝐼subscript𝐻𝐾𝜃0subscript𝑅italic-ϕ\partial I(H_{K,\theta,0})\cap R_{\phi}∂ italic_I ( italic_H start_POSTSUBSCRIPT italic_K , italic_θ , 0 end_POSTSUBSCRIPT ) ∩ italic_R start_POSTSUBSCRIPT italic_ϕ end_POSTSUBSCRIPT contains exactly one element.

Proof.

The set I(HK,θ,0)𝐼subscript𝐻𝐾𝜃0I(H_{K,\theta,0})italic_I ( italic_H start_POSTSUBSCRIPT italic_K , italic_θ , 0 end_POSTSUBSCRIPT ) contains a neighborhood of infinity, so I(HK,θ,0)Rϕ𝐼subscript𝐻𝐾𝜃0subscript𝑅italic-ϕI(H_{K,\theta,0})\cap R_{\phi}italic_I ( italic_H start_POSTSUBSCRIPT italic_K , italic_θ , 0 end_POSTSUBSCRIPT ) ∩ italic_R start_POSTSUBSCRIPT italic_ϕ end_POSTSUBSCRIPT is non-empty. By Lemma 2.3, there exists r>0𝑟0r>0italic_r > 0 such that {z:|z|<r}conditional-set𝑧𝑧𝑟\{z:|z|<r\}{ italic_z : | italic_z | < italic_r } is in the interior of BO(HK,θ,0)𝐵𝑂subscript𝐻𝐾𝜃0BO(H_{K,\theta,0})italic_B italic_O ( italic_H start_POSTSUBSCRIPT italic_K , italic_θ , 0 end_POSTSUBSCRIPT ). Thus, BO(HK,θ,0)Rϕ𝐵𝑂subscript𝐻𝐾𝜃0subscript𝑅italic-ϕBO(H_{K,\theta,0})\cap R_{\phi}italic_B italic_O ( italic_H start_POSTSUBSCRIPT italic_K , italic_θ , 0 end_POSTSUBSCRIPT ) ∩ italic_R start_POSTSUBSCRIPT italic_ϕ end_POSTSUBSCRIPT is non-empty. As such, HK,θ,0Rϕsubscript𝐻𝐾𝜃0subscript𝑅italic-ϕ\partial H_{K,\theta,0}\cap R_{\phi}∂ italic_H start_POSTSUBSCRIPT italic_K , italic_θ , 0 end_POSTSUBSCRIPT ∩ italic_R start_POSTSUBSCRIPT italic_ϕ end_POSTSUBSCRIPT contains at least one element. Moreover, I(HK,θ,0)𝐼subscript𝐻𝐾𝜃0\partial I(H_{K,\theta,0})∂ italic_I ( italic_H start_POSTSUBSCRIPT italic_K , italic_θ , 0 end_POSTSUBSCRIPT ) contains no elements that are less than r𝑟ritalic_r in absolute value.

Suppose towards a contradiction that I(HK,θ,0)Rϕ𝐼subscript𝐻𝐾𝜃0subscript𝑅italic-ϕ\partial I(H_{K,\theta,0})\cap R_{\phi}∂ italic_I ( italic_H start_POSTSUBSCRIPT italic_K , italic_θ , 0 end_POSTSUBSCRIPT ) ∩ italic_R start_POSTSUBSCRIPT italic_ϕ end_POSTSUBSCRIPT contains at least two elements. Then there are elements z1subscript𝑧1z_{1}italic_z start_POSTSUBSCRIPT 1 end_POSTSUBSCRIPT and z2subscript𝑧2z_{2}italic_z start_POSTSUBSCRIPT 2 end_POSTSUBSCRIPT in I(HK,θ,0)Rϕ𝐼subscript𝐻𝐾𝜃0subscript𝑅italic-ϕ\partial I(H_{K,\theta,0})\cap R_{\phi}∂ italic_I ( italic_H start_POSTSUBSCRIPT italic_K , italic_θ , 0 end_POSTSUBSCRIPT ) ∩ italic_R start_POSTSUBSCRIPT italic_ϕ end_POSTSUBSCRIPT such that z1=tz2subscript𝑧1𝑡subscript𝑧2z_{1}=tz_{2}italic_z start_POSTSUBSCRIPT 1 end_POSTSUBSCRIPT = italic_t italic_z start_POSTSUBSCRIPT 2 end_POSTSUBSCRIPT, where t>1𝑡1t>1italic_t > 1. By (2.1), Lemma 2.3, and the complete invariance of I(HK,θ,c)𝐼subscript𝐻𝐾𝜃𝑐\partial I(H_{K,\theta,c})∂ italic_I ( italic_H start_POSTSUBSCRIPT italic_K , italic_θ , italic_c end_POSTSUBSCRIPT ), for n𝑛n\in\mathbb{N}italic_n ∈ blackboard_N we have

|HK,θ,0n(z1)|subscriptsuperscript𝐻𝑛𝐾𝜃0subscript𝑧1\displaystyle|H^{n}_{K,\theta,0}(z_{1})|| italic_H start_POSTSUPERSCRIPT italic_n end_POSTSUPERSCRIPT start_POSTSUBSCRIPT italic_K , italic_θ , 0 end_POSTSUBSCRIPT ( italic_z start_POSTSUBSCRIPT 1 end_POSTSUBSCRIPT ) | =|HK,θ,0n(tz2)|absentsubscriptsuperscript𝐻𝑛𝐾𝜃0𝑡subscript𝑧2\displaystyle=|H^{n}_{K,\theta,0}(tz_{2})|= | italic_H start_POSTSUPERSCRIPT italic_n end_POSTSUPERSCRIPT start_POSTSUBSCRIPT italic_K , italic_θ , 0 end_POSTSUBSCRIPT ( italic_t italic_z start_POSTSUBSCRIPT 2 end_POSTSUBSCRIPT ) |
=|t2nHK,θ,0n(z2)|absentsuperscript𝑡superscript2𝑛subscriptsuperscript𝐻𝑛𝐾𝜃0subscript𝑧2\displaystyle=|t^{2^{n}}H^{n}_{K,\theta,0}(z_{2})|= | italic_t start_POSTSUPERSCRIPT 2 start_POSTSUPERSCRIPT italic_n end_POSTSUPERSCRIPT end_POSTSUPERSCRIPT italic_H start_POSTSUPERSCRIPT italic_n end_POSTSUPERSCRIPT start_POSTSUBSCRIPT italic_K , italic_θ , 0 end_POSTSUBSCRIPT ( italic_z start_POSTSUBSCRIPT 2 end_POSTSUBSCRIPT ) |
=t2n|HK,θ,0n(z2)|absentsuperscript𝑡superscript2𝑛subscriptsuperscript𝐻𝑛𝐾𝜃0subscript𝑧2\displaystyle=t^{2^{n}}|H^{n}_{K,\theta,0}(z_{2})|= italic_t start_POSTSUPERSCRIPT 2 start_POSTSUPERSCRIPT italic_n end_POSTSUPERSCRIPT end_POSTSUPERSCRIPT | italic_H start_POSTSUPERSCRIPT italic_n end_POSTSUPERSCRIPT start_POSTSUBSCRIPT italic_K , italic_θ , 0 end_POSTSUBSCRIPT ( italic_z start_POSTSUBSCRIPT 2 end_POSTSUBSCRIPT ) |
t2n2K2.absentsuperscript𝑡superscript2𝑛2superscript𝐾2\displaystyle\geq\frac{t^{2^{n}}}{2K^{2}}.≥ divide start_ARG italic_t start_POSTSUPERSCRIPT 2 start_POSTSUPERSCRIPT italic_n end_POSTSUPERSCRIPT end_POSTSUPERSCRIPT end_ARG start_ARG 2 italic_K start_POSTSUPERSCRIPT 2 end_POSTSUPERSCRIPT end_ARG .

Therefore, z1I(HK,θ,0)subscript𝑧1𝐼subscript𝐻𝐾𝜃0z_{1}\in I(H_{K,\theta,0})italic_z start_POSTSUBSCRIPT 1 end_POSTSUBSCRIPT ∈ italic_I ( italic_H start_POSTSUBSCRIPT italic_K , italic_θ , 0 end_POSTSUBSCRIPT ), which is a contradiction. ∎

In light of Lemma 3.1, we make the following definition.

Definition 3.2.

Let K>1𝐾1K>1italic_K > 1 and θ(π/2,π/2]𝜃𝜋2𝜋2\theta\in(-\pi/2,\pi/2]italic_θ ∈ ( - italic_π / 2 , italic_π / 2 ]. Denote by bK,θ(ϕ)subscript𝑏𝐾𝜃italic-ϕb_{K,\theta}(\phi)\in\mathbb{C}italic_b start_POSTSUBSCRIPT italic_K , italic_θ end_POSTSUBSCRIPT ( italic_ϕ ) ∈ blackboard_C the unique element of I(HK,θ,0)Rϕ𝐼subscript𝐻𝐾𝜃0subscript𝑅italic-ϕ\partial I(H_{K,\theta,0})\cap R_{\phi}∂ italic_I ( italic_H start_POSTSUBSCRIPT italic_K , italic_θ , 0 end_POSTSUBSCRIPT ) ∩ italic_R start_POSTSUBSCRIPT italic_ϕ end_POSTSUBSCRIPT.

Our aim is to use bK,θsubscript𝑏𝐾𝜃b_{K,\theta}italic_b start_POSTSUBSCRIPT italic_K , italic_θ end_POSTSUBSCRIPT to model the dynamics of HK,θ,0subscript𝐻𝐾𝜃0H_{K,\theta,0}italic_H start_POSTSUBSCRIPT italic_K , italic_θ , 0 end_POSTSUBSCRIPT on a squaring map.

Definition 3.3.

Let K>1𝐾1K>1italic_K > 1 and θ(π/2,π/2]𝜃𝜋2𝜋2\theta\in(-\pi/2,\pi/2]italic_θ ∈ ( - italic_π / 2 , italic_π / 2 ]. Define τK,θ,0:+:subscript𝜏𝐾𝜃0superscript\tau_{K,\theta,0}:\mathbb{C}\to\mathbb{R}^{+}italic_τ start_POSTSUBSCRIPT italic_K , italic_θ , 0 end_POSTSUBSCRIPT : blackboard_C → blackboard_R start_POSTSUPERSCRIPT + end_POSTSUPERSCRIPT for z0𝑧0z\neq 0italic_z ≠ 0 by τK,θ,0(z)=zbK,θ(arg(z))subscript𝜏𝐾𝜃0𝑧𝑧subscript𝑏𝐾𝜃𝑧\tau_{K,\theta,0}(z)=\frac{z}{b_{K,\theta}(\arg(z))}italic_τ start_POSTSUBSCRIPT italic_K , italic_θ , 0 end_POSTSUBSCRIPT ( italic_z ) = divide start_ARG italic_z end_ARG start_ARG italic_b start_POSTSUBSCRIPT italic_K , italic_θ end_POSTSUBSCRIPT ( roman_arg ( italic_z ) ) end_ARG, and set τK,θ,0(0)=0subscript𝜏𝐾𝜃000\tau_{K,\theta,0}(0)=0italic_τ start_POSTSUBSCRIPT italic_K , italic_θ , 0 end_POSTSUBSCRIPT ( 0 ) = 0.

Observe that arg(bK,θ(arg(z)))=arg(z)subscript𝑏𝐾𝜃𝑧𝑧\arg(b_{K,\theta}(\arg(z)))=\arg(z)roman_arg ( italic_b start_POSTSUBSCRIPT italic_K , italic_θ end_POSTSUBSCRIPT ( roman_arg ( italic_z ) ) ) = roman_arg ( italic_z ). The key point here is that τK,θ,01(1)=I(HK,θ,0)superscriptsubscript𝜏𝐾𝜃011𝐼subscript𝐻𝐾𝜃0\tau_{K,\theta,0}^{-1}(1)=\partial I(H_{K,\theta,0})italic_τ start_POSTSUBSCRIPT italic_K , italic_θ , 0 end_POSTSUBSCRIPT start_POSTSUPERSCRIPT - 1 end_POSTSUPERSCRIPT ( 1 ) = ∂ italic_I ( italic_H start_POSTSUBSCRIPT italic_K , italic_θ , 0 end_POSTSUBSCRIPT ).

Lemma 3.4.

For any r>0𝑟0r>0italic_r > 0, τK,θ,0(rz)=rτK,θ,0(z)subscript𝜏𝐾𝜃0𝑟𝑧𝑟subscript𝜏𝐾𝜃0𝑧\tau_{K,\theta,0}(rz)=r\tau_{K,\theta,0}(z)italic_τ start_POSTSUBSCRIPT italic_K , italic_θ , 0 end_POSTSUBSCRIPT ( italic_r italic_z ) = italic_r italic_τ start_POSTSUBSCRIPT italic_K , italic_θ , 0 end_POSTSUBSCRIPT ( italic_z ).

Proof.

The claim is clear if r=0𝑟0r=0italic_r = 0. Otherwise,

τK,θ,0(rz)subscript𝜏𝐾𝜃0𝑟𝑧\displaystyle\tau_{K,\theta,0}(rz)italic_τ start_POSTSUBSCRIPT italic_K , italic_θ , 0 end_POSTSUBSCRIPT ( italic_r italic_z ) =rzbK,θ(arg(z))absent𝑟𝑧subscript𝑏𝐾𝜃𝑧\displaystyle=\frac{rz}{b_{K,\theta}(\arg(z))}= divide start_ARG italic_r italic_z end_ARG start_ARG italic_b start_POSTSUBSCRIPT italic_K , italic_θ end_POSTSUBSCRIPT ( roman_arg ( italic_z ) ) end_ARG
=rτK,θ,0(z),absent𝑟subscript𝜏𝐾𝜃0𝑧\displaystyle=r\tau_{K,\theta,0}(z),= italic_r italic_τ start_POSTSUBSCRIPT italic_K , italic_θ , 0 end_POSTSUBSCRIPT ( italic_z ) ,

as required. ∎

The following lemma is the whole point of introducing τK,θ,0subscript𝜏𝐾𝜃0\tau_{K,\theta,0}italic_τ start_POSTSUBSCRIPT italic_K , italic_θ , 0 end_POSTSUBSCRIPT: it semi-conjugates between HK,θ,0subscript𝐻𝐾𝜃0H_{K,\theta,0}italic_H start_POSTSUBSCRIPT italic_K , italic_θ , 0 end_POSTSUBSCRIPT in \mathbb{C}blackboard_C and the squaring map on +superscript\mathbb{R}^{+}blackboard_R start_POSTSUPERSCRIPT + end_POSTSUPERSCRIPT.

Lemma 3.5.

Let K>1𝐾1K>1italic_K > 1 and θ(π/2,π/2]𝜃𝜋2𝜋2\theta\in(-\pi/2,\pi/2]italic_θ ∈ ( - italic_π / 2 , italic_π / 2 ]. For any z𝑧z\in\mathbb{C}italic_z ∈ blackboard_C,

τK,θ,0(HK,θ,0(z))=[τK,θ,0(z)]2.subscript𝜏𝐾𝜃0subscript𝐻𝐾𝜃0𝑧superscriptdelimited-[]subscript𝜏𝐾𝜃0𝑧2\tau_{K,\theta,0}(H_{K,\theta,0}(z))=[\tau_{K,\theta,0}(z)]^{2}.italic_τ start_POSTSUBSCRIPT italic_K , italic_θ , 0 end_POSTSUBSCRIPT ( italic_H start_POSTSUBSCRIPT italic_K , italic_θ , 0 end_POSTSUBSCRIPT ( italic_z ) ) = [ italic_τ start_POSTSUBSCRIPT italic_K , italic_θ , 0 end_POSTSUBSCRIPT ( italic_z ) ] start_POSTSUPERSCRIPT 2 end_POSTSUPERSCRIPT .
Proof.

If z=0𝑧0z=0italic_z = 0, then the lemma is clear. Otherwise, suppose that z=reiϕ𝑧𝑟superscript𝑒𝑖italic-ϕz=re^{i\phi}italic_z = italic_r italic_e start_POSTSUPERSCRIPT italic_i italic_ϕ end_POSTSUPERSCRIPT. Then by (2.1) and Lemma 3.4, we have

τK,θ,0(HK,θ,0(reiϕ))subscript𝜏𝐾𝜃0subscript𝐻𝐾𝜃0𝑟superscript𝑒𝑖italic-ϕ\displaystyle\tau_{K,\theta,0}(H_{K,\theta,0}(re^{i\phi}))italic_τ start_POSTSUBSCRIPT italic_K , italic_θ , 0 end_POSTSUBSCRIPT ( italic_H start_POSTSUBSCRIPT italic_K , italic_θ , 0 end_POSTSUBSCRIPT ( italic_r italic_e start_POSTSUPERSCRIPT italic_i italic_ϕ end_POSTSUPERSCRIPT ) ) =τK,θ,0(HK,θ,0(r|bK,θ(ϕ)|eiϕ|bK,θ(ϕ)|))absentsubscript𝜏𝐾𝜃0subscript𝐻𝐾𝜃0𝑟subscript𝑏𝐾𝜃italic-ϕsuperscript𝑒𝑖italic-ϕsubscript𝑏𝐾𝜃italic-ϕ\displaystyle=\tau_{K,\theta,0}\left(H_{K,\theta,0}\left(\frac{r|b_{K,\theta}(% \phi)|e^{i\phi}}{|b_{K,\theta}(\phi)|}\right)\right)= italic_τ start_POSTSUBSCRIPT italic_K , italic_θ , 0 end_POSTSUBSCRIPT ( italic_H start_POSTSUBSCRIPT italic_K , italic_θ , 0 end_POSTSUBSCRIPT ( divide start_ARG italic_r | italic_b start_POSTSUBSCRIPT italic_K , italic_θ end_POSTSUBSCRIPT ( italic_ϕ ) | italic_e start_POSTSUPERSCRIPT italic_i italic_ϕ end_POSTSUPERSCRIPT end_ARG start_ARG | italic_b start_POSTSUBSCRIPT italic_K , italic_θ end_POSTSUBSCRIPT ( italic_ϕ ) | end_ARG ) )
=r2|bK,θ(ϕ)|2τK,θ,0(HK,θ,0(|bK,θ(ϕ)|eiϕ))absentsuperscript𝑟2superscriptsubscript𝑏𝐾𝜃italic-ϕ2subscript𝜏𝐾𝜃0subscript𝐻𝐾𝜃0subscript𝑏𝐾𝜃italic-ϕsuperscript𝑒𝑖italic-ϕ\displaystyle=\frac{r^{2}}{|b_{K,\theta}(\phi)|^{2}}\tau_{K,\theta,0}(H_{K,% \theta,0}(|b_{K,\theta}(\phi)|e^{i\phi}))= divide start_ARG italic_r start_POSTSUPERSCRIPT 2 end_POSTSUPERSCRIPT end_ARG start_ARG | italic_b start_POSTSUBSCRIPT italic_K , italic_θ end_POSTSUBSCRIPT ( italic_ϕ ) | start_POSTSUPERSCRIPT 2 end_POSTSUPERSCRIPT end_ARG italic_τ start_POSTSUBSCRIPT italic_K , italic_θ , 0 end_POSTSUBSCRIPT ( italic_H start_POSTSUBSCRIPT italic_K , italic_θ , 0 end_POSTSUBSCRIPT ( | italic_b start_POSTSUBSCRIPT italic_K , italic_θ end_POSTSUBSCRIPT ( italic_ϕ ) | italic_e start_POSTSUPERSCRIPT italic_i italic_ϕ end_POSTSUPERSCRIPT ) )
=r2|bK,θ(ϕ)|2τK,θ,0(HK,θ,0(bK,θ(ϕ)))absentsuperscript𝑟2superscriptsubscript𝑏𝐾𝜃italic-ϕ2subscript𝜏𝐾𝜃0subscript𝐻𝐾𝜃0subscript𝑏𝐾𝜃italic-ϕ\displaystyle=\frac{r^{2}}{|b_{K,\theta}(\phi)|^{2}}\tau_{K,\theta,0}(H_{K,% \theta,0}(b_{K,\theta}(\phi)))= divide start_ARG italic_r start_POSTSUPERSCRIPT 2 end_POSTSUPERSCRIPT end_ARG start_ARG | italic_b start_POSTSUBSCRIPT italic_K , italic_θ end_POSTSUBSCRIPT ( italic_ϕ ) | start_POSTSUPERSCRIPT 2 end_POSTSUPERSCRIPT end_ARG italic_τ start_POSTSUBSCRIPT italic_K , italic_θ , 0 end_POSTSUBSCRIPT ( italic_H start_POSTSUBSCRIPT italic_K , italic_θ , 0 end_POSTSUBSCRIPT ( italic_b start_POSTSUBSCRIPT italic_K , italic_θ end_POSTSUBSCRIPT ( italic_ϕ ) ) )

As bK,θ(ϕ)I(HK,θ,0)subscript𝑏𝐾𝜃italic-ϕ𝐼subscript𝐻𝐾𝜃0b_{K,\theta}(\phi)\in\partial I(H_{K,\theta,0})italic_b start_POSTSUBSCRIPT italic_K , italic_θ end_POSTSUBSCRIPT ( italic_ϕ ) ∈ ∂ italic_I ( italic_H start_POSTSUBSCRIPT italic_K , italic_θ , 0 end_POSTSUBSCRIPT ) and as I(HK,θ,0)𝐼subscript𝐻𝐾𝜃0\partial I(H_{K,\theta,0})∂ italic_I ( italic_H start_POSTSUBSCRIPT italic_K , italic_θ , 0 end_POSTSUBSCRIPT ) is completely invariant, we see that τK,θ,0(HK,θ,0(bK,θ(ϕ)))=τK,θ,0(bK,θ(ϕ))=1subscript𝜏𝐾𝜃0subscript𝐻𝐾𝜃0subscript𝑏𝐾𝜃italic-ϕsubscript𝜏𝐾𝜃0subscript𝑏𝐾𝜃italic-ϕ1\tau_{K,\theta,0}(H_{K,\theta,0}(b_{K,\theta}(\phi)))=\tau_{K,\theta,0}(b_{K,% \theta}(\phi))=1italic_τ start_POSTSUBSCRIPT italic_K , italic_θ , 0 end_POSTSUBSCRIPT ( italic_H start_POSTSUBSCRIPT italic_K , italic_θ , 0 end_POSTSUBSCRIPT ( italic_b start_POSTSUBSCRIPT italic_K , italic_θ end_POSTSUBSCRIPT ( italic_ϕ ) ) ) = italic_τ start_POSTSUBSCRIPT italic_K , italic_θ , 0 end_POSTSUBSCRIPT ( italic_b start_POSTSUBSCRIPT italic_K , italic_θ end_POSTSUBSCRIPT ( italic_ϕ ) ) = 1. Therefore,

τK,θ,0(HK,θ,0(reiϕ))subscript𝜏𝐾𝜃0subscript𝐻𝐾𝜃0𝑟superscript𝑒𝑖italic-ϕ\displaystyle\tau_{K,\theta,0}(H_{K,\theta,0}(re^{i\phi}))italic_τ start_POSTSUBSCRIPT italic_K , italic_θ , 0 end_POSTSUBSCRIPT ( italic_H start_POSTSUBSCRIPT italic_K , italic_θ , 0 end_POSTSUBSCRIPT ( italic_r italic_e start_POSTSUPERSCRIPT italic_i italic_ϕ end_POSTSUPERSCRIPT ) ) =r2|bK,θ(ϕ)|2absentsuperscript𝑟2superscriptsubscript𝑏𝐾𝜃italic-ϕ2\displaystyle=\frac{r^{2}}{|b_{K,\theta}(\phi)|^{2}}= divide start_ARG italic_r start_POSTSUPERSCRIPT 2 end_POSTSUPERSCRIPT end_ARG start_ARG | italic_b start_POSTSUBSCRIPT italic_K , italic_θ end_POSTSUBSCRIPT ( italic_ϕ ) | start_POSTSUPERSCRIPT 2 end_POSTSUPERSCRIPT end_ARG
=(r|bK,θ(ϕ)|τK,θ,0(bK,θ(ϕ)))2absentsuperscript𝑟subscript𝑏𝐾𝜃italic-ϕsubscript𝜏𝐾𝜃0subscript𝑏𝐾𝜃italic-ϕ2\displaystyle=\left(\frac{r}{|b_{K,\theta}(\phi)|}\tau_{K,\theta,0}(b_{K,% \theta}(\phi))\right)^{2}= ( divide start_ARG italic_r end_ARG start_ARG | italic_b start_POSTSUBSCRIPT italic_K , italic_θ end_POSTSUBSCRIPT ( italic_ϕ ) | end_ARG italic_τ start_POSTSUBSCRIPT italic_K , italic_θ , 0 end_POSTSUBSCRIPT ( italic_b start_POSTSUBSCRIPT italic_K , italic_θ end_POSTSUBSCRIPT ( italic_ϕ ) ) ) start_POSTSUPERSCRIPT 2 end_POSTSUPERSCRIPT
=[τK,θ,0(r|bK,θ(ϕ)|bK,θ(ϕ))]2absentsuperscriptdelimited-[]subscript𝜏𝐾𝜃0𝑟subscript𝑏𝐾𝜃italic-ϕsubscript𝑏𝐾𝜃italic-ϕ2\displaystyle=\left[\tau_{K,\theta,0}\left(\frac{r}{|b_{K,\theta}(\phi)|}b_{K,% \theta}(\phi)\right)\right]^{2}= [ italic_τ start_POSTSUBSCRIPT italic_K , italic_θ , 0 end_POSTSUBSCRIPT ( divide start_ARG italic_r end_ARG start_ARG | italic_b start_POSTSUBSCRIPT italic_K , italic_θ end_POSTSUBSCRIPT ( italic_ϕ ) | end_ARG italic_b start_POSTSUBSCRIPT italic_K , italic_θ end_POSTSUBSCRIPT ( italic_ϕ ) ) ] start_POSTSUPERSCRIPT 2 end_POSTSUPERSCRIPT
=[τK,θ,0(reiθ)]2.absentsuperscriptdelimited-[]subscript𝜏𝐾𝜃0𝑟superscript𝑒𝑖𝜃2\displaystyle=[\tau_{K,\theta,0}(re^{i\theta})]^{2}.= [ italic_τ start_POSTSUBSCRIPT italic_K , italic_θ , 0 end_POSTSUBSCRIPT ( italic_r italic_e start_POSTSUPERSCRIPT italic_i italic_θ end_POSTSUPERSCRIPT ) ] start_POSTSUPERSCRIPT 2 end_POSTSUPERSCRIPT .

We next extend the definition of τK,θ,0subscript𝜏𝐾𝜃0\tau_{K,\theta,0}italic_τ start_POSTSUBSCRIPT italic_K , italic_θ , 0 end_POSTSUBSCRIPT to τK,θ,csubscript𝜏𝐾𝜃𝑐\tau_{K,\theta,c}italic_τ start_POSTSUBSCRIPT italic_K , italic_θ , italic_c end_POSTSUBSCRIPT for general c𝑐c\in\mathbb{C}italic_c ∈ blackboard_C by using the Böttcher coordinate, with the caveat that τK,θ,csubscript𝜏𝐾𝜃𝑐\tau_{K,\theta,c}italic_τ start_POSTSUBSCRIPT italic_K , italic_θ , italic_c end_POSTSUBSCRIPT is not (immediately) globally defined. Recall the quasiconformal map φK,θ,csubscript𝜑𝐾𝜃𝑐\varphi_{K,\theta,c}italic_φ start_POSTSUBSCRIPT italic_K , italic_θ , italic_c end_POSTSUBSCRIPT defined in the neighborhood WK,θ,csubscript𝑊𝐾𝜃𝑐W_{K,\theta,c}italic_W start_POSTSUBSCRIPT italic_K , italic_θ , italic_c end_POSTSUBSCRIPT of \infty from Theroem 2.5.

Definition 3.6.

Let K>1𝐾1K>1italic_K > 1, θ(π/2,π/2]𝜃𝜋2𝜋2\theta\in(-\pi/2,\pi/2]italic_θ ∈ ( - italic_π / 2 , italic_π / 2 ] and c𝑐c\in\mathbb{C}italic_c ∈ blackboard_C. For zWK,θ,c𝑧subscript𝑊𝐾𝜃𝑐z\in W_{K,\theta,c}italic_z ∈ italic_W start_POSTSUBSCRIPT italic_K , italic_θ , italic_c end_POSTSUBSCRIPT, we define τK,θ,c:WK,θ,c+:subscript𝜏𝐾𝜃𝑐subscript𝑊𝐾𝜃𝑐superscript\tau_{K,\theta,c}:W_{K,\theta,c}\to\mathbb{R}^{+}italic_τ start_POSTSUBSCRIPT italic_K , italic_θ , italic_c end_POSTSUBSCRIPT : italic_W start_POSTSUBSCRIPT italic_K , italic_θ , italic_c end_POSTSUBSCRIPT → blackboard_R start_POSTSUPERSCRIPT + end_POSTSUPERSCRIPT by

τK,θ,c(z)=τK,θ,0(φK,θ,c(z)).subscript𝜏𝐾𝜃𝑐𝑧subscript𝜏𝐾𝜃0subscript𝜑𝐾𝜃𝑐𝑧\tau_{K,\theta,c}(z)=\tau_{K,\theta,0}(\varphi_{K,\theta,c}(z)).italic_τ start_POSTSUBSCRIPT italic_K , italic_θ , italic_c end_POSTSUBSCRIPT ( italic_z ) = italic_τ start_POSTSUBSCRIPT italic_K , italic_θ , 0 end_POSTSUBSCRIPT ( italic_φ start_POSTSUBSCRIPT italic_K , italic_θ , italic_c end_POSTSUBSCRIPT ( italic_z ) ) .

Importantly, this generalized version of τK,θ,0subscript𝜏𝐾𝜃0\tau_{K,\theta,0}italic_τ start_POSTSUBSCRIPT italic_K , italic_θ , 0 end_POSTSUBSCRIPT still semi-conjugates between HK,θ,csubscript𝐻𝐾𝜃𝑐H_{K,\theta,c}italic_H start_POSTSUBSCRIPT italic_K , italic_θ , italic_c end_POSTSUBSCRIPT in a neighborhood of \infty and the squaring map.

Lemma 3.7.

Let K>1𝐾1K>1italic_K > 1, θ(π/2,π/2]𝜃𝜋2𝜋2\theta\in(-\pi/2,\pi/2]italic_θ ∈ ( - italic_π / 2 , italic_π / 2 ] and c𝑐c\in\mathbb{C}italic_c ∈ blackboard_C. For any zWK,θ,c𝑧subscript𝑊𝐾𝜃𝑐z\in W_{K,\theta,c}italic_z ∈ italic_W start_POSTSUBSCRIPT italic_K , italic_θ , italic_c end_POSTSUBSCRIPT,

τK,θ,c(HK,θ,c(z))=[τK,θ,c(z)]2.subscript𝜏𝐾𝜃𝑐subscript𝐻𝐾𝜃𝑐𝑧superscriptdelimited-[]subscript𝜏𝐾𝜃𝑐𝑧2\tau_{K,\theta,c}(H_{K,\theta,c}(z))=[\tau_{K,\theta,c}(z)]^{2}.italic_τ start_POSTSUBSCRIPT italic_K , italic_θ , italic_c end_POSTSUBSCRIPT ( italic_H start_POSTSUBSCRIPT italic_K , italic_θ , italic_c end_POSTSUBSCRIPT ( italic_z ) ) = [ italic_τ start_POSTSUBSCRIPT italic_K , italic_θ , italic_c end_POSTSUBSCRIPT ( italic_z ) ] start_POSTSUPERSCRIPT 2 end_POSTSUPERSCRIPT .
Proof.

By Lemma 3.5 and Theorem 2.5, for zWK,θ,c𝑧subscript𝑊𝐾𝜃𝑐z\in W_{K,\theta,c}italic_z ∈ italic_W start_POSTSUBSCRIPT italic_K , italic_θ , italic_c end_POSTSUBSCRIPT, we have

τK,θ,c(HK,θ,c(z))subscript𝜏𝐾𝜃𝑐subscript𝐻𝐾𝜃𝑐𝑧\displaystyle\tau_{K,\theta,c}(H_{K,\theta,c}(z))italic_τ start_POSTSUBSCRIPT italic_K , italic_θ , italic_c end_POSTSUBSCRIPT ( italic_H start_POSTSUBSCRIPT italic_K , italic_θ , italic_c end_POSTSUBSCRIPT ( italic_z ) ) =τK,θ,0(φK,θ,cHK,θ,c)(z)absentsubscript𝜏𝐾𝜃0subscript𝜑𝐾𝜃𝑐subscript𝐻𝐾𝜃𝑐𝑧\displaystyle=\tau_{K,\theta,0}\circ(\varphi_{K,\theta,c}\circ H_{K,\theta,c})% (z)= italic_τ start_POSTSUBSCRIPT italic_K , italic_θ , 0 end_POSTSUBSCRIPT ∘ ( italic_φ start_POSTSUBSCRIPT italic_K , italic_θ , italic_c end_POSTSUBSCRIPT ∘ italic_H start_POSTSUBSCRIPT italic_K , italic_θ , italic_c end_POSTSUBSCRIPT ) ( italic_z )
=τK,θ,0(HK,θ,0φK,θ,c)(z)absentsubscript𝜏𝐾𝜃0subscript𝐻𝐾𝜃0subscript𝜑𝐾𝜃𝑐𝑧\displaystyle=\tau_{K,\theta,0}\circ(H_{K,\theta,0}\circ\varphi_{K,\theta,c})(z)= italic_τ start_POSTSUBSCRIPT italic_K , italic_θ , 0 end_POSTSUBSCRIPT ∘ ( italic_H start_POSTSUBSCRIPT italic_K , italic_θ , 0 end_POSTSUBSCRIPT ∘ italic_φ start_POSTSUBSCRIPT italic_K , italic_θ , italic_c end_POSTSUBSCRIPT ) ( italic_z )
=(τK,θ,0HK,θ,0)φK,θ,c(z)absentsubscript𝜏𝐾𝜃0subscript𝐻𝐾𝜃0subscript𝜑𝐾𝜃𝑐𝑧\displaystyle=(\tau_{K,\theta,0}\circ H_{K,\theta,0})\circ\varphi_{K,\theta,c}% (z)= ( italic_τ start_POSTSUBSCRIPT italic_K , italic_θ , 0 end_POSTSUBSCRIPT ∘ italic_H start_POSTSUBSCRIPT italic_K , italic_θ , 0 end_POSTSUBSCRIPT ) ∘ italic_φ start_POSTSUBSCRIPT italic_K , italic_θ , italic_c end_POSTSUBSCRIPT ( italic_z )
=[τK,θ,0(φK,θ,c(z))]2absentsuperscriptdelimited-[]subscript𝜏𝐾𝜃0subscript𝜑𝐾𝜃𝑐𝑧2\displaystyle=[\tau_{K,\theta,0}(\varphi_{K,\theta,c}(z))]^{2}= [ italic_τ start_POSTSUBSCRIPT italic_K , italic_θ , 0 end_POSTSUBSCRIPT ( italic_φ start_POSTSUBSCRIPT italic_K , italic_θ , italic_c end_POSTSUBSCRIPT ( italic_z ) ) ] start_POSTSUPERSCRIPT 2 end_POSTSUPERSCRIPT
=[τK,θ,c(z)]2.absentsuperscriptdelimited-[]subscript𝜏𝐾𝜃𝑐𝑧2\displaystyle=[\tau_{K,\theta,c}(z)]^{2}.= [ italic_τ start_POSTSUBSCRIPT italic_K , italic_θ , italic_c end_POSTSUBSCRIPT ( italic_z ) ] start_POSTSUPERSCRIPT 2 end_POSTSUPERSCRIPT .

Recall from (1.3) that the Green’s function Gcsubscript𝐺𝑐G_{c}italic_G start_POSTSUBSCRIPT italic_c end_POSTSUBSCRIPT semi-conjugates between Pcsubscript𝑃𝑐P_{c}italic_P start_POSTSUBSCRIPT italic_c end_POSTSUBSCRIPT and multiplication by 2222. To construct our analogue of the Green’s function, we may now just modify τK,θ,csubscript𝜏𝐾𝜃𝑐\tau_{K,\theta,c}italic_τ start_POSTSUBSCRIPT italic_K , italic_θ , italic_c end_POSTSUBSCRIPT as follows.

Definition 3.8.

Let K>1𝐾1K>1italic_K > 1, θ(π/2,π/2]𝜃𝜋2𝜋2\theta\in(-\pi/2,\pi/2]italic_θ ∈ ( - italic_π / 2 , italic_π / 2 ] and c𝑐c\in\mathbb{C}italic_c ∈ blackboard_C. We define GK,θ,c:WK,θ,c+:subscript𝐺𝐾𝜃𝑐subscript𝑊𝐾𝜃𝑐superscriptG_{K,\theta,c}:W_{K,\theta,c}\to\mathbb{R}^{+}italic_G start_POSTSUBSCRIPT italic_K , italic_θ , italic_c end_POSTSUBSCRIPT : italic_W start_POSTSUBSCRIPT italic_K , italic_θ , italic_c end_POSTSUBSCRIPT → blackboard_R start_POSTSUPERSCRIPT + end_POSTSUPERSCRIPT by

GK,θ,c(z)=log(τK,θ,c(z)).subscript𝐺𝐾𝜃𝑐𝑧subscript𝜏𝐾𝜃𝑐𝑧G_{K,\theta,c}(z)=\log(\tau_{K,\theta,c}(z)).italic_G start_POSTSUBSCRIPT italic_K , italic_θ , italic_c end_POSTSUBSCRIPT ( italic_z ) = roman_log ( italic_τ start_POSTSUBSCRIPT italic_K , italic_θ , italic_c end_POSTSUBSCRIPT ( italic_z ) ) .

It is then clear from Lemma 3.7 that for zWK,θ,c𝑧subscript𝑊𝐾𝜃𝑐z\in W_{K,\theta,c}italic_z ∈ italic_W start_POSTSUBSCRIPT italic_K , italic_θ , italic_c end_POSTSUBSCRIPT, we have

(3.1) GK,θ,c(HK,θ,c(z))=2GK,θ,c(z).subscript𝐺𝐾𝜃𝑐subscript𝐻𝐾𝜃𝑐𝑧2subscript𝐺𝐾𝜃𝑐𝑧G_{K,\theta,c}(H_{K,\theta,c}(z))=2G_{K,\theta,c}(z).italic_G start_POSTSUBSCRIPT italic_K , italic_θ , italic_c end_POSTSUBSCRIPT ( italic_H start_POSTSUBSCRIPT italic_K , italic_θ , italic_c end_POSTSUBSCRIPT ( italic_z ) ) = 2 italic_G start_POSTSUBSCRIPT italic_K , italic_θ , italic_c end_POSTSUBSCRIPT ( italic_z ) .

We are now a long way towards proving Theorem 1.1. The remaining task is to show that we can extend the domain of definition of GK,θ,csubscript𝐺𝐾𝜃𝑐G_{K,\theta,c}italic_G start_POSTSUBSCRIPT italic_K , italic_θ , italic_c end_POSTSUBSCRIPT to all of \mathbb{C}blackboard_C. Recall that the backward orbit of a set X𝑋Xitalic_X under a map f𝑓fitalic_f is

Of(X)=n=0fn(X).superscriptsubscript𝑂𝑓𝑋superscriptsubscript𝑛0superscript𝑓𝑛𝑋O_{f}^{-}(X)=\bigcup_{n=0}^{\infty}f^{-n}(X).italic_O start_POSTSUBSCRIPT italic_f end_POSTSUBSCRIPT start_POSTSUPERSCRIPT - end_POSTSUPERSCRIPT ( italic_X ) = ⋃ start_POSTSUBSCRIPT italic_n = 0 end_POSTSUBSCRIPT start_POSTSUPERSCRIPT ∞ end_POSTSUPERSCRIPT italic_f start_POSTSUPERSCRIPT - italic_n end_POSTSUPERSCRIPT ( italic_X ) .

If the context is clear, we will just write O(X)superscript𝑂𝑋O^{-}(X)italic_O start_POSTSUPERSCRIPT - end_POSTSUPERSCRIPT ( italic_X ).

Lemma 3.9.

Let K>1𝐾1K>1italic_K > 1, θ(π/2,π/2]𝜃𝜋2𝜋2\theta\in(-\pi/2,\pi/2]italic_θ ∈ ( - italic_π / 2 , italic_π / 2 ] and c𝑐c\in\mathbb{C}italic_c ∈ blackboard_C. If U𝑈Uitalic_U is a neighborhood of infinity contained in I(HK,θ,c)𝐼subscript𝐻𝐾𝜃𝑐I(H_{K,\theta,c})italic_I ( italic_H start_POSTSUBSCRIPT italic_K , italic_θ , italic_c end_POSTSUBSCRIPT ), then

I(HK,θ,c)=O(U).𝐼subscript𝐻𝐾𝜃𝑐superscript𝑂𝑈I(H_{K,\theta,c})=O^{-}(U).italic_I ( italic_H start_POSTSUBSCRIPT italic_K , italic_θ , italic_c end_POSTSUBSCRIPT ) = italic_O start_POSTSUPERSCRIPT - end_POSTSUPERSCRIPT ( italic_U ) .
Proof.

By the complete invariance of the escaping set, it is clear that OHK,θ,c(U)BO(HK,θ,c)=superscriptsubscript𝑂subscript𝐻𝐾𝜃𝑐𝑈𝐵𝑂subscript𝐻𝐾𝜃𝑐O_{H_{K,\theta,c}}^{-}(U)\cap BO(H_{K,\theta,c})=\emptysetitalic_O start_POSTSUBSCRIPT italic_H start_POSTSUBSCRIPT italic_K , italic_θ , italic_c end_POSTSUBSCRIPT end_POSTSUBSCRIPT start_POSTSUPERSCRIPT - end_POSTSUPERSCRIPT ( italic_U ) ∩ italic_B italic_O ( italic_H start_POSTSUBSCRIPT italic_K , italic_θ , italic_c end_POSTSUBSCRIPT ) = ∅ and so

O(U)I(HK,θ,c).superscript𝑂𝑈𝐼subscript𝐻𝐾𝜃𝑐O^{-}(U)\subset I(H_{K,\theta,c}).italic_O start_POSTSUPERSCRIPT - end_POSTSUPERSCRIPT ( italic_U ) ⊂ italic_I ( italic_H start_POSTSUBSCRIPT italic_K , italic_θ , italic_c end_POSTSUBSCRIPT ) .

On the other hand, if zI(HK,θ,c)𝑧𝐼subscript𝐻𝐾𝜃𝑐z\in I(H_{K,\theta,c})italic_z ∈ italic_I ( italic_H start_POSTSUBSCRIPT italic_K , italic_θ , italic_c end_POSTSUBSCRIPT ), then for a large enough iterate, fn(z)Usuperscript𝑓𝑛𝑧𝑈f^{n}(z)\in Uitalic_f start_POSTSUPERSCRIPT italic_n end_POSTSUPERSCRIPT ( italic_z ) ∈ italic_U and so

I(HK,θ,c)O(U),𝐼subscript𝐻𝐾𝜃𝑐superscript𝑂𝑈I(H_{K,\theta,c})\subset O^{-}(U),italic_I ( italic_H start_POSTSUBSCRIPT italic_K , italic_θ , italic_c end_POSTSUBSCRIPT ) ⊂ italic_O start_POSTSUPERSCRIPT - end_POSTSUPERSCRIPT ( italic_U ) ,

which completes the proof. ∎

We can now complete the proof of Theorem 1.1

Proof of Theorem 1.1.

From Lemma 3.7 and (3.1), we have the existence of GK,θ,csubscript𝐺𝐾𝜃𝑐G_{K,\theta,c}italic_G start_POSTSUBSCRIPT italic_K , italic_θ , italic_c end_POSTSUBSCRIPT in WK,θ,csubscript𝑊𝐾𝜃𝑐W_{K,\theta,c}italic_W start_POSTSUBSCRIPT italic_K , italic_θ , italic_c end_POSTSUBSCRIPT and the required functional equation is satisfied there. The idea is to pullback the domain of definition of GK,θ,csubscript𝐺𝐾𝜃𝑐G_{K,\theta,c}italic_G start_POSTSUBSCRIPT italic_K , italic_θ , italic_c end_POSTSUBSCRIPT via the functional equation. That is, if zHK,θ,cn(WK,θ,c)𝑧superscriptsubscript𝐻𝐾𝜃𝑐𝑛subscript𝑊𝐾𝜃𝑐z\in H_{K,\theta,c}^{-n}(W_{K,\theta,c})italic_z ∈ italic_H start_POSTSUBSCRIPT italic_K , italic_θ , italic_c end_POSTSUBSCRIPT start_POSTSUPERSCRIPT - italic_n end_POSTSUPERSCRIPT ( italic_W start_POSTSUBSCRIPT italic_K , italic_θ , italic_c end_POSTSUBSCRIPT ), we define

(3.2) GK,θ,c(z)=GK,θ,c(HK,θ,cn(z))2n.subscript𝐺𝐾𝜃𝑐𝑧subscript𝐺𝐾𝜃𝑐subscriptsuperscript𝐻𝑛𝐾𝜃𝑐𝑧superscript2𝑛G_{K,\theta,c}(z)=\frac{G_{K,\theta,c}(H^{n}_{K,\theta,c}(z))}{2^{n}}.italic_G start_POSTSUBSCRIPT italic_K , italic_θ , italic_c end_POSTSUBSCRIPT ( italic_z ) = divide start_ARG italic_G start_POSTSUBSCRIPT italic_K , italic_θ , italic_c end_POSTSUBSCRIPT ( italic_H start_POSTSUPERSCRIPT italic_n end_POSTSUPERSCRIPT start_POSTSUBSCRIPT italic_K , italic_θ , italic_c end_POSTSUBSCRIPT ( italic_z ) ) end_ARG start_ARG 2 start_POSTSUPERSCRIPT italic_n end_POSTSUPERSCRIPT end_ARG .

Let us show that this is well-defined, that is, it is independent of the specific choice of n𝑛nitalic_n. Suppose that zHK,θ,cn1(WK,θ,c)HK,θ,cn2(WK,θ,c)𝑧subscriptsuperscript𝐻subscript𝑛1𝐾𝜃𝑐subscript𝑊𝐾𝜃𝑐subscriptsuperscript𝐻subscript𝑛2𝐾𝜃𝑐subscript𝑊𝐾𝜃𝑐z\in H^{-n_{1}}_{K,\theta,c}(W_{K,\theta,c})\cap H^{-n_{2}}_{K,\theta,c}(W_{K,% \theta,c})italic_z ∈ italic_H start_POSTSUPERSCRIPT - italic_n start_POSTSUBSCRIPT 1 end_POSTSUBSCRIPT end_POSTSUPERSCRIPT start_POSTSUBSCRIPT italic_K , italic_θ , italic_c end_POSTSUBSCRIPT ( italic_W start_POSTSUBSCRIPT italic_K , italic_θ , italic_c end_POSTSUBSCRIPT ) ∩ italic_H start_POSTSUPERSCRIPT - italic_n start_POSTSUBSCRIPT 2 end_POSTSUBSCRIPT end_POSTSUPERSCRIPT start_POSTSUBSCRIPT italic_K , italic_θ , italic_c end_POSTSUBSCRIPT ( italic_W start_POSTSUBSCRIPT italic_K , italic_θ , italic_c end_POSTSUBSCRIPT ), where n1<n2subscript𝑛1subscript𝑛2n_{1}<n_{2}italic_n start_POSTSUBSCRIPT 1 end_POSTSUBSCRIPT < italic_n start_POSTSUBSCRIPT 2 end_POSTSUBSCRIPT. By (3.1), we have

GK,θ,c(z)subscript𝐺𝐾𝜃𝑐𝑧\displaystyle G_{K,\theta,c}(z)italic_G start_POSTSUBSCRIPT italic_K , italic_θ , italic_c end_POSTSUBSCRIPT ( italic_z ) =GK,θ,c(HK,θ,cn2(z))2n2absentsubscript𝐺𝐾𝜃𝑐subscriptsuperscript𝐻subscript𝑛2𝐾𝜃𝑐𝑧superscript2subscript𝑛2\displaystyle=\frac{G_{K,\theta,c}(H^{n_{2}}_{K,\theta,c}(z))}{2^{n_{2}}}= divide start_ARG italic_G start_POSTSUBSCRIPT italic_K , italic_θ , italic_c end_POSTSUBSCRIPT ( italic_H start_POSTSUPERSCRIPT italic_n start_POSTSUBSCRIPT 2 end_POSTSUBSCRIPT end_POSTSUPERSCRIPT start_POSTSUBSCRIPT italic_K , italic_θ , italic_c end_POSTSUBSCRIPT ( italic_z ) ) end_ARG start_ARG 2 start_POSTSUPERSCRIPT italic_n start_POSTSUBSCRIPT 2 end_POSTSUBSCRIPT end_POSTSUPERSCRIPT end_ARG
=GK,θ,c(HK,θ,cn2n1(HK,θ,cn1(z)))2n2absentsubscript𝐺𝐾𝜃𝑐subscriptsuperscript𝐻subscript𝑛2subscript𝑛1𝐾𝜃𝑐subscriptsuperscript𝐻subscript𝑛1𝐾𝜃𝑐𝑧superscript2subscript𝑛2\displaystyle=\frac{G_{K,\theta,c}(H^{n_{2}-n_{1}}_{K,\theta,c}(H^{n_{1}}_{K,% \theta,c}(z)))}{2^{n_{2}}}= divide start_ARG italic_G start_POSTSUBSCRIPT italic_K , italic_θ , italic_c end_POSTSUBSCRIPT ( italic_H start_POSTSUPERSCRIPT italic_n start_POSTSUBSCRIPT 2 end_POSTSUBSCRIPT - italic_n start_POSTSUBSCRIPT 1 end_POSTSUBSCRIPT end_POSTSUPERSCRIPT start_POSTSUBSCRIPT italic_K , italic_θ , italic_c end_POSTSUBSCRIPT ( italic_H start_POSTSUPERSCRIPT italic_n start_POSTSUBSCRIPT 1 end_POSTSUBSCRIPT end_POSTSUPERSCRIPT start_POSTSUBSCRIPT italic_K , italic_θ , italic_c end_POSTSUBSCRIPT ( italic_z ) ) ) end_ARG start_ARG 2 start_POSTSUPERSCRIPT italic_n start_POSTSUBSCRIPT 2 end_POSTSUBSCRIPT end_POSTSUPERSCRIPT end_ARG
=2n2n1GK,θ,c(HK,θ,cn1(z))2n2absentsuperscript2subscript𝑛2subscript𝑛1subscript𝐺𝐾𝜃𝑐subscriptsuperscript𝐻subscript𝑛1𝐾𝜃𝑐𝑧superscript2subscript𝑛2\displaystyle=\frac{2^{n_{2}-n_{1}}G_{K,\theta,c}(H^{n_{1}}_{K,\theta,c}(z))}{% 2^{n_{2}}}= divide start_ARG 2 start_POSTSUPERSCRIPT italic_n start_POSTSUBSCRIPT 2 end_POSTSUBSCRIPT - italic_n start_POSTSUBSCRIPT 1 end_POSTSUBSCRIPT end_POSTSUPERSCRIPT italic_G start_POSTSUBSCRIPT italic_K , italic_θ , italic_c end_POSTSUBSCRIPT ( italic_H start_POSTSUPERSCRIPT italic_n start_POSTSUBSCRIPT 1 end_POSTSUBSCRIPT end_POSTSUPERSCRIPT start_POSTSUBSCRIPT italic_K , italic_θ , italic_c end_POSTSUBSCRIPT ( italic_z ) ) end_ARG start_ARG 2 start_POSTSUPERSCRIPT italic_n start_POSTSUBSCRIPT 2 end_POSTSUBSCRIPT end_POSTSUPERSCRIPT end_ARG
=GK,θ,c(HK,θ,cn1(z))2n1.absentsubscript𝐺𝐾𝜃𝑐subscriptsuperscript𝐻subscript𝑛1𝐾𝜃𝑐𝑧superscript2subscript𝑛1\displaystyle=\frac{G_{K,\theta,c}(H^{n_{1}}_{K,\theta,c}(z))}{2^{n_{1}}}.= divide start_ARG italic_G start_POSTSUBSCRIPT italic_K , italic_θ , italic_c end_POSTSUBSCRIPT ( italic_H start_POSTSUPERSCRIPT italic_n start_POSTSUBSCRIPT 1 end_POSTSUBSCRIPT end_POSTSUPERSCRIPT start_POSTSUBSCRIPT italic_K , italic_θ , italic_c end_POSTSUBSCRIPT ( italic_z ) ) end_ARG start_ARG 2 start_POSTSUPERSCRIPT italic_n start_POSTSUBSCRIPT 1 end_POSTSUBSCRIPT end_POSTSUPERSCRIPT end_ARG .

Further, this extension of GK,θ,csubscript𝐺𝐾𝜃𝑐G_{K,\theta,c}italic_G start_POSTSUBSCRIPT italic_K , italic_θ , italic_c end_POSTSUBSCRIPT to HK,θ,cn(WK,θ,c)superscriptsubscript𝐻𝐾𝜃𝑐𝑛subscript𝑊𝐾𝜃𝑐H_{K,\theta,c}^{-n}(W_{K,\theta,c})italic_H start_POSTSUBSCRIPT italic_K , italic_θ , italic_c end_POSTSUBSCRIPT start_POSTSUPERSCRIPT - italic_n end_POSTSUPERSCRIPT ( italic_W start_POSTSUBSCRIPT italic_K , italic_θ , italic_c end_POSTSUBSCRIPT ) is continuous as all the maps in the composition given in (3.2) are continuous. By Lemma 3.9, we may extend GK,θ,csubscript𝐺𝐾𝜃𝑐G_{K,\theta,c}italic_G start_POSTSUBSCRIPT italic_K , italic_θ , italic_c end_POSTSUBSCRIPT continuously to all of I(HK,θ,c)𝐼subscript𝐻𝐾𝜃𝑐I(H_{K,\theta,c})italic_I ( italic_H start_POSTSUBSCRIPT italic_K , italic_θ , italic_c end_POSTSUBSCRIPT ) and, moreover, (3.2) implies that we have (3.1) on all of I(HK,θ,c)𝐼subscript𝐻𝐾𝜃𝑐I(H_{K,\theta,c})italic_I ( italic_H start_POSTSUBSCRIPT italic_K , italic_θ , italic_c end_POSTSUBSCRIPT ).

Next, we show that GK,θ,c(z)0subscript𝐺𝐾𝜃𝑐𝑧0G_{K,\theta,c}(z)\to 0italic_G start_POSTSUBSCRIPT italic_K , italic_θ , italic_c end_POSTSUBSCRIPT ( italic_z ) → 0 as zI(HK,θ,c)𝑧𝐼subscript𝐻𝐾𝜃𝑐z\to\partial I(H_{K,\theta,c})italic_z → ∂ italic_I ( italic_H start_POSTSUBSCRIPT italic_K , italic_θ , italic_c end_POSTSUBSCRIPT ). We may assume that the open neighborhood of infinity WK,θ,csubscript𝑊𝐾𝜃𝑐W_{K,\theta,c}italic_W start_POSTSUBSCRIPT italic_K , italic_θ , italic_c end_POSTSUBSCRIPT is a strictly positive distance from I(HK,θ,c)𝐼subscript𝐻𝐾𝜃𝑐\partial I(H_{K,\theta,c})∂ italic_I ( italic_H start_POSTSUBSCRIPT italic_K , italic_θ , italic_c end_POSTSUBSCRIPT ). Fix N𝑁N\in\mathbb{N}italic_N ∈ blackboard_N and let

U=HK,θ,c(N+1)(WK,θ,c)¯HK,θ,cN(WK,θ,c).𝑈¯subscriptsuperscript𝐻𝑁1𝐾𝜃𝑐subscript𝑊𝐾𝜃𝑐subscriptsuperscript𝐻𝑁𝐾𝜃𝑐subscript𝑊𝐾𝜃𝑐U=\overline{H^{-(N+1)}_{K,\theta,c}(W_{K,\theta,c})}\setminus H^{-N}_{K,\theta% ,c}(W_{K,\theta,c}).italic_U = over¯ start_ARG italic_H start_POSTSUPERSCRIPT - ( italic_N + 1 ) end_POSTSUPERSCRIPT start_POSTSUBSCRIPT italic_K , italic_θ , italic_c end_POSTSUBSCRIPT ( italic_W start_POSTSUBSCRIPT italic_K , italic_θ , italic_c end_POSTSUBSCRIPT ) end_ARG ∖ italic_H start_POSTSUPERSCRIPT - italic_N end_POSTSUPERSCRIPT start_POSTSUBSCRIPT italic_K , italic_θ , italic_c end_POSTSUBSCRIPT ( italic_W start_POSTSUBSCRIPT italic_K , italic_θ , italic_c end_POSTSUBSCRIPT ) .

As U𝑈Uitalic_U is a compact set and GK,θ,csubscript𝐺𝐾𝜃𝑐G_{K,\theta,c}italic_G start_POSTSUBSCRIPT italic_K , italic_θ , italic_c end_POSTSUBSCRIPT is a continuous function, we have

supzUGK,θ,c(z)=T<.subscriptsupremum𝑧𝑈subscript𝐺𝐾𝜃𝑐𝑧𝑇\sup_{z\in U}G_{K,\theta,c}(z)=T<\infty.roman_sup start_POSTSUBSCRIPT italic_z ∈ italic_U end_POSTSUBSCRIPT italic_G start_POSTSUBSCRIPT italic_K , italic_θ , italic_c end_POSTSUBSCRIPT ( italic_z ) = italic_T < ∞ .

It then follows by construction that every component of HK,θ,c1(U)superscriptsubscript𝐻𝐾𝜃𝑐1𝑈H_{K,\theta,c}^{-1}(U)italic_H start_POSTSUBSCRIPT italic_K , italic_θ , italic_c end_POSTSUBSCRIPT start_POSTSUPERSCRIPT - 1 end_POSTSUPERSCRIPT ( italic_U ) is contained in a bounded component of the complement of U𝑈Uitalic_U. Moreover, by (3.2), for zHK,θ,c1(U)𝑧superscriptsubscript𝐻𝐾𝜃𝑐1𝑈z\in H_{K,\theta,c}^{-1}(U)italic_z ∈ italic_H start_POSTSUBSCRIPT italic_K , italic_θ , italic_c end_POSTSUBSCRIPT start_POSTSUPERSCRIPT - 1 end_POSTSUPERSCRIPT ( italic_U ) we have

GK,θ,c(z)=GK,θ,c(HK,θ,c)2T2.subscript𝐺𝐾𝜃𝑐𝑧subscript𝐺𝐾𝜃𝑐subscript𝐻𝐾𝜃𝑐2𝑇2G_{K,\theta,c}(z)=\frac{G_{K,\theta,c}(H_{K,\theta,c})}{2}\leq\frac{T}{2}.italic_G start_POSTSUBSCRIPT italic_K , italic_θ , italic_c end_POSTSUBSCRIPT ( italic_z ) = divide start_ARG italic_G start_POSTSUBSCRIPT italic_K , italic_θ , italic_c end_POSTSUBSCRIPT ( italic_H start_POSTSUBSCRIPT italic_K , italic_θ , italic_c end_POSTSUBSCRIPT ) end_ARG start_ARG 2 end_ARG ≤ divide start_ARG italic_T end_ARG start_ARG 2 end_ARG .

By induction, for zHK,θ,cn(U)𝑧superscriptsubscript𝐻𝐾𝜃𝑐𝑛𝑈z\in H_{K,\theta,c}^{-n}(U)italic_z ∈ italic_H start_POSTSUBSCRIPT italic_K , italic_θ , italic_c end_POSTSUBSCRIPT start_POSTSUPERSCRIPT - italic_n end_POSTSUPERSCRIPT ( italic_U ), we have

GK,θ,c(z)T2n.subscript𝐺𝐾𝜃𝑐𝑧𝑇superscript2𝑛G_{K,\theta,c}(z)\leq\frac{T}{2^{n}}.italic_G start_POSTSUBSCRIPT italic_K , italic_θ , italic_c end_POSTSUBSCRIPT ( italic_z ) ≤ divide start_ARG italic_T end_ARG start_ARG 2 start_POSTSUPERSCRIPT italic_n end_POSTSUPERSCRIPT end_ARG .

From this we conclude that GK,θ,c(z)0subscript𝐺𝐾𝜃𝑐𝑧0G_{K,\theta,c}(z)\to 0italic_G start_POSTSUBSCRIPT italic_K , italic_θ , italic_c end_POSTSUBSCRIPT ( italic_z ) → 0 as zI(HK,θ,c)𝑧𝐼subscript𝐻𝐾𝜃𝑐z\to\partial I(H_{K,\theta,c})italic_z → ∂ italic_I ( italic_H start_POSTSUBSCRIPT italic_K , italic_θ , italic_c end_POSTSUBSCRIPT ). We may therefore extend GK,θ,csubscript𝐺𝐾𝜃𝑐G_{K,\theta,c}italic_G start_POSTSUBSCRIPT italic_K , italic_θ , italic_c end_POSTSUBSCRIPT to all of \mathbb{C}blackboard_C by setting it equal to 00 on BO(HK,θ,c)𝐵𝑂subscript𝐻𝐾𝜃𝑐BO(H_{K,\theta,c})italic_B italic_O ( italic_H start_POSTSUBSCRIPT italic_K , italic_θ , italic_c end_POSTSUBSCRIPT ). This completes the proof. ∎

4. Equipotentials

We use our version of Green’s function to define equipotentials.

Definition 4.1.

Let K>1𝐾1K>1italic_K > 1, θ(π/2,π/2]𝜃𝜋2𝜋2\theta\in(-\pi/2,\pi/2]italic_θ ∈ ( - italic_π / 2 , italic_π / 2 ] and c𝑐c\in\mathbb{C}italic_c ∈ blackboard_C. For any t>0𝑡0t>0italic_t > 0, we define

EK,θ,c(t)={z:GK,θ,c(z)=t}subscript𝐸𝐾𝜃𝑐𝑡conditional-set𝑧subscript𝐺𝐾𝜃𝑐𝑧𝑡E_{K,\theta,c}(t)=\{z\in\mathbb{C}:G_{K,\theta,c}(z)=t\}italic_E start_POSTSUBSCRIPT italic_K , italic_θ , italic_c end_POSTSUBSCRIPT ( italic_t ) = { italic_z ∈ blackboard_C : italic_G start_POSTSUBSCRIPT italic_K , italic_θ , italic_c end_POSTSUBSCRIPT ( italic_z ) = italic_t }

and

UK,θ,c(t)={z:GK,θ,c(z)>t}.subscript𝑈𝐾𝜃𝑐𝑡conditional-set𝑧subscript𝐺𝐾𝜃𝑐𝑧𝑡U_{K,\theta,c}(t)=\{z\in\mathbb{C}:G_{K,\theta,c}(z)>t\}.italic_U start_POSTSUBSCRIPT italic_K , italic_θ , italic_c end_POSTSUBSCRIPT ( italic_t ) = { italic_z ∈ blackboard_C : italic_G start_POSTSUBSCRIPT italic_K , italic_θ , italic_c end_POSTSUBSCRIPT ( italic_z ) > italic_t } .

We could also make these definitions for t=0𝑡0t=0italic_t = 0, but in this case EK,θ,c(0)=BO(HK,θ,c)subscript𝐸𝐾𝜃𝑐0𝐵𝑂subscript𝐻𝐾𝜃𝑐E_{K,\theta,c}(0)=BO(H_{K,\theta,c})italic_E start_POSTSUBSCRIPT italic_K , italic_θ , italic_c end_POSTSUBSCRIPT ( 0 ) = italic_B italic_O ( italic_H start_POSTSUBSCRIPT italic_K , italic_θ , italic_c end_POSTSUBSCRIPT ) and UK,θ,c(0)=I(HK,θ,c)subscript𝑈𝐾𝜃𝑐0𝐼subscript𝐻𝐾𝜃𝑐U_{K,\theta,c}(0)=I(H_{K,\theta,c})italic_U start_POSTSUBSCRIPT italic_K , italic_θ , italic_c end_POSTSUBSCRIPT ( 0 ) = italic_I ( italic_H start_POSTSUBSCRIPT italic_K , italic_θ , italic_c end_POSTSUBSCRIPT ). If the context is clear, we will just write E(t)𝐸𝑡E(t)italic_E ( italic_t ) and U(t)𝑈𝑡U(t)italic_U ( italic_t ).

Lemma 4.2.

Let K>1𝐾1K>1italic_K > 1, θ(π/2,π/2]𝜃𝜋2𝜋2\theta\in(-\pi/2,\pi/2]italic_θ ∈ ( - italic_π / 2 , italic_π / 2 ] and c𝑐c\in\mathbb{C}italic_c ∈ blackboard_C. For any n𝑛n\in\mathbb{N}italic_n ∈ blackboard_N and any t>0𝑡0t>0italic_t > 0, we have

E(2nt)=HK,θ,cn(E(t)) and U(2nt)=HK,θ,cn(U(t)).formulae-sequence𝐸superscript2𝑛𝑡superscriptsubscript𝐻𝐾𝜃𝑐𝑛𝐸𝑡 and 𝑈superscript2𝑛𝑡superscriptsubscript𝐻𝐾𝜃𝑐𝑛𝑈𝑡E(2^{n}t)=H_{K,\theta,c}^{n}(E(t))\quad\text{ and }\quad U(2^{n}t)=H_{K,\theta% ,c}^{n}(U(t)).italic_E ( 2 start_POSTSUPERSCRIPT italic_n end_POSTSUPERSCRIPT italic_t ) = italic_H start_POSTSUBSCRIPT italic_K , italic_θ , italic_c end_POSTSUBSCRIPT start_POSTSUPERSCRIPT italic_n end_POSTSUPERSCRIPT ( italic_E ( italic_t ) ) and italic_U ( 2 start_POSTSUPERSCRIPT italic_n end_POSTSUPERSCRIPT italic_t ) = italic_H start_POSTSUBSCRIPT italic_K , italic_θ , italic_c end_POSTSUBSCRIPT start_POSTSUPERSCRIPT italic_n end_POSTSUPERSCRIPT ( italic_U ( italic_t ) ) .
Proof.

Suppose that zE(2nt)𝑧𝐸superscript2𝑛𝑡z\in E(2^{n}t)italic_z ∈ italic_E ( 2 start_POSTSUPERSCRIPT italic_n end_POSTSUPERSCRIPT italic_t ) so that GK,θ,c(z)=2ntsubscript𝐺𝐾𝜃𝑐𝑧superscript2𝑛𝑡G_{K,\theta,c}(z)=2^{n}titalic_G start_POSTSUBSCRIPT italic_K , italic_θ , italic_c end_POSTSUBSCRIPT ( italic_z ) = 2 start_POSTSUPERSCRIPT italic_n end_POSTSUPERSCRIPT italic_t. For any w𝑤w\in\mathbb{C}italic_w ∈ blackboard_C such that HK,θ,cn(w)=zsubscriptsuperscript𝐻𝑛𝐾𝜃𝑐𝑤𝑧H^{n}_{K,\theta,c}(w)=zitalic_H start_POSTSUPERSCRIPT italic_n end_POSTSUPERSCRIPT start_POSTSUBSCRIPT italic_K , italic_θ , italic_c end_POSTSUBSCRIPT ( italic_w ) = italic_z, by Theorem 1.1, we have

t=GK,θ,c(z)2n=GK,θ,c(HK,θ,cn(w))2n=GK,θ,c(w)𝑡subscript𝐺𝐾𝜃𝑐𝑧superscript2𝑛subscript𝐺𝐾𝜃𝑐superscriptsubscript𝐻𝐾𝜃𝑐𝑛𝑤superscript2𝑛subscript𝐺𝐾𝜃𝑐𝑤t=\frac{G_{K,\theta,c}(z)}{2^{n}}=\frac{G_{K,\theta,c}(H_{K,\theta,c}^{n}(w))}% {2^{n}}=G_{K,\theta,c}(w)italic_t = divide start_ARG italic_G start_POSTSUBSCRIPT italic_K , italic_θ , italic_c end_POSTSUBSCRIPT ( italic_z ) end_ARG start_ARG 2 start_POSTSUPERSCRIPT italic_n end_POSTSUPERSCRIPT end_ARG = divide start_ARG italic_G start_POSTSUBSCRIPT italic_K , italic_θ , italic_c end_POSTSUBSCRIPT ( italic_H start_POSTSUBSCRIPT italic_K , italic_θ , italic_c end_POSTSUBSCRIPT start_POSTSUPERSCRIPT italic_n end_POSTSUPERSCRIPT ( italic_w ) ) end_ARG start_ARG 2 start_POSTSUPERSCRIPT italic_n end_POSTSUPERSCRIPT end_ARG = italic_G start_POSTSUBSCRIPT italic_K , italic_θ , italic_c end_POSTSUBSCRIPT ( italic_w )

and wE(t)𝑤𝐸𝑡w\in E(t)italic_w ∈ italic_E ( italic_t ). Thus zHK,θ,cn(E(t))𝑧superscriptsubscript𝐻𝐾𝜃𝑐𝑛𝐸𝑡z\in H_{K,\theta,c}^{n}(E(t))italic_z ∈ italic_H start_POSTSUBSCRIPT italic_K , italic_θ , italic_c end_POSTSUBSCRIPT start_POSTSUPERSCRIPT italic_n end_POSTSUPERSCRIPT ( italic_E ( italic_t ) ). The reverse inclusion holds analogously which gives the first result. The analogous result for U𝑈Uitalic_U follows by a judicious replacement of an equality by an inequality in the above. ∎

Lemma 4.3.

Let K>1𝐾1K>1italic_K > 1, θ(π/2,π/2]𝜃𝜋2𝜋2\theta\in(-\pi/2,\pi/2]italic_θ ∈ ( - italic_π / 2 , italic_π / 2 ] and c𝑐c\in\mathbb{C}italic_c ∈ blackboard_C. Then for any t>0𝑡0t>0italic_t > 0, we have E(t)=U(t)𝐸𝑡𝑈𝑡E(t)=\partial U(t)italic_E ( italic_t ) = ∂ italic_U ( italic_t ).

Proof.

By the continuity of GK,θ,csubscript𝐺𝐾𝜃𝑐G_{K,\theta,c}italic_G start_POSTSUBSCRIPT italic_K , italic_θ , italic_c end_POSTSUBSCRIPT, it is evident that U(t)E(t)𝑈𝑡𝐸𝑡\partial U(t)\subset E(t)∂ italic_U ( italic_t ) ⊂ italic_E ( italic_t ). On the other hand, if zE(t)U(t)𝑧𝐸𝑡𝑈𝑡z\in E(t)\setminus\partial U(t)italic_z ∈ italic_E ( italic_t ) ∖ ∂ italic_U ( italic_t ), then z𝑧zitalic_z would be a local maximum for GK,θ,csubscript𝐺𝐾𝜃𝑐G_{K,\theta,c}italic_G start_POSTSUBSCRIPT italic_K , italic_θ , italic_c end_POSTSUBSCRIPT. By Theorem 1.1, HK,θ,cn(z)subscriptsuperscript𝐻𝑛𝐾𝜃𝑐𝑧H^{n}_{K,\theta,c}(z)italic_H start_POSTSUPERSCRIPT italic_n end_POSTSUPERSCRIPT start_POSTSUBSCRIPT italic_K , italic_θ , italic_c end_POSTSUBSCRIPT ( italic_z ) would also be a local maximum of GK,θ,csubscript𝐺𝐾𝜃𝑐G_{K,\theta,c}italic_G start_POSTSUBSCRIPT italic_K , italic_θ , italic_c end_POSTSUBSCRIPT for all n𝑛nitalic_n. However, for large |z|𝑧|z|| italic_z |, we have GK,θ,c=(logτK,θ,0)φK,θ,csubscript𝐺𝐾𝜃𝑐subscript𝜏𝐾𝜃0subscript𝜑𝐾𝜃𝑐G_{K,\theta,c}=(\log\tau_{K,\theta,0})\circ\varphi_{K,\theta,c}italic_G start_POSTSUBSCRIPT italic_K , italic_θ , italic_c end_POSTSUBSCRIPT = ( roman_log italic_τ start_POSTSUBSCRIPT italic_K , italic_θ , 0 end_POSTSUBSCRIPT ) ∘ italic_φ start_POSTSUBSCRIPT italic_K , italic_θ , italic_c end_POSTSUBSCRIPT. As φK,θ,csubscript𝜑𝐾𝜃𝑐\varphi_{K,\theta,c}italic_φ start_POSTSUBSCRIPT italic_K , italic_θ , italic_c end_POSTSUBSCRIPT is quasiconformal and τK,θ,0subscript𝜏𝐾𝜃0\tau_{K,\theta,0}italic_τ start_POSTSUBSCRIPT italic_K , italic_θ , 0 end_POSTSUBSCRIPT has no local maxima, this is a contradiction. The result follows. ∎

Lemma 4.4.

Let K>1𝐾1K>1italic_K > 1, θ(π/2,π/2]𝜃𝜋2𝜋2\theta\in(-\pi/2,\pi/2]italic_θ ∈ ( - italic_π / 2 , italic_π / 2 ] and c𝑐c\in\mathbb{C}italic_c ∈ blackboard_C.

  1. (a)

    There exists T>0𝑇0T>0italic_T > 0 such that if t>T𝑡𝑇t>Titalic_t > italic_T then E(t)𝐸𝑡E(t)italic_E ( italic_t ) is a simple closed curve.

  2. (b)

    For any t>0𝑡0t>0italic_t > 0, E(t)𝐸𝑡E(t)italic_E ( italic_t ) is a finite collection of closed curves and U(t)𝑈𝑡U(t)italic_U ( italic_t ) is an open neighborhood of infinity.

  3. (c)

    Suppose that 0<s<t0𝑠𝑡0<s<t0 < italic_s < italic_t. Then U(t)¯¯𝑈𝑡\overline{U(t)}over¯ start_ARG italic_U ( italic_t ) end_ARG is contained in U(s)𝑈𝑠U(s)italic_U ( italic_s ). Moreover, every component of E(s)𝐸𝑠E(s)italic_E ( italic_s ) is contained in a bounded component of E(t)𝐸𝑡\mathbb{C}\setminus E(t)blackboard_C ∖ italic_E ( italic_t ).

Proof.

For part (a), suppose first that t>0𝑡0t>0italic_t > 0 is large enough that GK,θ,c1(t)=τK,θ,c1(et)WK,θ,csuperscriptsubscript𝐺𝐾𝜃𝑐1𝑡superscriptsubscript𝜏𝐾𝜃𝑐1superscript𝑒𝑡subscript𝑊𝐾𝜃𝑐G_{K,\theta,c}^{-1}(t)=\tau_{K,\theta,c}^{-1}(e^{t})\subset W_{K,\theta,c}italic_G start_POSTSUBSCRIPT italic_K , italic_θ , italic_c end_POSTSUBSCRIPT start_POSTSUPERSCRIPT - 1 end_POSTSUPERSCRIPT ( italic_t ) = italic_τ start_POSTSUBSCRIPT italic_K , italic_θ , italic_c end_POSTSUBSCRIPT start_POSTSUPERSCRIPT - 1 end_POSTSUPERSCRIPT ( italic_e start_POSTSUPERSCRIPT italic_t end_POSTSUPERSCRIPT ) ⊂ italic_W start_POSTSUBSCRIPT italic_K , italic_θ , italic_c end_POSTSUBSCRIPT, recalling that this latter set is where the Böttcher coordinate φK,θ,csubscript𝜑𝐾𝜃𝑐\varphi_{K,\theta,c}italic_φ start_POSTSUBSCRIPT italic_K , italic_θ , italic_c end_POSTSUBSCRIPT is defined. As τK,θ,c=τK,θ,0φK,θ,csubscript𝜏𝐾𝜃𝑐subscript𝜏𝐾𝜃0subscript𝜑𝐾𝜃𝑐\tau_{K,\theta,c}=\tau_{K,\theta,0}\circ\varphi_{K,\theta,c}italic_τ start_POSTSUBSCRIPT italic_K , italic_θ , italic_c end_POSTSUBSCRIPT = italic_τ start_POSTSUBSCRIPT italic_K , italic_θ , 0 end_POSTSUBSCRIPT ∘ italic_φ start_POSTSUBSCRIPT italic_K , italic_θ , italic_c end_POSTSUBSCRIPT, we see that

GK,θ,c1(t)=φK,θ,c1(τK,θ,01(et)).superscriptsubscript𝐺𝐾𝜃𝑐1𝑡superscriptsubscript𝜑𝐾𝜃𝑐1superscriptsubscript𝜏𝐾𝜃01superscript𝑒𝑡G_{K,\theta,c}^{-1}(t)=\varphi_{K,\theta,c}^{-1}(\tau_{K,\theta,0}^{-1}(e^{t})).italic_G start_POSTSUBSCRIPT italic_K , italic_θ , italic_c end_POSTSUBSCRIPT start_POSTSUPERSCRIPT - 1 end_POSTSUPERSCRIPT ( italic_t ) = italic_φ start_POSTSUBSCRIPT italic_K , italic_θ , italic_c end_POSTSUBSCRIPT start_POSTSUPERSCRIPT - 1 end_POSTSUPERSCRIPT ( italic_τ start_POSTSUBSCRIPT italic_K , italic_θ , 0 end_POSTSUBSCRIPT start_POSTSUPERSCRIPT - 1 end_POSTSUPERSCRIPT ( italic_e start_POSTSUPERSCRIPT italic_t end_POSTSUPERSCRIPT ) ) .

As τK,θ,01(et)superscriptsubscript𝜏𝐾𝜃01superscript𝑒𝑡\tau_{K,\theta,0}^{-1}(e^{t})italic_τ start_POSTSUBSCRIPT italic_K , italic_θ , 0 end_POSTSUBSCRIPT start_POSTSUPERSCRIPT - 1 end_POSTSUPERSCRIPT ( italic_e start_POSTSUPERSCRIPT italic_t end_POSTSUPERSCRIPT ) is a simple closed curve, and as φK,θ,csubscript𝜑𝐾𝜃𝑐\varphi_{K,\theta,c}italic_φ start_POSTSUBSCRIPT italic_K , italic_θ , italic_c end_POSTSUBSCRIPT is quasiconformal, we see that E(t)𝐸𝑡E(t)italic_E ( italic_t ) is a simple closed curve for large enough t𝑡titalic_t.

For part (b), suppose that t>0𝑡0t>0italic_t > 0 is arbitrary and suppose n𝑛n\in\mathbb{N}italic_n ∈ blackboard_N is chosen large enough that part(a) applies to show E(2nt)𝐸superscript2𝑛𝑡E(2^{n}t)italic_E ( 2 start_POSTSUPERSCRIPT italic_n end_POSTSUPERSCRIPT italic_t ) is a simple closed curve. By Lemma 4.2, as HK,θ,cnsuperscriptsubscript𝐻𝐾𝜃𝑐𝑛H_{K,\theta,c}^{n}italic_H start_POSTSUBSCRIPT italic_K , italic_θ , italic_c end_POSTSUBSCRIPT start_POSTSUPERSCRIPT italic_n end_POSTSUPERSCRIPT is a finite degree quasiregular map, then the inverse image of E(2nt)𝐸superscript2𝑛𝑡E(2^{n}t)italic_E ( 2 start_POSTSUPERSCRIPT italic_n end_POSTSUPERSCRIPT italic_t ) under HK,θ,cnsuperscriptsubscript𝐻𝐾𝜃𝑐𝑛H_{K,\theta,c}^{n}italic_H start_POSTSUBSCRIPT italic_K , italic_θ , italic_c end_POSTSUBSCRIPT start_POSTSUPERSCRIPT italic_n end_POSTSUPERSCRIPT is a finite collection of closed curves.

As GK,θ,csubscript𝐺𝐾𝜃𝑐G_{K,\theta,c}italic_G start_POSTSUBSCRIPT italic_K , italic_θ , italic_c end_POSTSUBSCRIPT is continuous, it follows that U(t)𝑈𝑡U(t)italic_U ( italic_t ) is open and connected for all t>0𝑡0t>0italic_t > 0. Moreover, by Lemma 4.3, U(t)𝑈𝑡U(t)italic_U ( italic_t ) is the unbounded component of the complement of E(t)𝐸𝑡E(t)italic_E ( italic_t ) and hence is a neighborhood of infinity.

For part (c), if zU(t)𝑧𝑈𝑡z\in U(t)italic_z ∈ italic_U ( italic_t ), then GK,θ,c(z)>t>ssubscript𝐺𝐾𝜃𝑐𝑧𝑡𝑠G_{K,\theta,c}(z)>t>sitalic_G start_POSTSUBSCRIPT italic_K , italic_θ , italic_c end_POSTSUBSCRIPT ( italic_z ) > italic_t > italic_s, so zU(s)𝑧𝑈𝑠z\in U(s)italic_z ∈ italic_U ( italic_s ). Suppose that zU(t)𝑧𝑈𝑡z\in\partial U(t)italic_z ∈ ∂ italic_U ( italic_t ). Then GK,θ,c(z)tsubscript𝐺𝐾𝜃𝑐𝑧𝑡G_{K,\theta,c}(z)\leq titalic_G start_POSTSUBSCRIPT italic_K , italic_θ , italic_c end_POSTSUBSCRIPT ( italic_z ) ≤ italic_t. Any open neighborhood of z𝑧zitalic_z contains an element w𝑤witalic_w such that GK,θ,c(w)>tsubscript𝐺𝐾𝜃𝑐𝑤𝑡G_{K,\theta,c}(w)>titalic_G start_POSTSUBSCRIPT italic_K , italic_θ , italic_c end_POSTSUBSCRIPT ( italic_w ) > italic_t, so GK,θ,c(z)subscript𝐺𝐾𝜃𝑐𝑧G_{K,\theta,c}(z)italic_G start_POSTSUBSCRIPT italic_K , italic_θ , italic_c end_POSTSUBSCRIPT ( italic_z ) must equal t𝑡titalic_t since GK,θ,csubscript𝐺𝐾𝜃𝑐G_{K,\theta,c}italic_G start_POSTSUBSCRIPT italic_K , italic_θ , italic_c end_POSTSUBSCRIPT is continuous. It follows that U(t)U(s)𝑈𝑡𝑈𝑠\partial U(t)\subset U(s)∂ italic_U ( italic_t ) ⊂ italic_U ( italic_s ). We conclude that U(t)¯U(s)¯𝑈𝑡𝑈𝑠\overline{U(t)}\subset U(s)over¯ start_ARG italic_U ( italic_t ) end_ARG ⊂ italic_U ( italic_s ). Moreover, by Lemma 4.3, we see that E(t)U(s)𝐸𝑡𝑈𝑠E(t)\subset U(s)italic_E ( italic_t ) ⊂ italic_U ( italic_s ) and the final claim follows. ∎

Observe that each component of E(t)𝐸𝑡E(t)italic_E ( italic_t ) need not be a simple closed curve. For example, if cK,θ𝑐subscript𝐾𝜃c\notin\mathcal{M}_{K,\theta}italic_c ∉ caligraphic_M start_POSTSUBSCRIPT italic_K , italic_θ end_POSTSUBSCRIPT then there exists t0>0subscript𝑡00t_{0}>0italic_t start_POSTSUBSCRIPT 0 end_POSTSUBSCRIPT > 0 such that E(t0)𝐸subscript𝑡0E(t_{0})italic_E ( italic_t start_POSTSUBSCRIPT 0 end_POSTSUBSCRIPT ) contains the critical point 00 of HK,θ,csubscript𝐻𝐾𝜃𝑐H_{K,\theta,c}italic_H start_POSTSUBSCRIPT italic_K , italic_θ , italic_c end_POSTSUBSCRIPT. Then E(t0)𝐸subscript𝑡0E(t_{0})italic_E ( italic_t start_POSTSUBSCRIPT 0 end_POSTSUBSCRIPT ) will be a topological figure eight. The image of E(t0)𝐸subscript𝑡0E(t_{0})italic_E ( italic_t start_POSTSUBSCRIPT 0 end_POSTSUBSCRIPT ) under HK,θ,csubscript𝐻𝐾𝜃𝑐H_{K,\theta,c}italic_H start_POSTSUBSCRIPT italic_K , italic_θ , italic_c end_POSTSUBSCRIPT will be a simple closed curve. We make these observations more precise in the following results.

Lemma 4.5.

Let K>1𝐾1K>1italic_K > 1 and θ(π/2,π/2]𝜃𝜋2𝜋2\theta\in(-\pi/2,\pi/2]italic_θ ∈ ( - italic_π / 2 , italic_π / 2 ]. If cK,θ𝑐subscript𝐾𝜃c\in\mathcal{M}_{K,\theta}italic_c ∈ caligraphic_M start_POSTSUBSCRIPT italic_K , italic_θ end_POSTSUBSCRIPT, then for any t>0𝑡0t>0italic_t > 0, UK,θ,c(t){}subscript𝑈𝐾𝜃𝑐𝑡U_{K,\theta,c}(t)\cup\{\infty\}italic_U start_POSTSUBSCRIPT italic_K , italic_θ , italic_c end_POSTSUBSCRIPT ( italic_t ) ∪ { ∞ } is a simply connected subdomain of subscript\mathbb{C}_{\infty}blackboard_C start_POSTSUBSCRIPT ∞ end_POSTSUBSCRIPT and EK,θ,c(t)subscript𝐸𝐾𝜃𝑐𝑡E_{K,\theta,c}(t)italic_E start_POSTSUBSCRIPT italic_K , italic_θ , italic_c end_POSTSUBSCRIPT ( italic_t ) has one component.

Proof.

Fix cK,θ𝑐subscript𝐾𝜃c\in\mathcal{M}_{K,\theta}italic_c ∈ caligraphic_M start_POSTSUBSCRIPT italic_K , italic_θ end_POSTSUBSCRIPT. For t>0𝑡0t>0italic_t > 0, set U(t)=UK,θ,c(t)𝑈𝑡subscript𝑈𝐾𝜃𝑐𝑡U(t)=U_{K,\theta,c}(t)italic_U ( italic_t ) = italic_U start_POSTSUBSCRIPT italic_K , italic_θ , italic_c end_POSTSUBSCRIPT ( italic_t ). By Lemma 4.4 (a) and (b), there exists T>0𝑇0T>0italic_T > 0 such that if t>T𝑡𝑇t>Titalic_t > italic_T, then U(t)𝑈𝑡U(t)italic_U ( italic_t ) has the claimed properties.

Suppose now that t>0𝑡0t>0italic_t > 0 is arbitrary and find n𝑛n\in\mathbb{N}italic_n ∈ blackboard_N such that 2nt>Tsuperscript2𝑛𝑡𝑇2^{n}t>T2 start_POSTSUPERSCRIPT italic_n end_POSTSUPERSCRIPT italic_t > italic_T. We will apply the Riemann-Hurwitz formula, Theorem 2.6, with f=HK,θ,cn𝑓superscriptsubscript𝐻𝐾𝜃𝑐𝑛f=H_{K,\theta,c}^{n}italic_f = italic_H start_POSTSUBSCRIPT italic_K , italic_θ , italic_c end_POSTSUBSCRIPT start_POSTSUPERSCRIPT italic_n end_POSTSUPERSCRIPT, D1=U(t){}subscript𝐷1𝑈𝑡D_{1}=U(t)\cup\{\infty\}italic_D start_POSTSUBSCRIPT 1 end_POSTSUBSCRIPT = italic_U ( italic_t ) ∪ { ∞ } and D2=U(2nt){}subscript𝐷2𝑈superscript2𝑛𝑡D_{2}=U(2^{n}t)\cup\{\infty\}italic_D start_POSTSUBSCRIPT 2 end_POSTSUBSCRIPT = italic_U ( 2 start_POSTSUPERSCRIPT italic_n end_POSTSUPERSCRIPT italic_t ) ∪ { ∞ }. We observe that HK,θ,cnsuperscriptsubscript𝐻𝐾𝜃𝑐𝑛H_{K,\theta,c}^{n}italic_H start_POSTSUBSCRIPT italic_K , italic_θ , italic_c end_POSTSUBSCRIPT start_POSTSUPERSCRIPT italic_n end_POSTSUPERSCRIPT is a proper map, as if XD2𝑋subscript𝐷2X\subset D_{2}italic_X ⊂ italic_D start_POSTSUBSCRIPT 2 end_POSTSUBSCRIPT is compact, then XU(s)𝑋𝑈𝑠X\subset U(s)italic_X ⊂ italic_U ( italic_s ) for some s>2nt𝑠superscript2𝑛𝑡s>2^{n}titalic_s > 2 start_POSTSUPERSCRIPT italic_n end_POSTSUPERSCRIPT italic_t and so HK,θ,cn(X)U(2ns)subscriptsuperscript𝐻𝑛𝐾𝜃𝑐𝑋𝑈superscript2𝑛𝑠H^{-n}_{K,\theta,c}(X)\subset U(2^{-n}s)italic_H start_POSTSUPERSCRIPT - italic_n end_POSTSUPERSCRIPT start_POSTSUBSCRIPT italic_K , italic_θ , italic_c end_POSTSUBSCRIPT ( italic_X ) ⊂ italic_U ( 2 start_POSTSUPERSCRIPT - italic_n end_POSTSUPERSCRIPT italic_s ) which is compactly contained in D1subscript𝐷1D_{1}italic_D start_POSTSUBSCRIPT 1 end_POSTSUBSCRIPT.

The degree of HK,θ,cnsuperscriptsubscript𝐻𝐾𝜃𝑐𝑛H_{K,\theta,c}^{n}italic_H start_POSTSUBSCRIPT italic_K , italic_θ , italic_c end_POSTSUBSCRIPT start_POSTSUPERSCRIPT italic_n end_POSTSUPERSCRIPT is 2nsuperscript2𝑛2^{n}2 start_POSTSUPERSCRIPT italic_n end_POSTSUPERSCRIPT. As cK,θ𝑐subscript𝐾𝜃c\in\mathcal{M}_{K,\theta}italic_c ∈ caligraphic_M start_POSTSUBSCRIPT italic_K , italic_θ end_POSTSUBSCRIPT, 0I(HK,θ,c)0𝐼subscript𝐻𝐾𝜃𝑐0\notin I(H_{K,\theta,c})0 ∉ italic_I ( italic_H start_POSTSUBSCRIPT italic_K , italic_θ , italic_c end_POSTSUBSCRIPT ) and so the number of branch points of HK,θ,cnsubscriptsuperscript𝐻𝑛𝐾𝜃𝑐H^{n}_{K,\theta,c}italic_H start_POSTSUPERSCRIPT italic_n end_POSTSUPERSCRIPT start_POSTSUBSCRIPT italic_K , italic_θ , italic_c end_POSTSUBSCRIPT in D1subscript𝐷1D_{1}italic_D start_POSTSUBSCRIPT 1 end_POSTSUBSCRIPT counting multiplicity just comes from the point at infinity, and is thus L=2n1𝐿superscript2𝑛1L=2^{n}-1italic_L = 2 start_POSTSUPERSCRIPT italic_n end_POSTSUPERSCRIPT - 1. As D2subscript𝐷2D_{2}italic_D start_POSTSUBSCRIPT 2 end_POSTSUBSCRIPT has only one boundary component, we have d2=1subscript𝑑21d_{2}=1italic_d start_POSTSUBSCRIPT 2 end_POSTSUBSCRIPT = 1. Solving

2d1=2n(21)2n+12subscript𝑑1superscript2𝑛21superscript2𝑛12-d_{1}=2^{n}(2-1)-2^{n}+12 - italic_d start_POSTSUBSCRIPT 1 end_POSTSUBSCRIPT = 2 start_POSTSUPERSCRIPT italic_n end_POSTSUPERSCRIPT ( 2 - 1 ) - 2 start_POSTSUPERSCRIPT italic_n end_POSTSUPERSCRIPT + 1

for d1subscript𝑑1d_{1}italic_d start_POSTSUBSCRIPT 1 end_POSTSUBSCRIPT yields d1=1subscript𝑑11d_{1}=1italic_d start_POSTSUBSCRIPT 1 end_POSTSUBSCRIPT = 1. We conclude that U(t)𝑈𝑡U(t)italic_U ( italic_t ) has one boundary component and hence U(t){}𝑈𝑡U(t)\cup\{\infty\}italic_U ( italic_t ) ∪ { ∞ } is simply connected. ∎

Lemma 4.6.

Let K>1𝐾1K>1italic_K > 1 and θ(π/2,π/2]𝜃𝜋2𝜋2\theta\in(-\pi/2,\pi/2]italic_θ ∈ ( - italic_π / 2 , italic_π / 2 ]. If cK,θ𝑐subscript𝐾𝜃c\notin\mathcal{M}_{K,\theta}italic_c ∉ caligraphic_M start_POSTSUBSCRIPT italic_K , italic_θ end_POSTSUBSCRIPT, set t0=GK,θ,c(0)>0subscript𝑡0subscript𝐺𝐾𝜃𝑐00t_{0}=G_{K,\theta,c}(0)>0italic_t start_POSTSUBSCRIPT 0 end_POSTSUBSCRIPT = italic_G start_POSTSUBSCRIPT italic_K , italic_θ , italic_c end_POSTSUBSCRIPT ( 0 ) > 0. If tt0𝑡subscript𝑡0t\geq t_{0}italic_t ≥ italic_t start_POSTSUBSCRIPT 0 end_POSTSUBSCRIPT then UK,θ,c(t){}subscript𝑈𝐾𝜃𝑐𝑡U_{K,\theta,c}(t)\cup\{\infty\}italic_U start_POSTSUBSCRIPT italic_K , italic_θ , italic_c end_POSTSUBSCRIPT ( italic_t ) ∪ { ∞ } is a simply connected subdomain of subscript\mathbb{C}_{\infty}blackboard_C start_POSTSUBSCRIPT ∞ end_POSTSUBSCRIPT and EK,θ,c(t)subscript𝐸𝐾𝜃𝑐𝑡E_{K,\theta,c}(t)italic_E start_POSTSUBSCRIPT italic_K , italic_θ , italic_c end_POSTSUBSCRIPT ( italic_t ) has one component. If t<t0𝑡subscript𝑡0t<t_{0}italic_t < italic_t start_POSTSUBSCRIPT 0 end_POSTSUBSCRIPT, let

(4.1) m=ln(t0/t)ln2.𝑚subscript𝑡0𝑡2m=\left\lceil\frac{\ln(t_{0}/t)}{\ln 2}\right\rceil\in\mathbb{N}.italic_m = ⌈ divide start_ARG roman_ln ( italic_t start_POSTSUBSCRIPT 0 end_POSTSUBSCRIPT / italic_t ) end_ARG start_ARG roman_ln 2 end_ARG ⌉ ∈ blackboard_N .

Then EK,θ,c(t)subscript𝐸𝐾𝜃𝑐𝑡E_{K,\theta,c}(t)italic_E start_POSTSUBSCRIPT italic_K , italic_θ , italic_c end_POSTSUBSCRIPT ( italic_t ) has 2msuperscript2𝑚2^{m}2 start_POSTSUPERSCRIPT italic_m end_POSTSUPERSCRIPT components and UK,θ,c(t)subscript𝑈𝐾𝜃𝑐𝑡U_{K,\theta,c}(t)italic_U start_POSTSUBSCRIPT italic_K , italic_θ , italic_c end_POSTSUBSCRIPT ( italic_t ) is 2msuperscript2𝑚2^{m}2 start_POSTSUPERSCRIPT italic_m end_POSTSUPERSCRIPT-connected.

Proof.

Fix cK,θ𝑐subscript𝐾𝜃c\notin\mathcal{M}_{K,\theta}italic_c ∉ caligraphic_M start_POSTSUBSCRIPT italic_K , italic_θ end_POSTSUBSCRIPT and for t>0𝑡0t>0italic_t > 0 set U(t)=UK,θ,c(t)𝑈𝑡subscript𝑈𝐾𝜃𝑐𝑡U(t)=U_{K,\theta,c}(t)italic_U ( italic_t ) = italic_U start_POSTSUBSCRIPT italic_K , italic_θ , italic_c end_POSTSUBSCRIPT ( italic_t ). The proof of Lemma 4.5 shows that as long as the critical point 00 does not lie in U(t)𝑈𝑡U(t)italic_U ( italic_t ), then U(t){}𝑈𝑡U(t)\cup\{\infty\}italic_U ( italic_t ) ∪ { ∞ } is simply connected. With t0=GK,θ,c(0)subscript𝑡0subscript𝐺𝐾𝜃𝑐0t_{0}=G_{K,\theta,c}(0)italic_t start_POSTSUBSCRIPT 0 end_POSTSUBSCRIPT = italic_G start_POSTSUBSCRIPT italic_K , italic_θ , italic_c end_POSTSUBSCRIPT ( 0 ), E(t0)𝐸subscript𝑡0E(t_{0})italic_E ( italic_t start_POSTSUBSCRIPT 0 end_POSTSUBSCRIPT ) is a figure-eight shape, but U(t0){}𝑈subscript𝑡0U(t_{0})\cup\{\infty\}italic_U ( italic_t start_POSTSUBSCRIPT 0 end_POSTSUBSCRIPT ) ∪ { ∞ } is still simply connected.

Now suppose that t<t0𝑡subscript𝑡0t<t_{0}italic_t < italic_t start_POSTSUBSCRIPT 0 end_POSTSUBSCRIPT and m𝑚mitalic_m is the smallest integer such that 2mtt0superscript2𝑚𝑡subscript𝑡02^{m}t\geq t_{0}2 start_POSTSUPERSCRIPT italic_m end_POSTSUPERSCRIPT italic_t ≥ italic_t start_POSTSUBSCRIPT 0 end_POSTSUBSCRIPT. Then m𝑚mitalic_m is given by (4.1). We will apply the Riemann-Hurwitz formula to the proper map HK,θ,cmsubscriptsuperscript𝐻𝑚𝐾𝜃𝑐H^{m}_{K,\theta,c}italic_H start_POSTSUPERSCRIPT italic_m end_POSTSUPERSCRIPT start_POSTSUBSCRIPT italic_K , italic_θ , italic_c end_POSTSUBSCRIPT with D1=U(t){}subscript𝐷1𝑈𝑡D_{1}=U(t)\cup\{\infty\}italic_D start_POSTSUBSCRIPT 1 end_POSTSUBSCRIPT = italic_U ( italic_t ) ∪ { ∞ } and D2=U(2mt){}subscript𝐷2𝑈superscript2𝑚𝑡D_{2}=U(2^{m}t)\cup\{\infty\}italic_D start_POSTSUBSCRIPT 2 end_POSTSUBSCRIPT = italic_U ( 2 start_POSTSUPERSCRIPT italic_m end_POSTSUPERSCRIPT italic_t ) ∪ { ∞ }. The critical points of HK,θ,cmsubscriptsuperscript𝐻𝑚𝐾𝜃𝑐H^{m}_{K,\theta,c}italic_H start_POSTSUPERSCRIPT italic_m end_POSTSUPERSCRIPT start_POSTSUBSCRIPT italic_K , italic_θ , italic_c end_POSTSUBSCRIPT contained in D1subscript𝐷1D_{1}italic_D start_POSTSUBSCRIPT 1 end_POSTSUBSCRIPT are at infinity, with multiplicity 2m1superscript2𝑚12^{m}-12 start_POSTSUPERSCRIPT italic_m end_POSTSUPERSCRIPT - 1, and in the set {HK,θ,cj(0):j=0,,m1}conditional-setsuperscriptsubscript𝐻𝐾𝜃𝑐𝑗0𝑗0𝑚1\{H_{K,\theta,c}^{-j}(0):j=0,\ldots,m-1\}{ italic_H start_POSTSUBSCRIPT italic_K , italic_θ , italic_c end_POSTSUBSCRIPT start_POSTSUPERSCRIPT - italic_j end_POSTSUPERSCRIPT ( 0 ) : italic_j = 0 , … , italic_m - 1 }, each with multiplicity 1111. As HK,θ,cj(0)superscriptsubscript𝐻𝐾𝜃𝑐𝑗0H_{K,\theta,c}^{-j}(0)italic_H start_POSTSUBSCRIPT italic_K , italic_θ , italic_c end_POSTSUBSCRIPT start_POSTSUPERSCRIPT - italic_j end_POSTSUPERSCRIPT ( 0 ) contains 2jsuperscript2𝑗2^{j}2 start_POSTSUPERSCRIPT italic_j end_POSTSUPERSCRIPT elements, we have

L=(2m1)+(1++2m1)=2(2m1).𝐿superscript2𝑚11superscript2𝑚12superscript2𝑚1L=(2^{m}-1)+(1+\ldots+2^{m-1})=2(2^{m}-1).italic_L = ( 2 start_POSTSUPERSCRIPT italic_m end_POSTSUPERSCRIPT - 1 ) + ( 1 + … + 2 start_POSTSUPERSCRIPT italic_m - 1 end_POSTSUPERSCRIPT ) = 2 ( 2 start_POSTSUPERSCRIPT italic_m end_POSTSUPERSCRIPT - 1 ) .

The number of boundary components of D2subscript𝐷2D_{2}italic_D start_POSTSUBSCRIPT 2 end_POSTSUBSCRIPT is d2=1subscript𝑑21d_{2}=1italic_d start_POSTSUBSCRIPT 2 end_POSTSUBSCRIPT = 1 and so the Riemann-Hurwitz formula gives

2d1=2m(21)2(2m1),2subscript𝑑1superscript2𝑚212superscript2𝑚12-d_{1}=2^{m}(2-1)-2(2^{m}-1),2 - italic_d start_POSTSUBSCRIPT 1 end_POSTSUBSCRIPT = 2 start_POSTSUPERSCRIPT italic_m end_POSTSUPERSCRIPT ( 2 - 1 ) - 2 ( 2 start_POSTSUPERSCRIPT italic_m end_POSTSUPERSCRIPT - 1 ) ,

which implies d1=2msubscript𝑑1superscript2𝑚d_{1}=2^{m}italic_d start_POSTSUBSCRIPT 1 end_POSTSUBSCRIPT = 2 start_POSTSUPERSCRIPT italic_m end_POSTSUPERSCRIPT as required. ∎

We are now in a position to prove that the bounded orbit set contains either one, or uncountably many, components.

Proof of Theorem 1.2.

By Lemma 4.4 (c), we can reformulate the bounded orbit set as

(4.2) BO(HK,θ,c)=t>0(UK,θ,c(t)).𝐵𝑂subscript𝐻𝐾𝜃𝑐subscript𝑡0subscript𝑈𝐾𝜃𝑐𝑡BO(H_{K,\theta,c})=\bigcap_{t>0}\left(\mathbb{C}\setminus U_{K,\theta,c}(t)% \right).italic_B italic_O ( italic_H start_POSTSUBSCRIPT italic_K , italic_θ , italic_c end_POSTSUBSCRIPT ) = ⋂ start_POSTSUBSCRIPT italic_t > 0 end_POSTSUBSCRIPT ( blackboard_C ∖ italic_U start_POSTSUBSCRIPT italic_K , italic_θ , italic_c end_POSTSUBSCRIPT ( italic_t ) ) .

If cK,θ𝑐subscript𝐾𝜃c\in\mathcal{M}_{K,\theta}italic_c ∈ caligraphic_M start_POSTSUBSCRIPT italic_K , italic_θ end_POSTSUBSCRIPT, then Lemma 4.5 implies that BO(HK,θ,c)𝐵𝑂subscript𝐻𝐾𝜃𝑐BO(H_{K,\theta,c})italic_B italic_O ( italic_H start_POSTSUBSCRIPT italic_K , italic_θ , italic_c end_POSTSUBSCRIPT ) contains one component.

On the other hand, suppose that cK,θ𝑐subscript𝐾𝜃c\notin\mathcal{M}_{K,\theta}italic_c ∉ caligraphic_M start_POSTSUBSCRIPT italic_K , italic_θ end_POSTSUBSCRIPT. Set t0=GK,θ,c(0)>0subscript𝑡0subscript𝐺𝐾𝜃𝑐00t_{0}=G_{K,\theta,c}(0)>0italic_t start_POSTSUBSCRIPT 0 end_POSTSUBSCRIPT = italic_G start_POSTSUBSCRIPT italic_K , italic_θ , italic_c end_POSTSUBSCRIPT ( 0 ) > 0 and choose t1(t0,2t0)subscript𝑡1subscript𝑡02subscript𝑡0t_{1}\in(t_{0},2t_{0})italic_t start_POSTSUBSCRIPT 1 end_POSTSUBSCRIPT ∈ ( italic_t start_POSTSUBSCRIPT 0 end_POSTSUBSCRIPT , 2 italic_t start_POSTSUBSCRIPT 0 end_POSTSUBSCRIPT ). Then by the proof of Lemma 4.6, E(t1)𝐸subscript𝑡1E(t_{1})italic_E ( italic_t start_POSTSUBSCRIPT 1 end_POSTSUBSCRIPT ) is a simple closed curve and E(t1/2)𝐸subscript𝑡12E(t_{1}/2)italic_E ( italic_t start_POSTSUBSCRIPT 1 end_POSTSUBSCRIPT / 2 ) consists of two simple closed curves. Denote by X0subscript𝑋0X_{0}italic_X start_POSTSUBSCRIPT 0 end_POSTSUBSCRIPT the bounded component of the complement of E(t1)𝐸subscript𝑡1E(t_{1})italic_E ( italic_t start_POSTSUBSCRIPT 1 end_POSTSUBSCRIPT ) and by Yjsubscript𝑌𝑗Y_{j}italic_Y start_POSTSUBSCRIPT italic_j end_POSTSUBSCRIPT, for j=1,2𝑗12j=1,2italic_j = 1 , 2, the two bounded components of the complement of E(t1/2)𝐸subscript𝑡12E(t_{1}/2)italic_E ( italic_t start_POSTSUBSCRIPT 1 end_POSTSUBSCRIPT / 2 ). Then HK,θ,csubscript𝐻𝐾𝜃𝑐H_{K,\theta,c}italic_H start_POSTSUBSCRIPT italic_K , italic_θ , italic_c end_POSTSUBSCRIPT restricted to each Yjsubscript𝑌𝑗Y_{j}italic_Y start_POSTSUBSCRIPT italic_j end_POSTSUBSCRIPT is a homeomorphism onto X0subscript𝑋0X_{0}italic_X start_POSTSUBSCRIPT 0 end_POSTSUBSCRIPT and we may therefore define two branches of the inverse, say

gj:X0Yj:subscript𝑔𝑗subscript𝑋0subscript𝑌𝑗g_{j}:X_{0}\to Y_{j}italic_g start_POSTSUBSCRIPT italic_j end_POSTSUBSCRIPT : italic_X start_POSTSUBSCRIPT 0 end_POSTSUBSCRIPT → italic_Y start_POSTSUBSCRIPT italic_j end_POSTSUBSCRIPT

for j=1,2𝑗12j=1,2italic_j = 1 , 2. Evidently, Yj¯X0¯subscript𝑌𝑗subscript𝑋0\overline{Y_{j}}\subset X_{0}over¯ start_ARG italic_Y start_POSTSUBSCRIPT italic_j end_POSTSUBSCRIPT end_ARG ⊂ italic_X start_POSTSUBSCRIPT 0 end_POSTSUBSCRIPT for j=1,2𝑗12j=1,2italic_j = 1 , 2.

Here it is convenient to introduce symbolic notation. Given an integer k0𝑘0k\geq 0italic_k ≥ 0, we denote by {g1,g2}ksuperscriptsubscript𝑔1subscript𝑔2𝑘\{g_{1},g_{2}\}^{k}{ italic_g start_POSTSUBSCRIPT 1 end_POSTSUBSCRIPT , italic_g start_POSTSUBSCRIPT 2 end_POSTSUBSCRIPT } start_POSTSUPERSCRIPT italic_k end_POSTSUPERSCRIPT the set of words formed from the alphabet {g1,g2}subscript𝑔1subscript𝑔2\{g_{1},g_{2}\}{ italic_g start_POSTSUBSCRIPT 1 end_POSTSUBSCRIPT , italic_g start_POSTSUBSCRIPT 2 end_POSTSUBSCRIPT } that have length exactly k𝑘kitalic_k. Conventionally, we set {g1,g2}0={ε}superscriptsubscript𝑔1subscript𝑔20𝜀\{g_{1},g_{2}\}^{0}=\{\varepsilon\}{ italic_g start_POSTSUBSCRIPT 1 end_POSTSUBSCRIPT , italic_g start_POSTSUBSCRIPT 2 end_POSTSUBSCRIPT } start_POSTSUPERSCRIPT 0 end_POSTSUPERSCRIPT = { italic_ε } where ε𝜀\varepsilonitalic_ε is the empty word. We also denote by {g1,g2}=k0{g1,g2}ksuperscriptsubscript𝑔1subscript𝑔2subscript𝑘0superscriptsubscript𝑔1subscript𝑔2𝑘\{g_{1},g_{2}\}^{*}=\bigcup_{k\geq 0}\{g_{1},g_{2}\}^{k}{ italic_g start_POSTSUBSCRIPT 1 end_POSTSUBSCRIPT , italic_g start_POSTSUBSCRIPT 2 end_POSTSUBSCRIPT } start_POSTSUPERSCRIPT ∗ end_POSTSUPERSCRIPT = ⋃ start_POSTSUBSCRIPT italic_k ≥ 0 end_POSTSUBSCRIPT { italic_g start_POSTSUBSCRIPT 1 end_POSTSUBSCRIPT , italic_g start_POSTSUBSCRIPT 2 end_POSTSUBSCRIPT } start_POSTSUPERSCRIPT italic_k end_POSTSUPERSCRIPT the set of all finite words formed from {g1,g2}subscript𝑔1subscript𝑔2\{g_{1},g_{2}\}{ italic_g start_POSTSUBSCRIPT 1 end_POSTSUBSCRIPT , italic_g start_POSTSUBSCRIPT 2 end_POSTSUBSCRIPT }. Given a word w{g1,g2}𝑤superscriptsubscript𝑔1subscript𝑔2w\in\{g_{1},g_{2}\}^{*}italic_w ∈ { italic_g start_POSTSUBSCRIPT 1 end_POSTSUBSCRIPT , italic_g start_POSTSUBSCRIPT 2 end_POSTSUBSCRIPT } start_POSTSUPERSCRIPT ∗ end_POSTSUPERSCRIPT, we denote by |w|𝑤|w|| italic_w | the length of w𝑤witalic_w with the convention |ε|=0𝜀0|\varepsilon|=0| italic_ε | = 0.

With this convention, for w{g1,g2}𝑤superscriptsubscript𝑔1subscript𝑔2w\in\{g_{1},g_{2}\}^{*}italic_w ∈ { italic_g start_POSTSUBSCRIPT 1 end_POSTSUBSCRIPT , italic_g start_POSTSUBSCRIPT 2 end_POSTSUBSCRIPT } start_POSTSUPERSCRIPT ∗ end_POSTSUPERSCRIPT, we can define

Xw=w(X0),subscript𝑋𝑤𝑤subscript𝑋0X_{w}=w(X_{0}),italic_X start_POSTSUBSCRIPT italic_w end_POSTSUBSCRIPT = italic_w ( italic_X start_POSTSUBSCRIPT 0 end_POSTSUBSCRIPT ) ,

so that, for example, Xg1g2=g1(g2(X0))subscript𝑋subscript𝑔1subscript𝑔2subscript𝑔1subscript𝑔2subscript𝑋0X_{g_{1}g_{2}}=g_{1}(g_{2}(X_{0}))italic_X start_POSTSUBSCRIPT italic_g start_POSTSUBSCRIPT 1 end_POSTSUBSCRIPT italic_g start_POSTSUBSCRIPT 2 end_POSTSUBSCRIPT end_POSTSUBSCRIPT = italic_g start_POSTSUBSCRIPT 1 end_POSTSUBSCRIPT ( italic_g start_POSTSUBSCRIPT 2 end_POSTSUBSCRIPT ( italic_X start_POSTSUBSCRIPT 0 end_POSTSUBSCRIPT ) ). By Lemma 4.4 (c), we can rewrite (4.2) as

BO(HK,θ,c)=w{g1,g2}Xw.𝐵𝑂subscript𝐻𝐾𝜃𝑐subscript𝑤superscriptsubscript𝑔1subscript𝑔2subscript𝑋𝑤BO(H_{K,\theta,c})=\bigcap_{w\in\{g_{1},g_{2}\}^{*}}X_{w}.italic_B italic_O ( italic_H start_POSTSUBSCRIPT italic_K , italic_θ , italic_c end_POSTSUBSCRIPT ) = ⋂ start_POSTSUBSCRIPT italic_w ∈ { italic_g start_POSTSUBSCRIPT 1 end_POSTSUBSCRIPT , italic_g start_POSTSUBSCRIPT 2 end_POSTSUBSCRIPT } start_POSTSUPERSCRIPT ∗ end_POSTSUPERSCRIPT end_POSTSUBSCRIPT italic_X start_POSTSUBSCRIPT italic_w end_POSTSUBSCRIPT .

Every component of BO(HK,θ,c)𝐵𝑂subscript𝐻𝐾𝜃𝑐BO(H_{K,\theta,c})italic_B italic_O ( italic_H start_POSTSUBSCRIPT italic_K , italic_θ , italic_c end_POSTSUBSCRIPT ) can be identified with an infinite word from the alphabet {g1,g2}subscript𝑔1subscript𝑔2\{g_{1},g_{2}\}{ italic_g start_POSTSUBSCRIPT 1 end_POSTSUBSCRIPT , italic_g start_POSTSUBSCRIPT 2 end_POSTSUBSCRIPT }, and by a standard diagonal argument, the number of such components is uncountable. ∎

5. Fixed and periodic points

In this section, we will focus on the case where cK,θ𝑐subscript𝐾𝜃c\notin\mathcal{M}_{K,\theta}italic_c ∉ caligraphic_M start_POSTSUBSCRIPT italic_K , italic_θ end_POSTSUBSCRIPT and so BO(HK,θ,c)𝐵𝑂subscript𝐻𝐾𝜃𝑐BO(H_{K,\theta,c})italic_B italic_O ( italic_H start_POSTSUBSCRIPT italic_K , italic_θ , italic_c end_POSTSUBSCRIPT ) has uncountably many components.

Proof of Theorem 1.3.

Fix K>1𝐾1K>1italic_K > 1, θ(π/2,π/2]𝜃𝜋2𝜋2\theta\in(-\pi/2,\pi/2]italic_θ ∈ ( - italic_π / 2 , italic_π / 2 ], cK,θ𝑐subscript𝐾𝜃c\notin\mathcal{M}_{K,\theta}italic_c ∉ caligraphic_M start_POSTSUBSCRIPT italic_K , italic_θ end_POSTSUBSCRIPT and n𝑛n\in\mathbb{N}italic_n ∈ blackboard_N such that HK,θ,csubscript𝐻𝐾𝜃𝑐H_{K,\theta,c}italic_H start_POSTSUBSCRIPT italic_K , italic_θ , italic_c end_POSTSUBSCRIPT has at least 2n+1superscript2𝑛12^{n}+12 start_POSTSUPERSCRIPT italic_n end_POSTSUPERSCRIPT + 1 periodic points of period n𝑛nitalic_n. Then HK,θ,cnsubscriptsuperscript𝐻𝑛𝐾𝜃𝑐H^{n}_{K,\theta,c}italic_H start_POSTSUPERSCRIPT italic_n end_POSTSUPERSCRIPT start_POSTSUBSCRIPT italic_K , italic_θ , italic_c end_POSTSUBSCRIPT has at least 2n+1superscript2𝑛12^{n}+12 start_POSTSUPERSCRIPT italic_n end_POSTSUPERSCRIPT + 1 fixed points. Using the symbolic notation established in the proof of Theorem 1.2, and Lemma 4.6, the components of HK,θ,cn(X0)subscriptsuperscript𝐻𝑛𝐾𝜃𝑐subscript𝑋0H^{-n}_{K,\theta,c}(X_{0})italic_H start_POSTSUPERSCRIPT - italic_n end_POSTSUPERSCRIPT start_POSTSUBSCRIPT italic_K , italic_θ , italic_c end_POSTSUBSCRIPT ( italic_X start_POSTSUBSCRIPT 0 end_POSTSUBSCRIPT ) can be enumerated by w{g1,g2}n𝑤superscriptsubscript𝑔1subscript𝑔2𝑛w\in\{g_{1},g_{2}\}^{n}italic_w ∈ { italic_g start_POSTSUBSCRIPT 1 end_POSTSUBSCRIPT , italic_g start_POSTSUBSCRIPT 2 end_POSTSUBSCRIPT } start_POSTSUPERSCRIPT italic_n end_POSTSUPERSCRIPT. There are therefore 2nsuperscript2𝑛2^{n}2 start_POSTSUPERSCRIPT italic_n end_POSTSUPERSCRIPT components.

By the pigeonhole principle, at least two of the fixed points of HK,θ,cnsuperscriptsubscript𝐻𝐾𝜃𝑐𝑛H_{K,\theta,c}^{n}italic_H start_POSTSUBSCRIPT italic_K , italic_θ , italic_c end_POSTSUBSCRIPT start_POSTSUPERSCRIPT italic_n end_POSTSUPERSCRIPT, say z1z2subscript𝑧1subscript𝑧2z_{1}\neq z_{2}italic_z start_POSTSUBSCRIPT 1 end_POSTSUBSCRIPT ≠ italic_z start_POSTSUBSCRIPT 2 end_POSTSUBSCRIPT, must be in the same component, say A=w(X0)𝐴𝑤subscript𝑋0A=w(X_{0})italic_A = italic_w ( italic_X start_POSTSUBSCRIPT 0 end_POSTSUBSCRIPT ). Consider now w(A)𝑤𝐴w(A)italic_w ( italic_A ). As w𝑤witalic_w represents a homeomorphism, w(A)𝑤𝐴w(A)italic_w ( italic_A ) is connected and contains z1subscript𝑧1z_{1}italic_z start_POSTSUBSCRIPT 1 end_POSTSUBSCRIPT and z2subscript𝑧2z_{2}italic_z start_POSTSUBSCRIPT 2 end_POSTSUBSCRIPT. Repeating this argument, we see that

{z1,z2}k1wk(X0),subscript𝑧1subscript𝑧2subscript𝑘1superscript𝑤𝑘subscript𝑋0\{z_{1},z_{2}\}\subset\bigcap_{k\geq 1}w^{k}(X_{0}),{ italic_z start_POSTSUBSCRIPT 1 end_POSTSUBSCRIPT , italic_z start_POSTSUBSCRIPT 2 end_POSTSUBSCRIPT } ⊂ ⋂ start_POSTSUBSCRIPT italic_k ≥ 1 end_POSTSUBSCRIPT italic_w start_POSTSUPERSCRIPT italic_k end_POSTSUPERSCRIPT ( italic_X start_POSTSUBSCRIPT 0 end_POSTSUBSCRIPT ) ,

which means that z1,z2subscript𝑧1subscript𝑧2z_{1},z_{2}italic_z start_POSTSUBSCRIPT 1 end_POSTSUBSCRIPT , italic_z start_POSTSUBSCRIPT 2 end_POSTSUBSCRIPT are contained in the same component of BO(HK,θ,c)𝐵𝑂subscript𝐻𝐾𝜃𝑐BO(H_{K,\theta,c})italic_B italic_O ( italic_H start_POSTSUBSCRIPT italic_K , italic_θ , italic_c end_POSTSUBSCRIPT ). We conclude that the bounded orbit set is not totally disconnected. ∎

We will now show that it is possible for the hypothesis of Theorem 1.3 to hold. Here, we will set θ=0𝜃0\theta=0italic_θ = 0 and c𝑐c\in\mathbb{R}italic_c ∈ blackboard_R so that the mapping we consider is

HK,0,c(x+iy)=(K2x2y2+c)+i(2Kxy).subscript𝐻𝐾0𝑐𝑥𝑖𝑦superscript𝐾2superscript𝑥2superscript𝑦2𝑐𝑖2𝐾𝑥𝑦H_{K,0,c}(x+iy)=(K^{2}x^{2}-y^{2}+c)+i(2Kxy).italic_H start_POSTSUBSCRIPT italic_K , 0 , italic_c end_POSTSUBSCRIPT ( italic_x + italic_i italic_y ) = ( italic_K start_POSTSUPERSCRIPT 2 end_POSTSUPERSCRIPT italic_x start_POSTSUPERSCRIPT 2 end_POSTSUPERSCRIPT - italic_y start_POSTSUPERSCRIPT 2 end_POSTSUPERSCRIPT + italic_c ) + italic_i ( 2 italic_K italic_x italic_y ) .

Solving for fixed points, by equating real and imaginary parts we obtain the two equations

K2x2y2+c=x, and 2Kxy=y.formulae-sequencesuperscript𝐾2superscript𝑥2superscript𝑦2𝑐𝑥 and 2𝐾𝑥𝑦𝑦K^{2}x^{2}-y^{2}+c=x,\quad\text{ and }2Kxy=y.italic_K start_POSTSUPERSCRIPT 2 end_POSTSUPERSCRIPT italic_x start_POSTSUPERSCRIPT 2 end_POSTSUPERSCRIPT - italic_y start_POSTSUPERSCRIPT 2 end_POSTSUPERSCRIPT + italic_c = italic_x , and 2 italic_K italic_x italic_y = italic_y .

The second of these clearly has solutions y=0𝑦0y=0italic_y = 0 or x=(2K)1𝑥superscript2𝐾1x=(2K)^{-1}italic_x = ( 2 italic_K ) start_POSTSUPERSCRIPT - 1 end_POSTSUPERSCRIPT. In the first case when y=0𝑦0y=0italic_y = 0, the first equation yields

K2x2x+c=0x=1±14K2c2K2.superscript𝐾2superscript𝑥2𝑥𝑐0𝑥plus-or-minus114superscript𝐾2𝑐2superscript𝐾2K^{2}x^{2}-x+c=0\implies x=\frac{1\pm\sqrt{1-4K^{2}c}}{2K^{2}}.italic_K start_POSTSUPERSCRIPT 2 end_POSTSUPERSCRIPT italic_x start_POSTSUPERSCRIPT 2 end_POSTSUPERSCRIPT - italic_x + italic_c = 0 ⟹ italic_x = divide start_ARG 1 ± square-root start_ARG 1 - 4 italic_K start_POSTSUPERSCRIPT 2 end_POSTSUPERSCRIPT italic_c end_ARG end_ARG start_ARG 2 italic_K start_POSTSUPERSCRIPT 2 end_POSTSUPERSCRIPT end_ARG .

This has real solutions if and only if c(4K2)1𝑐superscript4superscript𝐾21c\leq(4K^{2})^{-1}italic_c ≤ ( 4 italic_K start_POSTSUPERSCRIPT 2 end_POSTSUPERSCRIPT ) start_POSTSUPERSCRIPT - 1 end_POSTSUPERSCRIPT. On the other hand, if x=(2K)1𝑥superscript2𝐾1x=(2K)^{-1}italic_x = ( 2 italic_K ) start_POSTSUPERSCRIPT - 1 end_POSTSUPERSCRIPT, then the first equation yields

14y212K+c=0y=±1412K+c.14superscript𝑦212𝐾𝑐0𝑦plus-or-minus1412𝐾𝑐\frac{1}{4}-y^{2}-\frac{1}{2K}+c=0\implies y=\pm\sqrt{\frac{1}{4}-\frac{1}{2K}% +c}.divide start_ARG 1 end_ARG start_ARG 4 end_ARG - italic_y start_POSTSUPERSCRIPT 2 end_POSTSUPERSCRIPT - divide start_ARG 1 end_ARG start_ARG 2 italic_K end_ARG + italic_c = 0 ⟹ italic_y = ± square-root start_ARG divide start_ARG 1 end_ARG start_ARG 4 end_ARG - divide start_ARG 1 end_ARG start_ARG 2 italic_K end_ARG + italic_c end_ARG .

This has real solutions if and only if c(2K)11/4𝑐superscript2𝐾114c\geq(2K)^{-1}-1/4italic_c ≥ ( 2 italic_K ) start_POSTSUPERSCRIPT - 1 end_POSTSUPERSCRIPT - 1 / 4. It is worth pointing out that we always have

12K1414K212𝐾1414superscript𝐾2\frac{1}{2K}-\frac{1}{4}\leq\frac{1}{4K^{2}}divide start_ARG 1 end_ARG start_ARG 2 italic_K end_ARG - divide start_ARG 1 end_ARG start_ARG 4 end_ARG ≤ divide start_ARG 1 end_ARG start_ARG 4 italic_K start_POSTSUPERSCRIPT 2 end_POSTSUPERSCRIPT end_ARG

as rearranging this yields (K1)20superscript𝐾120(K-1)^{2}\geq 0( italic_K - 1 ) start_POSTSUPERSCRIPT 2 end_POSTSUPERSCRIPT ≥ 0.

By analyzing the four cases that can occur here, we see that HK,0,csubscript𝐻𝐾0𝑐H_{K,0,c}italic_H start_POSTSUBSCRIPT italic_K , 0 , italic_c end_POSTSUBSCRIPT has four fixed points if c𝑐citalic_c lies in the interval

(12K14,14K2),12𝐾1414superscript𝐾2\left(\frac{1}{2K}-\frac{1}{4},\frac{1}{4K^{2}}\right),( divide start_ARG 1 end_ARG start_ARG 2 italic_K end_ARG - divide start_ARG 1 end_ARG start_ARG 4 end_ARG , divide start_ARG 1 end_ARG start_ARG 4 italic_K start_POSTSUPERSCRIPT 2 end_POSTSUPERSCRIPT end_ARG ) ,

three fixed points if c𝑐citalic_c is at either endpoint of this interval, and two fixed points otherwise.

Next, we show that there are parameters c𝑐citalic_c for which HK,0,csubscript𝐻𝐾0𝑐H_{K,0,c}italic_H start_POSTSUBSCRIPT italic_K , 0 , italic_c end_POSTSUBSCRIPT has four fixed points and for which cK,0𝑐subscript𝐾0c\notin\mathcal{M}_{K,0}italic_c ∉ caligraphic_M start_POSTSUBSCRIPT italic_K , 0 end_POSTSUBSCRIPT. To this end, recall from [11, Theorem 6.4] that

K,0=[2K2,14K2].subscript𝐾02superscript𝐾214superscript𝐾2\mathcal{M}_{K,0}\cap\mathbb{R}=\left[-\frac{2}{K^{2}},\frac{1}{4K^{2}}\right].caligraphic_M start_POSTSUBSCRIPT italic_K , 0 end_POSTSUBSCRIPT ∩ blackboard_R = [ - divide start_ARG 2 end_ARG start_ARG italic_K start_POSTSUPERSCRIPT 2 end_POSTSUPERSCRIPT end_ARG , divide start_ARG 1 end_ARG start_ARG 4 italic_K start_POSTSUPERSCRIPT 2 end_POSTSUPERSCRIPT end_ARG ] .

The right hand endpoint of this interval agrees with the endpoint of the interval above, and so we need to analyze the left hand endpoints. We want

12K14<2K2,12𝐾142superscript𝐾2\frac{1}{2K}-\frac{1}{4}<-\frac{2}{K^{2}},divide start_ARG 1 end_ARG start_ARG 2 italic_K end_ARG - divide start_ARG 1 end_ARG start_ARG 4 end_ARG < - divide start_ARG 2 end_ARG start_ARG italic_K start_POSTSUPERSCRIPT 2 end_POSTSUPERSCRIPT end_ARG ,

which after some elementary algebra simplifies to K>4𝐾4K>4italic_K > 4, recalling that we assume K>1𝐾1K>1italic_K > 1. We conclude that if this is so, then for

c(12K14,2K2)𝑐12𝐾142superscript𝐾2c\in\left(\frac{1}{2K}-\frac{1}{4},-\frac{2}{K^{2}}\right)italic_c ∈ ( divide start_ARG 1 end_ARG start_ARG 2 italic_K end_ARG - divide start_ARG 1 end_ARG start_ARG 4 end_ARG , - divide start_ARG 2 end_ARG start_ARG italic_K start_POSTSUPERSCRIPT 2 end_POSTSUPERSCRIPT end_ARG )

the map HK,0,csubscript𝐻𝐾0𝑐H_{K,0,c}italic_H start_POSTSUBSCRIPT italic_K , 0 , italic_c end_POSTSUBSCRIPT has four fixed points, and hence by Theorem 1.3, BO(HK,0,c)𝐵𝑂subscript𝐻𝐾0𝑐BO(H_{K,0,c})italic_B italic_O ( italic_H start_POSTSUBSCRIPT italic_K , 0 , italic_c end_POSTSUBSCRIPT ) has uncountably many components, but is not totally disconnected.

6. Classification of fixed points

It is clear that if z0subscript𝑧0z_{0}italic_z start_POSTSUBSCRIPT 0 end_POSTSUBSCRIPT is a fixed point of Pc(z)=z2+csubscript𝑃𝑐𝑧superscript𝑧2𝑐P_{c}(z)=z^{2}+citalic_P start_POSTSUBSCRIPT italic_c end_POSTSUBSCRIPT ( italic_z ) = italic_z start_POSTSUPERSCRIPT 2 end_POSTSUPERSCRIPT + italic_c, then it is attracting, neutral or repelling according to whether |z0|subscript𝑧0|z_{0}|| italic_z start_POSTSUBSCRIPT 0 end_POSTSUBSCRIPT | is less than, equal to, or greater than 1212\tfrac{1}{2}divide start_ARG 1 end_ARG start_ARG 2 end_ARG respectively. Here we establish the analogous result for HK,0,csubscript𝐻𝐾0𝑐H_{K,0,c}italic_H start_POSTSUBSCRIPT italic_K , 0 , italic_c end_POSTSUBSCRIPT. We focus on the case where θ=0𝜃0\theta=0italic_θ = 0 for the sake of clarity of exposition, but the generalization to arbitrary θ𝜃\thetaitalic_θ may be computed similarly. Now we fix K>1𝐾1K>1italic_K > 1 and c𝑐c\in\mathbb{C}italic_c ∈ blackboard_C, although the role of c𝑐citalic_c in what follows is unimportant, and set H=HK,0,c𝐻subscript𝐻𝐾0𝑐H=H_{K,0,c}italic_H = italic_H start_POSTSUBSCRIPT italic_K , 0 , italic_c end_POSTSUBSCRIPT.

Recall that

H(x+iy)=(Kx+iy)2+c=K2x2y2+Re(c)+[2Kxy+Im(c)]i𝐻𝑥𝑖𝑦superscript𝐾𝑥𝑖𝑦2𝑐superscript𝐾2superscript𝑥2superscript𝑦2Re𝑐delimited-[]2𝐾𝑥𝑦Im𝑐𝑖H(x+iy)=(Kx+iy)^{2}+c=K^{2}x^{2}-y^{2}+\operatorname{Re}(c)+[2Kxy+% \operatorname{Im}(c)]iitalic_H ( italic_x + italic_i italic_y ) = ( italic_K italic_x + italic_i italic_y ) start_POSTSUPERSCRIPT 2 end_POSTSUPERSCRIPT + italic_c = italic_K start_POSTSUPERSCRIPT 2 end_POSTSUPERSCRIPT italic_x start_POSTSUPERSCRIPT 2 end_POSTSUPERSCRIPT - italic_y start_POSTSUPERSCRIPT 2 end_POSTSUPERSCRIPT + roman_Re ( italic_c ) + [ 2 italic_K italic_x italic_y + roman_Im ( italic_c ) ] italic_i

and so the derivative of H𝐻Hitalic_H is [2K2x2y2Ky2Kx].matrix2superscript𝐾2𝑥2𝑦2𝐾𝑦2𝐾𝑥\displaystyle\begin{bmatrix}2K^{2}x&-2y\\ 2Ky&2Kx\end{bmatrix}.[ start_ARG start_ROW start_CELL 2 italic_K start_POSTSUPERSCRIPT 2 end_POSTSUPERSCRIPT italic_x end_CELL start_CELL - 2 italic_y end_CELL end_ROW start_ROW start_CELL 2 italic_K italic_y end_CELL start_CELL 2 italic_K italic_x end_CELL end_ROW end_ARG ] .

The eigenvalues of this matrix are solutions of the quadratic

λ2+(2K2x2Kx)λ+4K3x2+4Ky2=0,superscript𝜆22superscript𝐾2𝑥2𝐾𝑥𝜆4superscript𝐾3superscript𝑥24𝐾superscript𝑦20\lambda^{2}+(-2K^{2}x-2Kx)\lambda+4K^{3}x^{2}+4Ky^{2}=0,italic_λ start_POSTSUPERSCRIPT 2 end_POSTSUPERSCRIPT + ( - 2 italic_K start_POSTSUPERSCRIPT 2 end_POSTSUPERSCRIPT italic_x - 2 italic_K italic_x ) italic_λ + 4 italic_K start_POSTSUPERSCRIPT 3 end_POSTSUPERSCRIPT italic_x start_POSTSUPERSCRIPT 2 end_POSTSUPERSCRIPT + 4 italic_K italic_y start_POSTSUPERSCRIPT 2 end_POSTSUPERSCRIPT = 0 ,

which are

(6.1) λ=K2x+Kx±[(K2x+Kx)24(K3x2+Ky2)]1/2.𝜆plus-or-minussuperscript𝐾2𝑥𝐾𝑥superscriptdelimited-[]superscriptsuperscript𝐾2𝑥𝐾𝑥24superscript𝐾3superscript𝑥2𝐾superscript𝑦212\lambda=K^{2}x+Kx\pm[(K^{2}x+Kx)^{2}-4(K^{3}x^{2}+Ky^{2})]^{1/2}.italic_λ = italic_K start_POSTSUPERSCRIPT 2 end_POSTSUPERSCRIPT italic_x + italic_K italic_x ± [ ( italic_K start_POSTSUPERSCRIPT 2 end_POSTSUPERSCRIPT italic_x + italic_K italic_x ) start_POSTSUPERSCRIPT 2 end_POSTSUPERSCRIPT - 4 ( italic_K start_POSTSUPERSCRIPT 3 end_POSTSUPERSCRIPT italic_x start_POSTSUPERSCRIPT 2 end_POSTSUPERSCRIPT + italic_K italic_y start_POSTSUPERSCRIPT 2 end_POSTSUPERSCRIPT ) ] start_POSTSUPERSCRIPT 1 / 2 end_POSTSUPERSCRIPT .

We let λ1subscript𝜆1\lambda_{1}italic_λ start_POSTSUBSCRIPT 1 end_POSTSUBSCRIPT and λ2subscript𝜆2\lambda_{2}italic_λ start_POSTSUBSCRIPT 2 end_POSTSUBSCRIPT denote the two eigenvalues with the convention that |λ1||λ2|subscript𝜆1subscript𝜆2|\lambda_{1}|\leq|\lambda_{2}|| italic_λ start_POSTSUBSCRIPT 1 end_POSTSUBSCRIPT | ≤ | italic_λ start_POSTSUBSCRIPT 2 end_POSTSUBSCRIPT |. We aim to partition the regions of \mathbb{C}blackboard_C according to the absolute values of |λ1|subscript𝜆1|\lambda_{1}|| italic_λ start_POSTSUBSCRIPT 1 end_POSTSUBSCRIPT | and |λ2|subscript𝜆2|\lambda_{2}|| italic_λ start_POSTSUBSCRIPT 2 end_POSTSUBSCRIPT | in order to classify the type of fixed point at a given point (x0,y0)subscript𝑥0subscript𝑦0(x_{0},y_{0})( italic_x start_POSTSUBSCRIPT 0 end_POSTSUBSCRIPT , italic_y start_POSTSUBSCRIPT 0 end_POSTSUBSCRIPT ). We point out that if the eigenvalues at x+iy𝑥𝑖𝑦x+iyitalic_x + italic_i italic_y are given by λ1,λ2subscript𝜆1subscript𝜆2\lambda_{1},\lambda_{2}italic_λ start_POSTSUBSCRIPT 1 end_POSTSUBSCRIPT , italic_λ start_POSTSUBSCRIPT 2 end_POSTSUBSCRIPT, and if s𝑠s\in\mathbb{R}italic_s ∈ blackboard_R, then the eigenvalues at (sx)+i(sy)𝑠𝑥𝑖𝑠𝑦(sx)+i(sy)( italic_s italic_x ) + italic_i ( italic_s italic_y ) are given by

(6.2) sλ1,sλ2.𝑠subscript𝜆1𝑠subscript𝜆2s\lambda_{1},s\lambda_{2}.italic_s italic_λ start_POSTSUBSCRIPT 1 end_POSTSUBSCRIPT , italic_s italic_λ start_POSTSUBSCRIPT 2 end_POSTSUBSCRIPT .

First, we find where the eigenvalues are real or non-real.

Lemma 6.1.

Let SK+superscriptsubscript𝑆𝐾S_{K}^{+}italic_S start_POSTSUBSCRIPT italic_K end_POSTSUBSCRIPT start_POSTSUPERSCRIPT + end_POSTSUPERSCRIPT be the sector given by

y>K(K1)x2 and y>K(K1)x2,𝑦𝐾𝐾1𝑥2 and 𝑦𝐾𝐾1𝑥2y>\frac{\sqrt{K}(K-1)x}{2}\text{ and }y>-\frac{\sqrt{K}(K-1)x}{2},italic_y > divide start_ARG square-root start_ARG italic_K end_ARG ( italic_K - 1 ) italic_x end_ARG start_ARG 2 end_ARG and italic_y > - divide start_ARG square-root start_ARG italic_K end_ARG ( italic_K - 1 ) italic_x end_ARG start_ARG 2 end_ARG ,

and let SKsuperscriptsubscript𝑆𝐾S_{K}^{-}italic_S start_POSTSUBSCRIPT italic_K end_POSTSUBSCRIPT start_POSTSUPERSCRIPT - end_POSTSUPERSCRIPT be the sector given by

y<K(K1)x2 and y<K(K1)x2.𝑦𝐾𝐾1𝑥2 and 𝑦𝐾𝐾1𝑥2y<\frac{\sqrt{K}(K-1)x}{2}\text{ and }y<-\frac{\sqrt{K}(K-1)x}{2}.italic_y < divide start_ARG square-root start_ARG italic_K end_ARG ( italic_K - 1 ) italic_x end_ARG start_ARG 2 end_ARG and italic_y < - divide start_ARG square-root start_ARG italic_K end_ARG ( italic_K - 1 ) italic_x end_ARG start_ARG 2 end_ARG .

If x+iy𝑥𝑖𝑦x+iyitalic_x + italic_i italic_y lies in either SK±superscriptsubscript𝑆𝐾plus-or-minusS_{K}^{\pm}italic_S start_POSTSUBSCRIPT italic_K end_POSTSUBSCRIPT start_POSTSUPERSCRIPT ± end_POSTSUPERSCRIPT, then λ1subscript𝜆1\lambda_{1}italic_λ start_POSTSUBSCRIPT 1 end_POSTSUBSCRIPT and λ2subscript𝜆2\lambda_{2}italic_λ start_POSTSUBSCRIPT 2 end_POSTSUBSCRIPT are non-real complex conjugates. Otherwise λ1subscript𝜆1\lambda_{1}italic_λ start_POSTSUBSCRIPT 1 end_POSTSUBSCRIPT and λ2subscript𝜆2\lambda_{2}italic_λ start_POSTSUBSCRIPT 2 end_POSTSUBSCRIPT are real. Moreover, if x+iy𝑥𝑖𝑦x+iyitalic_x + italic_i italic_y lies on one of the two lines LK±superscriptsubscript𝐿𝐾plus-or-minusL_{K}^{\pm}italic_L start_POSTSUBSCRIPT italic_K end_POSTSUBSCRIPT start_POSTSUPERSCRIPT ± end_POSTSUPERSCRIPT given by

y=±K(K1)x2,𝑦plus-or-minus𝐾𝐾1𝑥2y=\pm\frac{\sqrt{K}(K-1)x}{2},italic_y = ± divide start_ARG square-root start_ARG italic_K end_ARG ( italic_K - 1 ) italic_x end_ARG start_ARG 2 end_ARG ,

then the two eigenvalues are the same (and real).

Proof.

It follows from (6.1) that λ1subscript𝜆1\lambda_{1}italic_λ start_POSTSUBSCRIPT 1 end_POSTSUBSCRIPT and λ2subscript𝜆2\lambda_{2}italic_λ start_POSTSUBSCRIPT 2 end_POSTSUBSCRIPT are non-real complex conjugates if and only if

(K2x+Kx)24(K3x2+Ky2)<0.superscriptsuperscript𝐾2𝑥𝐾𝑥24superscript𝐾3superscript𝑥2𝐾superscript𝑦20(K^{2}x+Kx)^{2}-4(K^{3}x^{2}+Ky^{2})<0.( italic_K start_POSTSUPERSCRIPT 2 end_POSTSUPERSCRIPT italic_x + italic_K italic_x ) start_POSTSUPERSCRIPT 2 end_POSTSUPERSCRIPT - 4 ( italic_K start_POSTSUPERSCRIPT 3 end_POSTSUPERSCRIPT italic_x start_POSTSUPERSCRIPT 2 end_POSTSUPERSCRIPT + italic_K italic_y start_POSTSUPERSCRIPT 2 end_POSTSUPERSCRIPT ) < 0 .

We can compute that

(K2x+Kx)24(K3x2+Ky2)superscriptsuperscript𝐾2𝑥𝐾𝑥24superscript𝐾3superscript𝑥2𝐾superscript𝑦2\displaystyle(K^{2}x+Kx)^{2}-4(K^{3}x^{2}+Ky^{2})( italic_K start_POSTSUPERSCRIPT 2 end_POSTSUPERSCRIPT italic_x + italic_K italic_x ) start_POSTSUPERSCRIPT 2 end_POSTSUPERSCRIPT - 4 ( italic_K start_POSTSUPERSCRIPT 3 end_POSTSUPERSCRIPT italic_x start_POSTSUPERSCRIPT 2 end_POSTSUPERSCRIPT + italic_K italic_y start_POSTSUPERSCRIPT 2 end_POSTSUPERSCRIPT ) =K4x22K3x2+K2x24Ky2absentsuperscript𝐾4superscript𝑥22superscript𝐾3superscript𝑥2superscript𝐾2superscript𝑥24𝐾superscript𝑦2\displaystyle=K^{4}x^{2}-2K^{3}x^{2}+K^{2}x^{2}-4Ky^{2}= italic_K start_POSTSUPERSCRIPT 4 end_POSTSUPERSCRIPT italic_x start_POSTSUPERSCRIPT 2 end_POSTSUPERSCRIPT - 2 italic_K start_POSTSUPERSCRIPT 3 end_POSTSUPERSCRIPT italic_x start_POSTSUPERSCRIPT 2 end_POSTSUPERSCRIPT + italic_K start_POSTSUPERSCRIPT 2 end_POSTSUPERSCRIPT italic_x start_POSTSUPERSCRIPT 2 end_POSTSUPERSCRIPT - 4 italic_K italic_y start_POSTSUPERSCRIPT 2 end_POSTSUPERSCRIPT
=K2x2(K1)24Ky2absentsuperscript𝐾2superscript𝑥2superscript𝐾124𝐾superscript𝑦2\displaystyle=K^{2}x^{2}(K-1)^{2}-4Ky^{2}= italic_K start_POSTSUPERSCRIPT 2 end_POSTSUPERSCRIPT italic_x start_POSTSUPERSCRIPT 2 end_POSTSUPERSCRIPT ( italic_K - 1 ) start_POSTSUPERSCRIPT 2 end_POSTSUPERSCRIPT - 4 italic_K italic_y start_POSTSUPERSCRIPT 2 end_POSTSUPERSCRIPT
=(K(K1)x2Ky)(K(K1)x+2Ky).absent𝐾𝐾1𝑥2𝐾𝑦𝐾𝐾1𝑥2𝐾𝑦\displaystyle=(K(K-1)x-2\sqrt{K}y)(K(K-1)x+2\sqrt{K}y).= ( italic_K ( italic_K - 1 ) italic_x - 2 square-root start_ARG italic_K end_ARG italic_y ) ( italic_K ( italic_K - 1 ) italic_x + 2 square-root start_ARG italic_K end_ARG italic_y ) .

The first part of the lemma follows by assigning one of these factors to be strictly positive and one to be strictly negative. The second part follows immediately from (6.1). ∎

In particular, it follows from Lemma 6.1 that on the imaginary axis, away from 00, the eigenvalues are always non-real. When the eigenvalues are complex conjugates, we can compute their absolute values as follows.

Lemma 6.2.

Suppose that λ1subscript𝜆1\lambda_{1}italic_λ start_POSTSUBSCRIPT 1 end_POSTSUBSCRIPT and λ2subscript𝜆2\lambda_{2}italic_λ start_POSTSUBSCRIPT 2 end_POSTSUBSCRIPT are complex conjugates. Then |λ1|=|λ2|=1subscript𝜆1subscript𝜆21|\lambda_{1}|=|\lambda_{2}|=1| italic_λ start_POSTSUBSCRIPT 1 end_POSTSUBSCRIPT | = | italic_λ start_POSTSUBSCRIPT 2 end_POSTSUBSCRIPT | = 1 when x+iy𝑥𝑖𝑦x+iyitalic_x + italic_i italic_y lies on the curve γKsubscript𝛾𝐾\gamma_{K}italic_γ start_POSTSUBSCRIPT italic_K end_POSTSUBSCRIPT given by

4K3x2+4Ky2=1.4superscript𝐾3superscript𝑥24𝐾superscript𝑦214K^{3}x^{2}+4Ky^{2}=1.4 italic_K start_POSTSUPERSCRIPT 3 end_POSTSUPERSCRIPT italic_x start_POSTSUPERSCRIPT 2 end_POSTSUPERSCRIPT + 4 italic_K italic_y start_POSTSUPERSCRIPT 2 end_POSTSUPERSCRIPT = 1 .

Clearly γKsubscript𝛾𝐾\gamma_{K}italic_γ start_POSTSUBSCRIPT italic_K end_POSTSUBSCRIPT is the boundary curve of an ellipse. Note that via (6.2), it follows that |λ1|=|λ2|=rsubscript𝜆1subscript𝜆2𝑟|\lambda_{1}|=|\lambda_{2}|=r| italic_λ start_POSTSUBSCRIPT 1 end_POSTSUBSCRIPT | = | italic_λ start_POSTSUBSCRIPT 2 end_POSTSUBSCRIPT | = italic_r when

4K3x2+4Ky2=r2.4superscript𝐾3superscript𝑥24𝐾superscript𝑦2superscript𝑟24K^{3}x^{2}+4Ky^{2}=r^{2}.4 italic_K start_POSTSUPERSCRIPT 3 end_POSTSUPERSCRIPT italic_x start_POSTSUPERSCRIPT 2 end_POSTSUPERSCRIPT + 4 italic_K italic_y start_POSTSUPERSCRIPT 2 end_POSTSUPERSCRIPT = italic_r start_POSTSUPERSCRIPT 2 end_POSTSUPERSCRIPT .
Proof.

As |λ1|=|λ2|=1subscript𝜆1subscript𝜆21|\lambda_{1}|=|\lambda_{2}|=1| italic_λ start_POSTSUBSCRIPT 1 end_POSTSUBSCRIPT | = | italic_λ start_POSTSUBSCRIPT 2 end_POSTSUBSCRIPT | = 1 and λ1subscript𝜆1\lambda_{1}italic_λ start_POSTSUBSCRIPT 1 end_POSTSUBSCRIPT is a complex conjugate of λ2subscript𝜆2\lambda_{2}italic_λ start_POSTSUBSCRIPT 2 end_POSTSUBSCRIPT, it follows that λ1λ2=1subscript𝜆1subscript𝜆21\lambda_{1}\lambda_{2}=1italic_λ start_POSTSUBSCRIPT 1 end_POSTSUBSCRIPT italic_λ start_POSTSUBSCRIPT 2 end_POSTSUBSCRIPT = 1. Therefore, we have

(K2x+Kx)2[(K2x+Kx)24(K3x2+Ky2)]=1.superscriptsuperscript𝐾2𝑥𝐾𝑥2delimited-[]superscriptsuperscript𝐾2𝑥𝐾𝑥24superscript𝐾3superscript𝑥2𝐾superscript𝑦21(K^{2}x+Kx)^{2}-[(K^{2}x+Kx)^{2}-4(K^{3}x^{2}+Ky^{2})]=1.( italic_K start_POSTSUPERSCRIPT 2 end_POSTSUPERSCRIPT italic_x + italic_K italic_x ) start_POSTSUPERSCRIPT 2 end_POSTSUPERSCRIPT - [ ( italic_K start_POSTSUPERSCRIPT 2 end_POSTSUPERSCRIPT italic_x + italic_K italic_x ) start_POSTSUPERSCRIPT 2 end_POSTSUPERSCRIPT - 4 ( italic_K start_POSTSUPERSCRIPT 3 end_POSTSUPERSCRIPT italic_x start_POSTSUPERSCRIPT 2 end_POSTSUPERSCRIPT + italic_K italic_y start_POSTSUPERSCRIPT 2 end_POSTSUPERSCRIPT ) ] = 1 .

Simplifying this expression completes the proof. ∎

We now turn to the case where the eigenvalues are real.

Lemma 6.3.

The set where one of the eigenvalues has values ±1plus-or-minus1\pm 1± 1 is given by the pair of ellipses EK±superscriptsubscript𝐸𝐾plus-or-minusE_{K}^{\pm}italic_E start_POSTSUBSCRIPT italic_K end_POSTSUBSCRIPT start_POSTSUPERSCRIPT ± end_POSTSUPERSCRIPT with centers

w±=±K+14K2,subscript𝑤plus-or-minusplus-or-minus𝐾14superscript𝐾2w_{\pm}=\pm\frac{K+1}{4K^{2}},italic_w start_POSTSUBSCRIPT ± end_POSTSUBSCRIPT = ± divide start_ARG italic_K + 1 end_ARG start_ARG 4 italic_K start_POSTSUPERSCRIPT 2 end_POSTSUPERSCRIPT end_ARG ,

major and minor semi-axes oriented vertically and horizontally respectively, with lengths

v=K14K and h=K14K2.𝑣𝐾14𝐾 and 𝐾14superscript𝐾2v=\frac{K-1}{4K}\text{ and }h=\frac{K-1}{4K^{2}}.italic_v = divide start_ARG italic_K - 1 end_ARG start_ARG 4 italic_K end_ARG and italic_h = divide start_ARG italic_K - 1 end_ARG start_ARG 4 italic_K start_POSTSUPERSCRIPT 2 end_POSTSUPERSCRIPT end_ARG .
Proof.

Suppose λ𝜆\lambdaitalic_λ is either ±1plus-or-minus1\pm 1± 1. Then it follows from (6.1) that

(λ(K2x+Kx))2=(K2x+Kx)24(K3x2+Ky2).superscript𝜆superscript𝐾2𝑥𝐾𝑥2superscriptsuperscript𝐾2𝑥𝐾𝑥24superscript𝐾3superscript𝑥2𝐾superscript𝑦2(\lambda-(K^{2}x+Kx))^{2}=(K^{2}x+Kx)^{2}-4(K^{3}x^{2}+Ky^{2}).( italic_λ - ( italic_K start_POSTSUPERSCRIPT 2 end_POSTSUPERSCRIPT italic_x + italic_K italic_x ) ) start_POSTSUPERSCRIPT 2 end_POSTSUPERSCRIPT = ( italic_K start_POSTSUPERSCRIPT 2 end_POSTSUPERSCRIPT italic_x + italic_K italic_x ) start_POSTSUPERSCRIPT 2 end_POSTSUPERSCRIPT - 4 ( italic_K start_POSTSUPERSCRIPT 3 end_POSTSUPERSCRIPT italic_x start_POSTSUPERSCRIPT 2 end_POSTSUPERSCRIPT + italic_K italic_y start_POSTSUPERSCRIPT 2 end_POSTSUPERSCRIPT ) .

By expanding out the brackets, we obtain

(K2x+Kx)22λ(K2x+Kx)+λ2=(K2x+Kx)24(K3x2+Ky2).superscriptsuperscript𝐾2𝑥𝐾𝑥22𝜆superscript𝐾2𝑥𝐾𝑥superscript𝜆2superscriptsuperscript𝐾2𝑥𝐾𝑥24superscript𝐾3superscript𝑥2𝐾superscript𝑦2(K^{2}x+Kx)^{2}-2\lambda(K^{2}x+Kx)+\lambda^{2}=(K^{2}x+Kx)^{2}-4(K^{3}x^{2}+% Ky^{2}).( italic_K start_POSTSUPERSCRIPT 2 end_POSTSUPERSCRIPT italic_x + italic_K italic_x ) start_POSTSUPERSCRIPT 2 end_POSTSUPERSCRIPT - 2 italic_λ ( italic_K start_POSTSUPERSCRIPT 2 end_POSTSUPERSCRIPT italic_x + italic_K italic_x ) + italic_λ start_POSTSUPERSCRIPT 2 end_POSTSUPERSCRIPT = ( italic_K start_POSTSUPERSCRIPT 2 end_POSTSUPERSCRIPT italic_x + italic_K italic_x ) start_POSTSUPERSCRIPT 2 end_POSTSUPERSCRIPT - 4 ( italic_K start_POSTSUPERSCRIPT 3 end_POSTSUPERSCRIPT italic_x start_POSTSUPERSCRIPT 2 end_POSTSUPERSCRIPT + italic_K italic_y start_POSTSUPERSCRIPT 2 end_POSTSUPERSCRIPT ) .

Collecting like terms together and simplifying, this yields

4K3x2+(2K2λ2Kλ)x+4Ky2+λ2=0.4superscript𝐾3superscript𝑥22superscript𝐾2𝜆2𝐾𝜆𝑥4𝐾superscript𝑦2superscript𝜆204K^{3}x^{2}+(-2K^{2}\lambda-2K\lambda)x+4Ky^{2}+\lambda^{2}=0.4 italic_K start_POSTSUPERSCRIPT 3 end_POSTSUPERSCRIPT italic_x start_POSTSUPERSCRIPT 2 end_POSTSUPERSCRIPT + ( - 2 italic_K start_POSTSUPERSCRIPT 2 end_POSTSUPERSCRIPT italic_λ - 2 italic_K italic_λ ) italic_x + 4 italic_K italic_y start_POSTSUPERSCRIPT 2 end_POSTSUPERSCRIPT + italic_λ start_POSTSUPERSCRIPT 2 end_POSTSUPERSCRIPT = 0 .

Completing the square in the x𝑥xitalic_x-variable, we get

(6.3) 4K3(x(λ(K+1)4K2))2+4Ky2=4K3(λ(K+1)4K2)2λ2.4superscript𝐾3superscript𝑥𝜆𝐾14superscript𝐾224𝐾superscript𝑦24superscript𝐾3superscript𝜆𝐾14superscript𝐾22superscript𝜆24K^{3}\left(x-\left(\frac{\lambda(K+1)}{4K^{2}}\right)\right)^{2}+4Ky^{2}=4K^{% 3}\left(\frac{\lambda(K+1)}{4K^{2}}\right)^{2}-\lambda^{2}.4 italic_K start_POSTSUPERSCRIPT 3 end_POSTSUPERSCRIPT ( italic_x - ( divide start_ARG italic_λ ( italic_K + 1 ) end_ARG start_ARG 4 italic_K start_POSTSUPERSCRIPT 2 end_POSTSUPERSCRIPT end_ARG ) ) start_POSTSUPERSCRIPT 2 end_POSTSUPERSCRIPT + 4 italic_K italic_y start_POSTSUPERSCRIPT 2 end_POSTSUPERSCRIPT = 4 italic_K start_POSTSUPERSCRIPT 3 end_POSTSUPERSCRIPT ( divide start_ARG italic_λ ( italic_K + 1 ) end_ARG start_ARG 4 italic_K start_POSTSUPERSCRIPT 2 end_POSTSUPERSCRIPT end_ARG ) start_POSTSUPERSCRIPT 2 end_POSTSUPERSCRIPT - italic_λ start_POSTSUPERSCRIPT 2 end_POSTSUPERSCRIPT .

Now, the right hand side of (6.3) is

4K3(λ(K+1)4K2)2λ24superscript𝐾3superscript𝜆𝐾14superscript𝐾22superscript𝜆2\displaystyle 4K^{3}\left(\frac{\lambda(K+1)}{4K^{2}}\right)^{2}-\lambda^{2}4 italic_K start_POSTSUPERSCRIPT 3 end_POSTSUPERSCRIPT ( divide start_ARG italic_λ ( italic_K + 1 ) end_ARG start_ARG 4 italic_K start_POSTSUPERSCRIPT 2 end_POSTSUPERSCRIPT end_ARG ) start_POSTSUPERSCRIPT 2 end_POSTSUPERSCRIPT - italic_λ start_POSTSUPERSCRIPT 2 end_POSTSUPERSCRIPT =4K3λ2(116K2+18K3+116K4)λ2absent4superscript𝐾3superscript𝜆2116superscript𝐾218superscript𝐾3116superscript𝐾4superscript𝜆2\displaystyle=4K^{3}\lambda^{2}\left(\frac{1}{16K^{2}}+\frac{1}{8K^{3}}+\frac{% 1}{16K^{4}}\right)-\lambda^{2}= 4 italic_K start_POSTSUPERSCRIPT 3 end_POSTSUPERSCRIPT italic_λ start_POSTSUPERSCRIPT 2 end_POSTSUPERSCRIPT ( divide start_ARG 1 end_ARG start_ARG 16 italic_K start_POSTSUPERSCRIPT 2 end_POSTSUPERSCRIPT end_ARG + divide start_ARG 1 end_ARG start_ARG 8 italic_K start_POSTSUPERSCRIPT 3 end_POSTSUPERSCRIPT end_ARG + divide start_ARG 1 end_ARG start_ARG 16 italic_K start_POSTSUPERSCRIPT 4 end_POSTSUPERSCRIPT end_ARG ) - italic_λ start_POSTSUPERSCRIPT 2 end_POSTSUPERSCRIPT
=Kλ24+λ22+λ24Kλ2absent𝐾superscript𝜆24superscript𝜆22superscript𝜆24𝐾superscript𝜆2\displaystyle=\frac{K\lambda^{2}}{4}+\frac{\lambda^{2}}{2}+\frac{\lambda^{2}}{% 4K}-\lambda^{2}= divide start_ARG italic_K italic_λ start_POSTSUPERSCRIPT 2 end_POSTSUPERSCRIPT end_ARG start_ARG 4 end_ARG + divide start_ARG italic_λ start_POSTSUPERSCRIPT 2 end_POSTSUPERSCRIPT end_ARG start_ARG 2 end_ARG + divide start_ARG italic_λ start_POSTSUPERSCRIPT 2 end_POSTSUPERSCRIPT end_ARG start_ARG 4 italic_K end_ARG - italic_λ start_POSTSUPERSCRIPT 2 end_POSTSUPERSCRIPT
=λ24K(K1)2.absentsuperscript𝜆24𝐾superscript𝐾12\displaystyle=\frac{\lambda^{2}}{4K}(K-1)^{2}.= divide start_ARG italic_λ start_POSTSUPERSCRIPT 2 end_POSTSUPERSCRIPT end_ARG start_ARG 4 italic_K end_ARG ( italic_K - 1 ) start_POSTSUPERSCRIPT 2 end_POSTSUPERSCRIPT .

As λ=±1𝜆plus-or-minus1\lambda=\pm 1italic_λ = ± 1 and K>1𝐾1K>1italic_K > 1, this term is strictly positive. From (6.3), we therefore obtain

(6.4) (x(λ(K+1)4K2))2(λ(K1)4K2)2+y2(λ(K1)4K)2=1.superscript𝑥𝜆𝐾14superscript𝐾22superscript𝜆𝐾14superscript𝐾22superscript𝑦2superscript𝜆𝐾14𝐾21\frac{\left(x-\left(\frac{\lambda(K+1)}{4K^{2}}\right)\right)^{2}}{\left(\frac% {\lambda(K-1)}{4K^{2}}\right)^{2}}+\frac{y^{2}}{\left(\frac{\lambda(K-1)}{4K}% \right)^{2}}=1.divide start_ARG ( italic_x - ( divide start_ARG italic_λ ( italic_K + 1 ) end_ARG start_ARG 4 italic_K start_POSTSUPERSCRIPT 2 end_POSTSUPERSCRIPT end_ARG ) ) start_POSTSUPERSCRIPT 2 end_POSTSUPERSCRIPT end_ARG start_ARG ( divide start_ARG italic_λ ( italic_K - 1 ) end_ARG start_ARG 4 italic_K start_POSTSUPERSCRIPT 2 end_POSTSUPERSCRIPT end_ARG ) start_POSTSUPERSCRIPT 2 end_POSTSUPERSCRIPT end_ARG + divide start_ARG italic_y start_POSTSUPERSCRIPT 2 end_POSTSUPERSCRIPT end_ARG start_ARG ( divide start_ARG italic_λ ( italic_K - 1 ) end_ARG start_ARG 4 italic_K end_ARG ) start_POSTSUPERSCRIPT 2 end_POSTSUPERSCRIPT end_ARG = 1 .

This gives the equations of the required ellipses. ∎

As w+h=(2K2)1>0subscript𝑤superscript2superscript𝐾210w_{+}-h=(2K^{2})^{-1}>0italic_w start_POSTSUBSCRIPT + end_POSTSUBSCRIPT - italic_h = ( 2 italic_K start_POSTSUPERSCRIPT 2 end_POSTSUPERSCRIPT ) start_POSTSUPERSCRIPT - 1 end_POSTSUPERSCRIPT > 0, it follows that EK+superscriptsubscript𝐸𝐾E_{K}^{+}italic_E start_POSTSUBSCRIPT italic_K end_POSTSUBSCRIPT start_POSTSUPERSCRIPT + end_POSTSUPERSCRIPT is contained in the right half-plane, and EKsuperscriptsubscript𝐸𝐾E_{K}^{-}italic_E start_POSTSUBSCRIPT italic_K end_POSTSUBSCRIPT start_POSTSUPERSCRIPT - end_POSTSUPERSCRIPT in the left half-plane.

Lemma 6.4.

The intersection of one the ellipses EK±superscriptsubscript𝐸𝐾plus-or-minusE_{K}^{\pm}italic_E start_POSTSUBSCRIPT italic_K end_POSTSUBSCRIPT start_POSTSUPERSCRIPT ± end_POSTSUPERSCRIPT and one of the lines LK±superscriptsubscript𝐿𝐾plus-or-minusL_{K}^{\pm}italic_L start_POSTSUBSCRIPT italic_K end_POSTSUBSCRIPT start_POSTSUPERSCRIPT ± end_POSTSUPERSCRIPT is a unique point given by

(±1K(K+1),±K12K(K+1)),plus-or-minus1𝐾𝐾1plus-or-minus𝐾12𝐾𝐾1\left(\pm\frac{1}{K(K+1)},\pm\frac{K-1}{2\sqrt{K}(K+1)}\right),( ± divide start_ARG 1 end_ARG start_ARG italic_K ( italic_K + 1 ) end_ARG , ± divide start_ARG italic_K - 1 end_ARG start_ARG 2 square-root start_ARG italic_K end_ARG ( italic_K + 1 ) end_ARG ) ,

for an appropriate choice of ±plus-or-minus\pm± in both coordinates to correspond to the four combinations of choices of lines and ellipses. Moreover, the intersection of γKsubscript𝛾𝐾\gamma_{K}italic_γ start_POSTSUBSCRIPT italic_K end_POSTSUBSCRIPT with LK±superscriptsubscript𝐿𝐾plus-or-minusL_{K}^{\pm}italic_L start_POSTSUBSCRIPT italic_K end_POSTSUBSCRIPT start_POSTSUPERSCRIPT ± end_POSTSUPERSCRIPT occurs at precisely the same four points.

Proof.

We will work with EK+superscriptsubscript𝐸𝐾E_{K}^{+}italic_E start_POSTSUBSCRIPT italic_K end_POSTSUBSCRIPT start_POSTSUPERSCRIPT + end_POSTSUPERSCRIPT and LK+superscriptsubscript𝐿𝐾L_{K}^{+}italic_L start_POSTSUBSCRIPT italic_K end_POSTSUBSCRIPT start_POSTSUPERSCRIPT + end_POSTSUPERSCRIPT. The other cases follow analogously. Plugging the equation y=K(K1)x/2𝑦𝐾𝐾1𝑥2y=\sqrt{K}(K-1)x/2italic_y = square-root start_ARG italic_K end_ARG ( italic_K - 1 ) italic_x / 2 for LK+superscriptsubscript𝐿𝐾L_{K}^{+}italic_L start_POSTSUBSCRIPT italic_K end_POSTSUBSCRIPT start_POSTSUPERSCRIPT + end_POSTSUPERSCRIPT into the equation for the ellipse EK+superscriptsubscript𝐸𝐾E_{K}^{+}italic_E start_POSTSUBSCRIPT italic_K end_POSTSUBSCRIPT start_POSTSUPERSCRIPT + end_POSTSUPERSCRIPT from (6.4), we obtain

K2(x(K+14K2))2+K(K1)2x24=(K1)216K2.superscript𝐾2superscript𝑥𝐾14superscript𝐾22𝐾superscript𝐾12superscript𝑥24superscript𝐾1216superscript𝐾2K^{2}\left(x-\left(\frac{K+1}{4K^{2}}\right)\right)^{2}+\frac{K(K-1)^{2}x^{2}}% {4}=\frac{(K-1)^{2}}{16K^{2}}.italic_K start_POSTSUPERSCRIPT 2 end_POSTSUPERSCRIPT ( italic_x - ( divide start_ARG italic_K + 1 end_ARG start_ARG 4 italic_K start_POSTSUPERSCRIPT 2 end_POSTSUPERSCRIPT end_ARG ) ) start_POSTSUPERSCRIPT 2 end_POSTSUPERSCRIPT + divide start_ARG italic_K ( italic_K - 1 ) start_POSTSUPERSCRIPT 2 end_POSTSUPERSCRIPT italic_x start_POSTSUPERSCRIPT 2 end_POSTSUPERSCRIPT end_ARG start_ARG 4 end_ARG = divide start_ARG ( italic_K - 1 ) start_POSTSUPERSCRIPT 2 end_POSTSUPERSCRIPT end_ARG start_ARG 16 italic_K start_POSTSUPERSCRIPT 2 end_POSTSUPERSCRIPT end_ARG .

Multiplying out the brackets, collecting like terms and simplifying, we obtain

x2(K(K+1)24)x(K+12)+14K=0.superscript𝑥2𝐾superscript𝐾124𝑥𝐾1214𝐾0x^{2}\left(\frac{K(K+1)^{2}}{4}\right)-x\left(\frac{K+1}{2}\right)+\frac{1}{4K% }=0.italic_x start_POSTSUPERSCRIPT 2 end_POSTSUPERSCRIPT ( divide start_ARG italic_K ( italic_K + 1 ) start_POSTSUPERSCRIPT 2 end_POSTSUPERSCRIPT end_ARG start_ARG 4 end_ARG ) - italic_x ( divide start_ARG italic_K + 1 end_ARG start_ARG 2 end_ARG ) + divide start_ARG 1 end_ARG start_ARG 4 italic_K end_ARG = 0 .

This factorizes as

(xK(K+1)212K)2=0,superscript𝑥𝐾𝐾1212𝐾20\left(\frac{x\sqrt{K}(K+1)}{2}-\frac{1}{2\sqrt{K}}\right)^{2}=0,( divide start_ARG italic_x square-root start_ARG italic_K end_ARG ( italic_K + 1 ) end_ARG start_ARG 2 end_ARG - divide start_ARG 1 end_ARG start_ARG 2 square-root start_ARG italic_K end_ARG end_ARG ) start_POSTSUPERSCRIPT 2 end_POSTSUPERSCRIPT = 0 ,

which has the unique solution x=1/(K(K+1))𝑥1𝐾𝐾1x=1/(K(K+1))italic_x = 1 / ( italic_K ( italic_K + 1 ) ). From this we obtain the value for y𝑦yitalic_y given in the statement of the lemma.

For the second part of the lemma, computing the intersection of γKsubscript𝛾𝐾\gamma_{K}italic_γ start_POSTSUBSCRIPT italic_K end_POSTSUBSCRIPT with LK±superscriptsubscript𝐿𝐾plus-or-minusL_{K}^{\pm}italic_L start_POSTSUBSCRIPT italic_K end_POSTSUBSCRIPT start_POSTSUPERSCRIPT ± end_POSTSUPERSCRIPT gives

4K3x2+K2(K1)2x2=1.4superscript𝐾3superscript𝑥2superscript𝐾2superscript𝐾12superscript𝑥214K^{3}x^{2}+K^{2}(K-1)^{2}x^{2}=1.4 italic_K start_POSTSUPERSCRIPT 3 end_POSTSUPERSCRIPT italic_x start_POSTSUPERSCRIPT 2 end_POSTSUPERSCRIPT + italic_K start_POSTSUPERSCRIPT 2 end_POSTSUPERSCRIPT ( italic_K - 1 ) start_POSTSUPERSCRIPT 2 end_POSTSUPERSCRIPT italic_x start_POSTSUPERSCRIPT 2 end_POSTSUPERSCRIPT = 1 .

This simplifies to

x2K2(K+1)2=1,superscript𝑥2superscript𝐾2superscript𝐾121x^{2}K^{2}(K+1)^{2}=1,italic_x start_POSTSUPERSCRIPT 2 end_POSTSUPERSCRIPT italic_K start_POSTSUPERSCRIPT 2 end_POSTSUPERSCRIPT ( italic_K + 1 ) start_POSTSUPERSCRIPT 2 end_POSTSUPERSCRIPT = 1 ,

from which the second part of the lemma follows. ∎

Refer to caption
Figure 3. The classification for K=2𝐾2K=2italic_K = 2. The red ellipses are E2±superscriptsubscript𝐸2plus-or-minusE_{2}^{\pm}italic_E start_POSTSUBSCRIPT 2 end_POSTSUBSCRIPT start_POSTSUPERSCRIPT ± end_POSTSUPERSCRIPT where fixed points are saddle points. The blue region is A2(E2+E2)¯subscript𝐴2¯superscriptsubscript𝐸2superscriptsubscript𝐸2A_{2}\setminus\overline{(E_{2}^{+}\cup E_{2}^{-})}italic_A start_POSTSUBSCRIPT 2 end_POSTSUBSCRIPT ∖ over¯ start_ARG ( italic_E start_POSTSUBSCRIPT 2 end_POSTSUBSCRIPT start_POSTSUPERSCRIPT + end_POSTSUPERSCRIPT ∪ italic_E start_POSTSUBSCRIPT 2 end_POSTSUBSCRIPT start_POSTSUPERSCRIPT - end_POSTSUPERSCRIPT ) end_ARG where fixed points are attracting. The green region gives fixed points that are repelling with complex conjugate eigenvalues. The purple region gives fixed points that are repelling with real eigenvalues.

We can now put everything together to classify the fixed points of H𝐻Hitalic_H in terms of their location. See Figure 3 for the example K=2𝐾2K=2italic_K = 2.

Theorem 6.5.

Let K>1𝐾1K>1italic_K > 1, let c𝑐c\in\mathbb{C}italic_c ∈ blackboard_C and let H=HK,0,c𝐻subscript𝐻𝐾0𝑐H=H_{K,0,c}italic_H = italic_H start_POSTSUBSCRIPT italic_K , 0 , italic_c end_POSTSUBSCRIPT. Denote by AKsubscript𝐴𝐾A_{K}italic_A start_POSTSUBSCRIPT italic_K end_POSTSUBSCRIPT the bounded component of the complement of the ellipse given by γ𝛾\gammaitalic_γ in Lemma 6.2. Suppose that z=x+iy𝑧𝑥𝑖𝑦z=x+iyitalic_z = italic_x + italic_i italic_y is a fixed point of H𝐻Hitalic_H. Then:

  1. (a)

    z𝑧zitalic_z is attracting if z𝑧zitalic_z lies in AK(EK+EK)¯subscript𝐴𝐾¯superscriptsubscript𝐸𝐾superscriptsubscript𝐸𝐾A_{K}\setminus\overline{(E_{K}^{+}\cup E_{K}^{-})}italic_A start_POSTSUBSCRIPT italic_K end_POSTSUBSCRIPT ∖ over¯ start_ARG ( italic_E start_POSTSUBSCRIPT italic_K end_POSTSUBSCRIPT start_POSTSUPERSCRIPT + end_POSTSUPERSCRIPT ∪ italic_E start_POSTSUBSCRIPT italic_K end_POSTSUBSCRIPT start_POSTSUPERSCRIPT - end_POSTSUPERSCRIPT ) end_ARG;

  2. (b)

    z𝑧zitalic_z is repelling if z𝑧zitalic_z lies in (AKEK+EK)¯¯subscript𝐴𝐾superscriptsubscript𝐸𝐾superscriptsubscript𝐸𝐾\mathbb{C}\setminus\overline{(A_{K}\cup E_{K}^{+}\cup E_{K}^{-})}blackboard_C ∖ over¯ start_ARG ( italic_A start_POSTSUBSCRIPT italic_K end_POSTSUBSCRIPT ∪ italic_E start_POSTSUBSCRIPT italic_K end_POSTSUBSCRIPT start_POSTSUPERSCRIPT + end_POSTSUPERSCRIPT ∪ italic_E start_POSTSUBSCRIPT italic_K end_POSTSUBSCRIPT start_POSTSUPERSCRIPT - end_POSTSUPERSCRIPT ) end_ARG;

  3. (c)

    z𝑧zitalic_z is a saddle point if z𝑧zitalic_z lies in EK±superscriptsubscript𝐸𝐾plus-or-minusE_{K}^{\pm}italic_E start_POSTSUBSCRIPT italic_K end_POSTSUBSCRIPT start_POSTSUPERSCRIPT ± end_POSTSUPERSCRIPT.

Proof.

By Lemma 6.1, λ1,λ2subscript𝜆1subscript𝜆2\lambda_{1},\lambda_{2}italic_λ start_POSTSUBSCRIPT 1 end_POSTSUBSCRIPT , italic_λ start_POSTSUBSCRIPT 2 end_POSTSUBSCRIPT are non-real complex conjugates in the sectors SK±superscriptsubscript𝑆𝐾plus-or-minusS_{K}^{\pm}italic_S start_POSTSUBSCRIPT italic_K end_POSTSUBSCRIPT start_POSTSUPERSCRIPT ± end_POSTSUPERSCRIPT. Then by Lemma 6.2 and (6.2), cases (a) and (b) in SK±superscriptsubscript𝑆𝐾plus-or-minusS_{K}^{\pm}italic_S start_POSTSUBSCRIPT italic_K end_POSTSUBSCRIPT start_POSTSUPERSCRIPT ± end_POSTSUPERSCRIPT follow directly. Denote by ΣK±superscriptsubscriptΣ𝐾plus-or-minus\Sigma_{K}^{\pm}roman_Σ start_POSTSUBSCRIPT italic_K end_POSTSUBSCRIPT start_POSTSUPERSCRIPT ± end_POSTSUPERSCRIPT the two sectors given by the complement of SK±superscriptsubscript𝑆𝐾plus-or-minusS_{K}^{\pm}italic_S start_POSTSUBSCRIPT italic_K end_POSTSUBSCRIPT start_POSTSUPERSCRIPT ± end_POSTSUPERSCRIPT. By Lemma 6.3, case (c) in ΣK±superscriptsubscriptΣ𝐾plus-or-minus\Sigma_{K}^{\pm}roman_Σ start_POSTSUBSCRIPT italic_K end_POSTSUBSCRIPT start_POSTSUPERSCRIPT ± end_POSTSUPERSCRIPT follows directly. By Lemma 6.4, as γKsubscript𝛾𝐾\gamma_{K}italic_γ start_POSTSUBSCRIPT italic_K end_POSTSUBSCRIPT, LK±superscriptsubscript𝐿𝐾plus-or-minusL_{K}^{\pm}italic_L start_POSTSUBSCRIPT italic_K end_POSTSUBSCRIPT start_POSTSUPERSCRIPT ± end_POSTSUPERSCRIPT and EK±superscriptsubscript𝐸𝐾plus-or-minus\partial E_{K}^{\pm}∂ italic_E start_POSTSUBSCRIPT italic_K end_POSTSUBSCRIPT start_POSTSUPERSCRIPT ± end_POSTSUPERSCRIPT all meet in common points, it follows that Ek±¯¯superscriptsubscript𝐸𝑘plus-or-minus\overline{E_{k}^{\pm}}over¯ start_ARG italic_E start_POSTSUBSCRIPT italic_k end_POSTSUBSCRIPT start_POSTSUPERSCRIPT ± end_POSTSUPERSCRIPT end_ARG partition each of ΣK±superscriptsubscriptΣ𝐾plus-or-minus\Sigma_{K}^{\pm}roman_Σ start_POSTSUBSCRIPT italic_K end_POSTSUBSCRIPT start_POSTSUPERSCRIPT ± end_POSTSUPERSCRIPT into two components, one where both eigenvalues are less than one, and one where both eigenvalues are greater than one. These cases complete (a) and (b). ∎

7. Examples

In this section, we will construct the examples that give Theorem 1.4. We already showed in Section 5 that it is possible for HK,0,csubscript𝐻𝐾0𝑐H_{K,0,c}italic_H start_POSTSUBSCRIPT italic_K , 0 , italic_c end_POSTSUBSCRIPT to have two, three or four fixed points. In general, solving HK,θ,c(z)=zsubscript𝐻𝐾𝜃𝑐𝑧𝑧H_{K,\theta,c}(z)=zitalic_H start_POSTSUBSCRIPT italic_K , italic_θ , italic_c end_POSTSUBSCRIPT ( italic_z ) = italic_z yields two quadratic equations in x𝑥xitalic_x and y𝑦yitalic_y. Bezout’s Theorem then implies that the maximum number of solutions is four unless there is a curve of fixed points of HK,θ,csubscript𝐻𝐾𝜃𝑐H_{K,\theta,c}italic_H start_POSTSUBSCRIPT italic_K , italic_θ , italic_c end_POSTSUBSCRIPT. However, this latter case would imply a curve of points where the derivative of HK,θ,csubscript𝐻𝐾𝜃𝑐H_{K,\theta,c}italic_H start_POSTSUBSCRIPT italic_K , italic_θ , italic_c end_POSTSUBSCRIPT is the identity, which does not occur. This proves Theorem 1.4 (a).

For part (b), consider the parameters K=0.5𝐾0.5K=0.5italic_K = 0.5 θ=0𝜃0\theta=0italic_θ = 0 and c=3/2i/2𝑐32𝑖2c=-3/2-i/2italic_c = - 3 / 2 - italic_i / 2. We typically choose K>1𝐾1K>1italic_K > 1, but our choice of parameters is conjugate to K=2𝐾2K=2italic_K = 2, θ=π/2𝜃𝜋2\theta=\pi/2italic_θ = italic_π / 2 and c=3/8i/8𝑐38𝑖8c=-3/8-i/8italic_c = - 3 / 8 - italic_i / 8 by [11, Proposition 3.1]. We can compute that a fixed point z0subscript𝑧0z_{0}italic_z start_POSTSUBSCRIPT 0 end_POSTSUBSCRIPT of H1/2,0,3/2i/2subscript𝐻12032𝑖2H_{1/2,0,-3/2-i/2}italic_H start_POSTSUBSCRIPT 1 / 2 , 0 , - 3 / 2 - italic_i / 2 end_POSTSUBSCRIPT lies at approximately 1.1950.228i1.1950.228𝑖-1.195-0.228i- 1.195 - 0.228 italic_i, see Figure 2.

Computing the eigenvalues of H1/2,0,3/2i/2(z0)superscriptsubscript𝐻12032𝑖2subscript𝑧0H_{1/2,0,-3/2-i/2}^{\prime}(z_{0})italic_H start_POSTSUBSCRIPT 1 / 2 , 0 , - 3 / 2 - italic_i / 2 end_POSTSUBSCRIPT start_POSTSUPERSCRIPT ′ end_POSTSUPERSCRIPT ( italic_z start_POSTSUBSCRIPT 0 end_POSTSUBSCRIPT ) yields two complex conjugate eigenvalues λ±subscript𝜆plus-or-minus\lambda_{\pm}italic_λ start_POSTSUBSCRIPT ± end_POSTSUBSCRIPT which are approximately 0.896±0.121iplus-or-minus0.8960.121𝑖-0.896\pm 0.121i- 0.896 ± 0.121 italic_i. This yields

|λ±|0.904subscript𝜆plus-or-minus0.904|\lambda_{\pm}|\approx 0.904| italic_λ start_POSTSUBSCRIPT ± end_POSTSUBSCRIPT | ≈ 0.904

and thus we conclude that z0subscript𝑧0z_{0}italic_z start_POSTSUBSCRIPT 0 end_POSTSUBSCRIPT is an attracting fixed point of H1/2,0,3/2i/2subscript𝐻12032𝑖2H_{1/2,0,-3/2-i/2}italic_H start_POSTSUBSCRIPT 1 / 2 , 0 , - 3 / 2 - italic_i / 2 end_POSTSUBSCRIPT. However, we may computationally check that

(7.1) H1/2,0,3/2i/28(z0)8.8751.193i.superscriptsubscript𝐻12032𝑖28subscript𝑧08.8751.193𝑖H_{1/2,0,-3/2-i/2}^{8}(z_{0})\approx-8.875-1.193i.italic_H start_POSTSUBSCRIPT 1 / 2 , 0 , - 3 / 2 - italic_i / 2 end_POSTSUBSCRIPT start_POSTSUPERSCRIPT 8 end_POSTSUPERSCRIPT ( italic_z start_POSTSUBSCRIPT 0 end_POSTSUBSCRIPT ) ≈ - 8.875 - 1.193 italic_i .

By [11, Theorem 6.3], the Mandelbrot set 1/2,0subscript120\mathcal{M}_{1/2,0}caligraphic_M start_POSTSUBSCRIPT 1 / 2 , 0 end_POSTSUBSCRIPT can be characterized as the set of c𝑐c\in\mathbb{C}italic_c ∈ blackboard_C for which H1/2,0,cn(0)8superscriptsubscript𝐻120𝑐𝑛08H_{1/2,0,c}^{n}(0)\leq 8italic_H start_POSTSUBSCRIPT 1 / 2 , 0 , italic_c end_POSTSUBSCRIPT start_POSTSUPERSCRIPT italic_n end_POSTSUPERSCRIPT ( 0 ) ≤ 8 for all n𝑛n\in\mathbb{N}italic_n ∈ blackboard_N. Combining this with (7.1), we conclude that 3/2i/21/2,032𝑖2subscript120-3/2-i/2\notin\mathcal{M}_{1/2,0}- 3 / 2 - italic_i / 2 ∉ caligraphic_M start_POSTSUBSCRIPT 1 / 2 , 0 end_POSTSUBSCRIPT, see Figure 4.

Refer to caption
Figure 4. The Mandelbrot set 1/2,0subscript120\mathcal{M}_{1/2,0}caligraphic_M start_POSTSUBSCRIPT 1 / 2 , 0 end_POSTSUBSCRIPT with c=3/2i/2𝑐32𝑖2c=-3/2-i/2italic_c = - 3 / 2 - italic_i / 2 marked and a zoom inset.

This example completes the proof of Theorem 1.4 (b). For part (c), we already know from Theorem 6.5 that HK,0,csubscript𝐻𝐾0𝑐H_{K,0,c}italic_H start_POSTSUBSCRIPT italic_K , 0 , italic_c end_POSTSUBSCRIPT may have saddle fixed points. We will look more in depth at the example corresponding to K=5𝐾5K=5italic_K = 5, θ=0𝜃0\theta=0italic_θ = 0 and c=0.1𝑐0.1c=-0.1italic_c = - 0.1. See Figure 5.

Refer to caption
Figure 5. Part of the Mandelbrot set 5,0subscript50\mathcal{M}_{5,0}caligraphic_M start_POSTSUBSCRIPT 5 , 0 end_POSTSUBSCRIPT with c=0.1𝑐0.1c=-0.1italic_c = - 0.1 marked.

We have

H5,0,0.1(z)=25x2y20.1+10ixy,subscript𝐻500.1𝑧25superscript𝑥2superscript𝑦20.110𝑖𝑥𝑦H_{5,0,-0.1}(z)=25x^{2}-y^{2}-0.1+10ixy,italic_H start_POSTSUBSCRIPT 5 , 0 , - 0.1 end_POSTSUBSCRIPT ( italic_z ) = 25 italic_x start_POSTSUPERSCRIPT 2 end_POSTSUPERSCRIPT - italic_y start_POSTSUPERSCRIPT 2 end_POSTSUPERSCRIPT - 0.1 + 10 italic_i italic_x italic_y ,

from which we see that restricting to the real axis gives the real quadratic polynomial H5,0,0.1(x)=25x20.1subscript𝐻500.1𝑥25superscript𝑥20.1H_{5,0,-0.1}(x)=25x^{2}-0.1italic_H start_POSTSUBSCRIPT 5 , 0 , - 0.1 end_POSTSUBSCRIPT ( italic_x ) = 25 italic_x start_POSTSUPERSCRIPT 2 end_POSTSUPERSCRIPT - 0.1. In particular, on the real axis, H5,0,0.1subscript𝐻500.1H_{5,0,-0.1}italic_H start_POSTSUBSCRIPT 5 , 0 , - 0.1 end_POSTSUBSCRIPT is conjugate to the quadratic polynomial P2.5(z)=z22.5subscript𝑃2.5𝑧superscript𝑧22.5P_{-2.5}(z)=z^{2}-2.5italic_P start_POSTSUBSCRIPT - 2.5 end_POSTSUBSCRIPT ( italic_z ) = italic_z start_POSTSUPERSCRIPT 2 end_POSTSUPERSCRIPT - 2.5. As 2.52.5-2.5- 2.5 is not in the Mandelbrot set, it follows that the Julia set of P2.5subscript𝑃2.5P_{-2.5}italic_P start_POSTSUBSCRIPT - 2.5 end_POSTSUBSCRIPT is a Cantor subset of \mathbb{R}blackboard_R. We deduce that BO(H5,0,0.1)𝐵𝑂subscript𝐻500.1BO(H_{5,0,-0.1})\cap\mathbb{R}italic_B italic_O ( italic_H start_POSTSUBSCRIPT 5 , 0 , - 0.1 end_POSTSUBSCRIPT ) ∩ blackboard_R is also a Cantor subset of \mathbb{R}blackboard_R.

Now, we can compute that H5,0,0.1subscript𝐻500.1H_{5,0,-0.1}italic_H start_POSTSUBSCRIPT 5 , 0 , - 0.1 end_POSTSUBSCRIPT has a fixed point at z00.086subscript𝑧00.086z_{0}\approx 0.086italic_z start_POSTSUBSCRIPT 0 end_POSTSUBSCRIPT ≈ 0.086. Computing the eigenvalues of H5,0,0.1(z0)subscriptsuperscript𝐻500.1subscript𝑧0H^{\prime}_{5,0,-0.1}(z_{0})italic_H start_POSTSUPERSCRIPT ′ end_POSTSUPERSCRIPT start_POSTSUBSCRIPT 5 , 0 , - 0.1 end_POSTSUBSCRIPT ( italic_z start_POSTSUBSCRIPT 0 end_POSTSUBSCRIPT ), we obtain λ1,λ2subscript𝜆1subscript𝜆2\lambda_{1},\lambda_{2}italic_λ start_POSTSUBSCRIPT 1 end_POSTSUBSCRIPT , italic_λ start_POSTSUBSCRIPT 2 end_POSTSUBSCRIPT which are approximately 4.3174.3174.3174.317 and 0.8630.8630.8630.863 respectively. Evidently this implies that z0subscript𝑧0z_{0}italic_z start_POSTSUBSCRIPT 0 end_POSTSUBSCRIPT is a saddle fixed point of H5,0,0.1subscript𝐻500.1H_{5,0,-0.1}italic_H start_POSTSUBSCRIPT 5 , 0 , - 0.1 end_POSTSUBSCRIPT. The eigenvectors for λ1,λ2subscript𝜆1subscript𝜆2\lambda_{1},\lambda_{2}italic_λ start_POSTSUBSCRIPT 1 end_POSTSUBSCRIPT , italic_λ start_POSTSUBSCRIPT 2 end_POSTSUBSCRIPT are (1,0)10(1,0)( 1 , 0 ) and (0,1)01(0,1)( 0 , 1 ) respectively, which means that H5,0,0.1subscript𝐻500.1H_{5,0,-0.1}italic_H start_POSTSUBSCRIPT 5 , 0 , - 0.1 end_POSTSUBSCRIPT is repelling in the x𝑥xitalic_x-direction and attracting in the y𝑦yitalic_y-direction near z0subscript𝑧0z_{0}italic_z start_POSTSUBSCRIPT 0 end_POSTSUBSCRIPT.

More precisely, the version of the Hartman-Grobman Theorem from [12] provides a C1superscript𝐶1C^{1}italic_C start_POSTSUPERSCRIPT 1 end_POSTSUPERSCRIPT conjugacy in a neighborhood 𝒩𝒩\mathcal{N}caligraphic_N of z0subscript𝑧0z_{0}italic_z start_POSTSUBSCRIPT 0 end_POSTSUBSCRIPT of the diffeomorphism H5,0,0.1|𝒩evaluated-atsubscript𝐻500.1𝒩H_{5,0,-0.1}|_{\mathcal{N}}italic_H start_POSTSUBSCRIPT 5 , 0 , - 0.1 end_POSTSUBSCRIPT | start_POSTSUBSCRIPT caligraphic_N end_POSTSUBSCRIPT to the derivative A:=H5,0,0.1(z0)assign𝐴subscriptsuperscript𝐻500.1subscript𝑧0A:=H^{\prime}_{5,0,-0.1}(z_{0})italic_A := italic_H start_POSTSUPERSCRIPT ′ end_POSTSUPERSCRIPT start_POSTSUBSCRIPT 5 , 0 , - 0.1 end_POSTSUBSCRIPT ( italic_z start_POSTSUBSCRIPT 0 end_POSTSUBSCRIPT ). That is, there exists a C1superscript𝐶1C^{1}italic_C start_POSTSUPERSCRIPT 1 end_POSTSUPERSCRIPT invertible function φ𝜑\varphiitalic_φ such that φ(z0)=0𝜑subscript𝑧00\varphi(z_{0})=0italic_φ ( italic_z start_POSTSUBSCRIPT 0 end_POSTSUBSCRIPT ) = 0 and

(7.2) φH5,0,0.1φ1=A.𝜑subscript𝐻500.1superscript𝜑1𝐴\varphi\circ H_{5,0,-0.1}\circ\varphi^{-1}=A.italic_φ ∘ italic_H start_POSTSUBSCRIPT 5 , 0 , - 0.1 end_POSTSUBSCRIPT ∘ italic_φ start_POSTSUPERSCRIPT - 1 end_POSTSUPERSCRIPT = italic_A .

Then the stable manifold WSsuperscript𝑊𝑆W^{S}italic_W start_POSTSUPERSCRIPT italic_S end_POSTSUPERSCRIPT is given by φ1(φ(𝒩)i)superscript𝜑1𝜑𝒩𝑖\varphi^{-1}(\varphi(\mathcal{N})\cap i\mathbb{R})italic_φ start_POSTSUPERSCRIPT - 1 end_POSTSUPERSCRIPT ( italic_φ ( caligraphic_N ) ∩ italic_i blackboard_R ) and the unstable manifold WUsuperscript𝑊𝑈W^{U}italic_W start_POSTSUPERSCRIPT italic_U end_POSTSUPERSCRIPT is given by φ1(φ(𝒩))superscript𝜑1𝜑𝒩\varphi^{-1}(\varphi(\mathcal{N})\cap\mathbb{R})italic_φ start_POSTSUPERSCRIPT - 1 end_POSTSUPERSCRIPT ( italic_φ ( caligraphic_N ) ∩ blackboard_R ).

Proposition 7.1.

The component of BO(H5,0,0.1)𝒩𝐵𝑂subscript𝐻500.1𝒩BO(H_{5,0,-0.1})\cap\mathcal{N}italic_B italic_O ( italic_H start_POSTSUBSCRIPT 5 , 0 , - 0.1 end_POSTSUBSCRIPT ) ∩ caligraphic_N containing z0subscript𝑧0z_{0}italic_z start_POSTSUBSCRIPT 0 end_POSTSUBSCRIPT is precisely the stable manifold WSsuperscript𝑊𝑆W^{S}italic_W start_POSTSUPERSCRIPT italic_S end_POSTSUPERSCRIPT and is, in particular, a curve.

Proof.

As BO(H5,0,0.1)𝐵𝑂subscript𝐻500.1BO(H_{5,0,-0.1})\cap\mathbb{R}italic_B italic_O ( italic_H start_POSTSUBSCRIPT 5 , 0 , - 0.1 end_POSTSUBSCRIPT ) ∩ blackboard_R is totally disconnected, for any δ>0𝛿0\delta>0italic_δ > 0, the real interval (z0δ,z0+δ)subscript𝑧0𝛿subscript𝑧0𝛿(z_{0}-\delta,z_{0}+\delta)( italic_z start_POSTSUBSCRIPT 0 end_POSTSUBSCRIPT - italic_δ , italic_z start_POSTSUBSCRIPT 0 end_POSTSUBSCRIPT + italic_δ ) intersects I(H5,0,0.1)𝐼subscript𝐻500.1I(H_{5,0,-0.1})italic_I ( italic_H start_POSTSUBSCRIPT 5 , 0 , - 0.1 end_POSTSUBSCRIPT ). Choose such a point zδsubscript𝑧𝛿z_{\delta}italic_z start_POSTSUBSCRIPT italic_δ end_POSTSUBSCRIPT. Then as I(H5,0,0.1)𝐼subscript𝐻500.1I(H_{5,0,-0.1})italic_I ( italic_H start_POSTSUBSCRIPT 5 , 0 , - 0.1 end_POSTSUBSCRIPT ) is open and connected, it is path connected. In particular, there is a closed smooth path γδsubscript𝛾𝛿\gamma_{\delta}italic_γ start_POSTSUBSCRIPT italic_δ end_POSTSUBSCRIPT in subscript\mathbb{C}_{\infty}blackboard_C start_POSTSUBSCRIPT ∞ end_POSTSUBSCRIPT that connects zδsubscript𝑧𝛿z_{\delta}italic_z start_POSTSUBSCRIPT italic_δ end_POSTSUBSCRIPT to \infty and otherwise is contained in I(H5,0,0.1)𝐼subscript𝐻500.1I(H_{5,0,-0.1})italic_I ( italic_H start_POSTSUBSCRIPT 5 , 0 , - 0.1 end_POSTSUBSCRIPT ). Choose δ>0𝛿0\delta>0italic_δ > 0 small enough that (z0δ,z0+δ)𝒩subscript𝑧0𝛿subscript𝑧0𝛿𝒩(z_{0}-\delta,z_{0}+\delta)\subset\mathcal{N}( italic_z start_POSTSUBSCRIPT 0 end_POSTSUBSCRIPT - italic_δ , italic_z start_POSTSUBSCRIPT 0 end_POSTSUBSCRIPT + italic_δ ) ⊂ caligraphic_N. Let T=φ(γδ𝒩)𝑇𝜑subscript𝛾𝛿𝒩T=\varphi(\gamma_{\delta}\cap\mathcal{N})italic_T = italic_φ ( italic_γ start_POSTSUBSCRIPT italic_δ end_POSTSUBSCRIPT ∩ caligraphic_N ). By construction, for zφ(N)𝑧𝜑𝑁z\in\varphi(N)italic_z ∈ italic_φ ( italic_N ) with non-zero imaginary part we have

|Im(Am(z))|=|Im(z)|λ2mImsuperscript𝐴𝑚𝑧Im𝑧superscriptsubscript𝜆2𝑚|\operatorname{Im}(A^{-m}(z))|=\frac{|\operatorname{Im}(z)|}{\lambda_{2}^{m}}\to\infty| roman_Im ( italic_A start_POSTSUPERSCRIPT - italic_m end_POSTSUPERSCRIPT ( italic_z ) ) | = divide start_ARG | roman_Im ( italic_z ) | end_ARG start_ARG italic_λ start_POSTSUBSCRIPT 2 end_POSTSUBSCRIPT start_POSTSUPERSCRIPT italic_m end_POSTSUPERSCRIPT end_ARG → ∞

as m𝑚m\to\inftyitalic_m → ∞ because λ2<1subscript𝜆21\lambda_{2}<1italic_λ start_POSTSUBSCRIPT 2 end_POSTSUBSCRIPT < 1. Moreover, for any zφ(N)𝑧𝜑𝑁z\in\varphi(N)italic_z ∈ italic_φ ( italic_N ),

Re(Am(z))=Re(z)λ1m0Resuperscript𝐴𝑚𝑧Re𝑧superscriptsubscript𝜆1𝑚0\operatorname{Re}(A^{-m}(z))=\frac{\operatorname{Re}(z)}{\lambda_{1}^{m}}\to 0roman_Re ( italic_A start_POSTSUPERSCRIPT - italic_m end_POSTSUPERSCRIPT ( italic_z ) ) = divide start_ARG roman_Re ( italic_z ) end_ARG start_ARG italic_λ start_POSTSUBSCRIPT 1 end_POSTSUBSCRIPT start_POSTSUPERSCRIPT italic_m end_POSTSUPERSCRIPT end_ARG → 0

as m𝑚m\to\inftyitalic_m → ∞ because λ1>1subscript𝜆11\lambda_{1}>1italic_λ start_POSTSUBSCRIPT 1 end_POSTSUBSCRIPT > 1.

Since φ(𝒩)𝜑𝒩\varphi(\mathcal{N})italic_φ ( caligraphic_N ) is open, we may assume that T𝑇Titalic_T contains elements with positive imaginary parts and elements with negative imaginary parts. Therefore, for any ε>0𝜀0\varepsilon>0italic_ε > 0, there exists M𝑀M\in\mathbb{N}italic_M ∈ blackboard_N such that for mM𝑚𝑀m\geq Mitalic_m ≥ italic_M we have dist(Am(T)φ(𝒩),i)<εdistsuperscript𝐴𝑚𝑇𝜑𝒩𝑖𝜀\operatorname{dist}(A^{-m}(T)\cap\varphi(\mathcal{N}),i\mathbb{R})<\varepsilonroman_dist ( italic_A start_POSTSUPERSCRIPT - italic_m end_POSTSUPERSCRIPT ( italic_T ) ∩ italic_φ ( caligraphic_N ) , italic_i blackboard_R ) < italic_ε and Am(T)superscript𝐴𝑚𝑇A^{-m}(T)italic_A start_POSTSUPERSCRIPT - italic_m end_POSTSUPERSCRIPT ( italic_T ) and the boundary of φ(𝒩)𝜑𝒩\varphi(\mathcal{N})italic_φ ( caligraphic_N ) share an element with positive imaginary part in common and, moreover, an element with negative imaginary part in common. In particular, Am(T)φ(𝒩)superscript𝐴𝑚𝑇𝜑𝒩A^{-m}(T)\cap\varphi(\mathcal{N})italic_A start_POSTSUPERSCRIPT - italic_m end_POSTSUPERSCRIPT ( italic_T ) ∩ italic_φ ( caligraphic_N ) separates φ(𝒩)𝜑𝒩\varphi(\mathcal{N})italic_φ ( caligraphic_N ) into two components. Evidently, as m𝑚mitalic_m increases, Am(T)φ(𝒩)superscript𝐴𝑚𝑇𝜑𝒩A^{-m}(T)\cap\varphi(\mathcal{N})italic_A start_POSTSUPERSCRIPT - italic_m end_POSTSUPERSCRIPT ( italic_T ) ∩ italic_φ ( caligraphic_N ) accumulates on iφ(𝒩)𝑖𝜑𝒩i\mathbb{R}\cap\varphi(\mathcal{N})italic_i blackboard_R ∩ italic_φ ( caligraphic_N ).

Transferring this back to the dynamical plane of H5,0,0.1subscript𝐻500.1H_{5,0,-0.1}italic_H start_POSTSUBSCRIPT 5 , 0 , - 0.1 end_POSTSUBSCRIPT, using the conjugacy (7.2), and the complete invariance of I(H5,0,0.1)𝐼subscript𝐻500.1I(H_{5,0,-0.1})italic_I ( italic_H start_POSTSUBSCRIPT 5 , 0 , - 0.1 end_POSTSUBSCRIPT ), we obtain curves φ1(Am(T)φ(𝒩))superscript𝜑1superscript𝐴𝑚𝑇𝜑𝒩\varphi^{-1}(A^{-m}(T)\cap\varphi(\mathcal{N}))italic_φ start_POSTSUPERSCRIPT - 1 end_POSTSUPERSCRIPT ( italic_A start_POSTSUPERSCRIPT - italic_m end_POSTSUPERSCRIPT ( italic_T ) ∩ italic_φ ( caligraphic_N ) ) contained in 𝒩I(H5,0,0.1)𝒩𝐼subscript𝐻500.1\mathcal{N}\cap I(H_{5,0,-0.1})caligraphic_N ∩ italic_I ( italic_H start_POSTSUBSCRIPT 5 , 0 , - 0.1 end_POSTSUBSCRIPT ) which accumulate on WS𝒩superscript𝑊𝑆𝒩W^{S}\cap\mathcal{N}italic_W start_POSTSUPERSCRIPT italic_S end_POSTSUPERSCRIPT ∩ caligraphic_N. As this argument holds for both cases zδ<z0subscript𝑧𝛿subscript𝑧0z_{\delta}<z_{0}italic_z start_POSTSUBSCRIPT italic_δ end_POSTSUBSCRIPT < italic_z start_POSTSUBSCRIPT 0 end_POSTSUBSCRIPT and zδ>z0subscript𝑧𝛿subscript𝑧0z_{\delta}>z_{0}italic_z start_POSTSUBSCRIPT italic_δ end_POSTSUBSCRIPT > italic_z start_POSTSUBSCRIPT 0 end_POSTSUBSCRIPT, we accumulate such curves on WSsuperscript𝑊𝑆W^{S}italic_W start_POSTSUPERSCRIPT italic_S end_POSTSUPERSCRIPT from both left and right. As WSBO(H5,0,0.1)superscript𝑊𝑆𝐵𝑂subscript𝐻500.1W^{S}\subset BO(H_{5,0,-0.1})italic_W start_POSTSUPERSCRIPT italic_S end_POSTSUPERSCRIPT ⊂ italic_B italic_O ( italic_H start_POSTSUBSCRIPT 5 , 0 , - 0.1 end_POSTSUBSCRIPT ), this completes the proof. ∎

We can extend the conjugacy φ𝜑\varphiitalic_φ along the stable manifold via (7.2) all the way to a pair of repelling fixed points at w±=1/10±i/25subscript𝑤plus-or-minusplus-or-minus110𝑖25w_{\pm}=1/10\pm i/2\sqrt{5}italic_w start_POSTSUBSCRIPT ± end_POSTSUBSCRIPT = 1 / 10 ± italic_i / 2 square-root start_ARG 5 end_ARG. This implies that the component of BO(H5,0,0.1)𝐵𝑂subscript𝐻500.1BO(H_{5,0,-0.1})italic_B italic_O ( italic_H start_POSTSUBSCRIPT 5 , 0 , - 0.1 end_POSTSUBSCRIPT ) containing z0subscript𝑧0z_{0}italic_z start_POSTSUBSCRIPT 0 end_POSTSUBSCRIPT is a curve connecting w+subscript𝑤w_{+}italic_w start_POSTSUBSCRIPT + end_POSTSUBSCRIPT to wsubscript𝑤w_{-}italic_w start_POSTSUBSCRIPT - end_POSTSUBSCRIPT passing through z0subscript𝑧0z_{0}italic_z start_POSTSUBSCRIPT 0 end_POSTSUBSCRIPT. However, we do not know if this is the whole component.

The same argument in Proposition 7.1 can be used at periodic points of H5,0,0.1subscript𝐻500.1H_{5,0,-0.1}italic_H start_POSTSUBSCRIPT 5 , 0 , - 0.1 end_POSTSUBSCRIPT contained in the real axis. Moreover, we can use complete invariance to take pre-images of WSsuperscript𝑊𝑆W^{S}italic_W start_POSTSUPERSCRIPT italic_S end_POSTSUPERSCRIPT. In both cases, we see that the intersection of components of BO(H5,0,0.1)𝐵𝑂subscript𝐻500.1BO(H_{5,0,-0.1})italic_B italic_O ( italic_H start_POSTSUBSCRIPT 5 , 0 , - 0.1 end_POSTSUBSCRIPT ) with a neighborhood of the real axis yields curves. See Figure 6 and a zoom back in Figure 1.

Refer to caption
Figure 6. The dynamical picture for H5,0,0.1subscript𝐻500.1H_{5,0,-0.1}italic_H start_POSTSUBSCRIPT 5 , 0 , - 0.1 end_POSTSUBSCRIPT.

Acknowledgements: The authors thanks James Waterman for many interesting conversations on the topic of this paper. Figure 3 was created using Desmos. The other figures were created using Ultra Fractal 6.

References

  • [1] A. F. Beardon, Iteration of Rational Functions, Springer, Berlin, 1991.
  • [2] W. Bergweiler, Iteration of quasiregular mappings, Comput. Methods Funct. Theory, 10 (2010), 455-481.
  • [3] W. Bergweiler, D. A. Nicks, Foundations for an iteration theory of entire quasiregular maps, Israel J. Math., 201 (2014), 147-184.
  • [4] B. Branner, N. Fagella, Quasiconformal surgery in holomorphic dynamics, Cambridge Stud. Adv. Math., 141, Cambridge University Press, Cambridge, 2014.
  • [5] L. Carleson, T. Gamelin, Complex Dynamics, Springer, New York, 1993.
  • [6] A. Douady, J. H. Hubbard, Dynamical study of complex polynomials. Part I, Publ. Math. Orsay, 84-2, (1985), 154 pp.
  • [7] A. Douady, J. H. Hubbard, Dynamical study of complex polynomials. Part II, Publ. Math. Orsay, 85-4 (1984), 75 pp.
  • [8] A. Douady, J. H. Hubbard, On the dynamics of polynomial-like mappings, Ann. Sci. École Norm. Sup., (4) 18 (1985), no. 2, 287-343.
  • [9] A. Fletcher, R. Fryer, On Böttcher Coordinates and quasiregular maps, Quasiconformal Mappings, Riemann Surfaces, and Teichmuller Spaces, Comtemporary Mathematics, 575 (2012), 53-76.
  • [10] A. Fletcher, R. Fryer, Dynamics of quasiregular mappings with constant complex dilatation, Ergod. Th. Dynam. Sys., 36, no.2 (2016), 514-549.
  • [11] A. Fletcher, D. Goodman, Quasiregular mappings of polynomial type in 2superscript2\mathbb{R}^{2}blackboard_R start_POSTSUPERSCRIPT 2 end_POSTSUPERSCRIPT, Conform. Geom. Dyn., 14 (2010), 322-336.
  • [12] P. Hartman, On local homeomorphisms of Euclidean spaces, Bol. Soc. Mat. Mexicana, 5, no.2 (1960), 220-241.
  • [13] T. Iwaniec, G. J. Martin, Geometric Function Theory and Non-linear Analysis, Oxford University Press, 2001.
  • [14] J. Milnor, Dynamics in one complex variable, Ann. of Math. Stud., 160, Princeton University Press, 2006.
  • [15] S. Rickman, Quasiregular Mappings, Ergebnisse der Mathematik und ihrer Grenzgebiete 26, Springer, 1993.
  • [16] D. Sullivan, Quasiconformal homeomorphisms and dynamics, I, Solution of the Fatou-Julia problem on wandering domains, Ann. of Math., 122 (1985), 401-418.